You are on page 1of 68

NCM 109 IMCI 8.

Nurse Yahosh is managing a 2-week-old sick


infant with pus on the umbilicus and as part of
MIDTERMS routine assessment, he documents a rectal
temperature of 37.5 C. The nurse would then
1. Given a yellow classification, the nurse would do
document this as what classification on his case
the following except:
recording form?
a. Advice the caregiver how to treat local
. Very Severe Disease
infections at home
a. Local Bacterial Infection
b. Give oral drugs
b. Severe Disease
c. Follow up – give specific date
c. Local Infection Unlikely
d. Treat the local infection in the health
center • Correct answer: a. Very
Severe Disease
• Correct answer: b. Give oral
9. In what part of the IMCI guidelines will the health
drugs
care provider re-examine conditions to see if the
2. When assessing all sick infants and children,
issues are improving, the same or worsening?
which of the following are being assessed?
. Counsel
. Signs of serious illness and common health
a. Provide Follow-up Care
conditions
b. Treat
a. Signs of Nutritional status and immunization
c. Classify
status
b. All except A. • Correct answer: Provide
c. Both A and B. Follow-up Care
10. In the management of childhood illness, the
• Correct answer: Both A and
usage of good communication skills is essential.
B.
The student nurse is learning these skills
3. The meaning of IMCI is:
because of the following reasons, except:
. Integrated Management of Childhood Illness
. Helps you get important information or
a. International Management of Childhood
assessment
Illnesses
a. Helps to reassure the caregiver that their child
b. Integrated Management for Childhood Illness
will receive good care
c. International Management for Childhood Illness
b. Helps you counsel and teach a caregiver on
• Correct answer: Integrated
care
Management of Childhood
c. Helps you support the traditional way of treating
Illness
an illness
4. The rectal temperature is at 37.5 C and there is
• Correct answer: d. Helps you
a draining pus in the umbilicus of the infant.
support the traditional way of
Upon identifying the classification, what will be
treating an illness
the first step of management will the nurse follow
11. Three-year-old Alora was brought by her mother
according to IMCI guidelines?
to the health center for consultation due to
. Give first dose of intramuscular antibiotics.
coughing for 7 days. Upon arrival, the nurse
a. Give an oral antibiotic.
further assessed the child for fast breathing,
b. Advise the mother to give home care.
chest indrawing and abnormal breath sounds.
c. Refer the patient to the hospital urgently.
Which of the following assessment findings will
• Correct answer: a. Give first
help the nurse choose a management that
dose of intramuscular
includes oral antibiotics, safe remedy to soothe
antibiotics.
throat, and relieve cough? *
5. Which of the following signs and symptoms will
. Coughing for 7 days
not signify a yellow or green classification of the
a. Respiratory rate of 36 breaths per minute
illness?
b. Lethargy
. Jaundice appearing after 24 hours of age
c. Chest indrawing in a calm child
a. Restless and irritable
• Correct answer: d.
b. Positive serological HIV test in the young infant
12. The APAC process is a reminder of important
c. Skin pinch goes back very slowly
communication skills that a nurse should use
• Correct answer: Skin pinch
every time they see a patient and caregiver.
goes back very slowly
Which part of the process will the nurse say,
6. In caring for a sick child with convulsions at the
“How often should you breastfeed your child?”
time of the initial visit, what is the initial nursing
. Ask and Listen
management to be done according to IMCI
a. Check understanding
guidelines?
b. Praise
. Quickly complete the assessment
c. Advise
a. Keep the child warm
• Correct answer: Check
b. Give diazepam injection solution via rectum
understanding
through a needleless syringe
13. The following cases are part of the limitations of
c. Urgently refer the patient to a hospital
using IMCI, except:
• Correct answer: Give
. Chronic heart diseases
diazepam injection solution
a. Severe Malnutrition
via rectum through a
b. Pneumonia
needleless syringe
c. Leukemia
7. In what part of the IMCI guidelines will the health
• Correct answer: Pneumonia
care provider identify or develop an integrated
14. A nurse will be giving the initial dose of
treatment plan?
Ampicillin via intramuscular route to a sick young
. Counsel
infant that has been classified to have a “Very
a. Provide Follow-up Care
Severe Disease”. If the Ampicillin preparation is
b. Treat
250 mg/1.5 ml, dosage is 50 mg per kilogram
c. Classify
and the patient’s weight is 3.5 kgs. How many
• Correct answer: Treat
milliliters of medication will the nurse administer b. Provide advice on counseling/follow up of the
if the infant needs 183 mg of Ampicillin? patient with severe classifications.
. 1.0 ml c. Instructions on how to classify the severity of the
a. 1.1 ml condition.
b. 1.2 ml • Correct answer: Instructions
c. 1.5 ml on how to classify the
• Correct answer: b. 1.1 ml severity of the condition.
15. Nurse Bie has identified “Pneumonia” as a 21. The following are main symptoms of the IMCI
classification for a 1-year-old patient who came case management process for a sick child,
to the health center due to coughing and fast except?
breathing. After giving appropriate nursing . Cough and difficulty breathing
interventions, the nurse would counsel the a. Immunization Status
mother to come back for a follow up visit. When b. HIV Status
will the patient and caregiver comeback if the c. Jaundice
initial visit was yesterday, March 23rd? • Correct answer: Jaundice
. March 29, 2021 22. Which part of the IMCI Case management
a. March 24, 2021 process will the nurse advise the caregiver to
b. March 26, 2021 return to a specific date for a follow up?
c. March 25, 2021 . Provide Follow Up Care
• Correct answer: March 26, a. Counsel
2021 b. Treat
16. What is the interval of the administration of c. Classify
inhaled salbutamol after every 2 puffs? • Correct answer: Counsel
. Every 3 minutes 23. For a 2-year-old patient that needs to be
a. Every 10 minutes administered with an ampicillin, which among
b. Every 15 minutes the following dosage and frequency is true?
c. Every 30 minutes . 3 ml via IM, once prior to referral
• Correct answer: Every 15 a. 5 ml via IM, once prior to referral
minutes b. 2 ml vial IM, every 6 hours
17. After 6 weeks, the infant will be given c. 5 ml via IM, every 6 hours
Pentavalent Vaccine 1, all but one are included • Correct answer: 3 ml via IM,
in this immunization? once prior to referral
. Polio immunization 24. When teaching a mother about cup feeding,
a. Hepatitis B immunization which of the following are true?
b. Diphtheria-Pertussis-Tetanus immunizations i. Put a cloth on the infant’s front to protect his
c. Haemophilus influenzae type B immunization clothes from spillage.
• Correct answer: Polio ii. Hold the infant straight on the lap.
immunization iii. Put a very small amount of milk in the cup.
18. The nurse found out that the mother gives iv. Hold the cup so that it rests lightly on the
“Bearbrand” with boiled rice water and infant’s upper lip.
sometimes, coffee as an alternative food aside v. Tip the cup so that the milk just reaches the
from breastmilk to her one-week-old infant. What infant’s lips.
would be the best approach of the nurse to the vi. Allow the infant to take the milk himself.
caregiver? vii. Try to pour the milk in the infant’s mouth.
. Praise the mother genuinely and encourage the a. i, iii, v, vi,
caregiver to give more breast milk than rice water with b. i, ii, iv, v, vi
milk formula. c. i, ii, iii, iv, v, vi, vii
a. Correct the practice and emphasize that d. i, v, vi
exclusive breastfeeding should be done until 6 months • Correct answer: i, v, vi
from birth. 25. If the gentamicin ampule contains 40 mg/ml and
b. Praise the caregiver and teach the caregiver it has 2 ml per ampule, how many milligrams of
proper feeding through a cup. gentamicin will the nurse administer to the child if she
c. Ask the caregiver good checking questions on will inject 1.0ml? *
proper breastfeeding. a. 80 mg
• Correct answer: Correct the b. 60 mg
practice and emphasize that c. 40 mg
exclusive breastfeeding d. 20 mg
should be done until 6 • Correct answer: 40 mg
months from birth. 26. The benefit of using integrated management in
19. Which of the following assessment findings in a treating childhood illness is a guidance to the health care
sick child will the nurse identify being under the provider through comprehensive care. The following the
pink classification? key points about the guidelines of IMCI, except:
. Stridor in a calm child . The charts helps classify a problem
a. Fast breathing a. The charts identify treatment and tells where to
b. Severe chest indrawing treat the child
c. Coughing for more than 14 days b. It helps us assess all children only.
• Correct answer: a. Stridor in a c. It helps us manage all common causes of illness
calm child and death.
20. Integrated management on IMCI include: • Correct answer: It helps us
. Assessment of signs and symptoms for patients assess all children only.
6 years old below. 27. On the other hand, Nurse Kay identified a
a. Treatment of serious and urgent conditions with different patient with “Cough and Cold”. After giving
the use of advanced technology. home instructions, when will the nurse counsel the
caregiver for a follow up visit if the initial visit was 33. The following are main symptoms included in the
Monday, March 22nd? management of a sick young infant, except for one?
. March 24, 2021 . Cough and difficulty of breathing
a. March 25, 2021 a. Jaundice
b. March 26, 2021 b. Diarrhea
c. March 29, 2021 c. Feeding problem or low weight for age
• Correct answer: March 25, • Correct answer: Cough and
2021 difficulty of breathing
28. Which of the following instructions to the 34. Which of the following is not included in the pink
caregiver about treating umbilical infection is correct? classification for IMCI?
i. Wash hands before and after treatment . Advice the caregiver
ii. Treatment should be done three times daily a. Give pre referral treatment
for 3 days b. Follow up care
iii. Gently wash off pus and crusts with soap and c. Refer to a patient to a higher level health facility
water • Correct answer: Follow up
iv. Dry the area after cleaning care
v. Treatment should be done twice daily for 5 35. In giving oral Amoxicillin via suspension to a 3-
days year-old patient with “Pneumonia”, the nurse will
vi. Apply 70% alcohol to the umbilicus administer the following correctly.
vii. Apply Mupirocin cream BID to the . Amoxicillin Suspension 15 ml BID for 5 days
umbilicus/cord until dry a. Amoxicillin Suspension 10 ml BID for 5 days
a. i, ii, iii, iv, v, vii b. Amoxicillin Suspension 20 ml BID for 3 days
b. i, ii, iii, iv, v, vi c. Amoxicillin Suspension 10 ml BID for 7 days
c. i, iii, iv, v, vii • Correct answer: Amoxicillin
d. i, iii, iv, v, vi Suspension 15 ml BID for 5
• Correct answer: i, iii, iv, v, days
vii 36. Mortality is higher in children living in the poorest
29. After greeting the caregiver upon arrival, the households, in rural areas or with mother that have little
following are the true in the next steps of the health care education. Which among the following factors is not
provider, except for one: included? *
a. Ask the infant/child’s age . Poor access to health facilities
b. Ask for the weight and temperature. a. Continued application of the traditional
c. Ask the child’s problems medicine
d. Ask if it is initial or a follow up visit b. Not enough food from farming
• Correct answer: Ask for the c. Lack of proper waste disposal
weight and temperature. • Correct answer: b. Continued
30. Which statement is true when giving inhaled application of the traditional
salbutamol to a sick child that has wheezing? medicine (?)
. Give salbutamol through a metered dose inhaler. 37. When a caregiver’s concern is on what container
a. There are 200 micrograms of salbutamol per will she be using to measure 1 liter of water for mixing
puff. ORS, which statement of the nurse is correct? *
b. The child must breathe in and out through the . “Thank you for telling me that but you should
nose. only use a one liter container.”
c. Wait for three to four breaths before a. “I see your concern. What containers do you
administering another puff. have at home?”
• Correct answer: a. Give b. “Do you have any more concerns?”
salbutamol through a c. “I see. Do you understand my instructions?”
metered dose inhaler. • Correct answer: “I see your
31. After giving health instructions on a safe remedy concern. What containers
to soothe the throat and relieve cough, which among the do you have at home?”
phrases indicate the use of good checking questions? 38. After identifying that the sick child has a “Very
. “Do you remember how to make Lagundi Severe Disease”, the nurse would then administer
syrup/concoction?” appropriate antibiotics prior to referral to a hospital. What
a. “Can you tell me the different ways on how to is the second line or alternate drug for this
safely relieve the cough of your child?” classification?
b. “Why is it important for you to wash your . Ampicillin and Gentamicin
hands?” a. Amoxicillin
c. “How many teaspoons of cough syrup will you b. Benzyl Penicillin
administer to your child?” c. Cotrimoxazole
• Correct answer: d. “How • Correct answer: a. Ampicillin
many teaspoons of cough and Gentamicin
syrup will you administer to 39. Which of the following is not a reason why IMCI
your child?” was created? *
32. Which of the following data supports a . Health care providers use state of the art
classification of “Confirmed HIV Infection” among sick facilities including radiology equipment to diagnose
young infants? multiple conditions
. Mother HIV positive, young infant is not yet a. First level health facilities do not have many
tested diagnostic tools.
a. Positive serological test in the young infant b. Health centers have limited medical supplies
b. Negative HIV test in mother and young infant and human resources.
c. Positive virological test in the young infant c. IMCI integrates case management of the most
• Correct answer: Positive common childhood problems
virological test in the young • Correct answer: Health care
infant providers use state of the
art facilities including a. “One of the goals of IMCI is to reduce how often
radiology equipment to children are sick.”
diagnose multiple b. “One of the goals of IMCI is to decrease the
conditions severity of the illness.”
40. For this alternate or second line medication for a c. “One of the goals of IMCI is to improve the
"Very Severe Disease" for a sick child, what is the growth and development among children.”
correct amount of diluent? • Correct answer: “One of the
. No diluent needed goals of IMCI is to increase
a. 2.1 ml of sterile water childhood mortality.”
b. 5 ml of sterile water 46. Nurse Doc has given oral Amoxicillin to the sick
c. 8 ml of sterile water child with Pneumonia in the RHU and instructed the
• Correct answer: 8 ml of sterile mother about its succeeding doses to be continued at
water home. He also counseled the mother to soothe the
41. Among the three basic teaching steps on throat and relieve the coughing with a safe remedy.
teaching a caregiver how to treat a child, which step is Which IMCI classification did the nurse follow?
the most important since the nurse will know what the . Red
caregiver understands about the home instructions? a. Pink
. Give information b. Yellow
a. Let caregiver practice c. Green
b. Show an example • Correct answer: Yellow
c. All of the above 47. Lola is a mother of a 1-year-old child who is
• Correct answer: Let caregiver suffering from difficulty of breathing and malnutrition.
practice She had finished her high school degree and is currently
42. A mother has had problems breastfeeding her living in an urban area. Her husband works as a
newborn infant, had only 7 breast feedings for the past construction worker that makes 350php per day for a
24 hours and thus bringing her infant to the nearest family of five (ages 1, 2 and 3). The father goes to work
health center. The nurse then classifies this as “Feeding walking for how many miles or sometimes, rides a
Problem”. In teaching correct positioning and jeepney, if able. Given the situation above, which factors
attachment, which of the following statements from the affect the health of their 1-year-old sick child?
caregiver will signify correct teaching from the nurse? i. Below-poverty line household
. “My baby should have slow, deep sucks and ii. Educational attainment
sometimes pausing.” iii. Residence
a. “My baby’s chin should be touching the areola of iv. Mother’s education
my breast.” v. Mode of transportation
b. “My baby’s upper lip should be turned outwards vi. Food insecurity
when sucking.” vii. Lack of basic resources
c. “My baby’s mouth should be partially open a. i,ii,iii,iv
during sucking.” b. vii,ii,iii
• Correct answer: a. “My baby c. i,iv,vi
should have slow, deep d. i, vi, vii
sucks and sometimes • Correct answer: i, vi, vii
pausing.” (?) 48. All phrases from the caregiver indicates
43. Nurse Keon is teaching the mother in preventing understanding of the counsel given by the nurse in
low blood sugar in a conscious young infant with “Severe giving home care for the young infant, except for one:
Jaundice”. The best way to prevent low blood sugar is a. “Since my child has low weight for age, we will
through breastfeeding but the mother has never come back after 30 days for follow up.”
produced breast milk even with an OB doctor’s help. b. “I will breastfeed my baby as frequently, as often
What would be the nurse’s correct option? as possible and as long as the infant wants.”
. Give 20-50 ml of sugar water (four tablespoons c. “I will make sure that my baby’s head, feet and
of sugar: 200 ml of clean water) orally body are covered with extra clothing during cool
a. Give 20-40 ml of sugar water (four teaspoons of weather.”
sugar: 100 ml of clean water) via NGT d. “We will come back to the health center
b. Give 20-50 ml of sugar water (four teaspoons of immediately if my baby is breastfeeding poorly and has a
sugar: 200 ml of clean water) orally reduced activity.”
c. Give 20-40 ml of sugar water (four tablespoons: • Correct answer: “Since my
100 ml of clean water) via NGT child has low weight for
• Correct answer: Give 20-50 ml age, we will come back after
of sugar water (four 30 days for follow up.”
teaspoons of sugar: 200 ml 49. A patient has been rushed to the Rural Health
of clean water) orally Center due to difficulty of breathing and severe chest
44. The following are major preventable causes of indrawing. The patient has just turned 2 months of age
child mortality worldwide, except: today. The infant suddenly loses his consciousness.
. Malnutrition What case recording form and IMCI chart booklet section
a. Pneumonia will the nurse use?
b. Congenital abnormalities . Management of a Sick Child: Very Severe
c. Diarrhea Disease
• Correct answer: Congenital a. Management of a Sick Young Infant: Very
abnormalities Severe Disease
45. Which statement will cause the clinical instructor b. Management of a Sick Child: Pneumonia
to correct the nursing student when it comes to the c. Management of a Sick Young Infant: Local
overall goal of IMCI? Bacterial Infection
. “One of the goals of IMCI is to increase • Correct answer: a.
childhood mortality.” Management of a Sick
Child: Very Severe Disease
50. Which of the identified cases will the nurse within 1 second. However, if the skin pinch goes back
consider as a general danger sign? more than 2 seconds, how will the nurse interpret
. A newborn infant 12 hours ago, who developed these findings?
jaundice. • Child is not dehydrated.
a. A 1-month old infant with 50 breaths per minute. • Child has Kwashiorkor.
b. A 3-year-old girl who vomited her last meal. • Child has severe dehydration.
c. A 4-year-old boy who refuses to drink water but • Child has some dehydration.
would like to eat.
• Correct answer: A newborn Correct answer: Child has severe dehydration.
infant 12 hours ago, who
developed jaundice.
5. In planning for the treatment and management,
which is the nurse’s priority?
FINALS
• Refer the child immediately for hospital
Scenario #1: Raya is at the rural health unit today due management.
to diarrhea for 15 days according to her mother. She • Give a measured amount of ORS, monitored
is a one year old girl. She weighs 10 kilograms and for 4 hours at the health center.
her temperature is at 37.7 C at the time of visit. But • Start IV fluids immediately in the health center.
according to the mother, her fever has been going on • Give zinc supplements and antibiotics as
and off for a week now. The family has no travel prescribed.
history for the past 3 weeks. Raya does not have any
general danger signs. She does not have any cough Correct answer: Give a measured amount of ORS,
or difficulty breathing. After asking several questions, monitored for 4 hours at the health center.
the nurse proceeds to assess Raya for dehydration
and finds out that she is restless and irritable but her 6. What is the purpose of giving zinc supplements to
eyes are not sunken. When offered a drink, Raya a patient with dehydration and diarrhea?
eagerly drinks it and consumes it immediately. • To reduce the duration and severity of the
diarrhea.
1. Prior to the assessment of diarrhea, what has been • To treat dehydration and malnutrition.
assessed from the patient according to IMCI • To strengthen the child’s immune system against
guidelines? bacteria.
• Fever • To increase the stool output and decrease the
• Cough and difficulty of breathing need for hospitalization.
• General danger signs
• Dysentery Correct answer: To reduce the duration and severity
of the diarrhea
Correct answer: cough and diff

7. As part of the treatment plan, the nurse has to


2. What is the classification that the nurse has to compute the approximate amount of ORS. What is the
write on her patient’s care recording form according needed amount to be consumed given the data above?
to her assessments regarding diarrhea? • 200 ml
• Severe Dehydration with severe persistent • 500 ml
diarrhea • 750 ml
• Some Dehydration with persistent diarrhea • 960 ml
• Some Dehydration with severe persistent
diarrhea Correct answer: 750ml
• No Dehydration

Correct answer: Some Dehydration with severe 8. When giving health instructions on follow up care,
persistent diarrhea when would the nurse schedule the follow up checkup
for the child and caregiver when the date of the initial
visit is today, May 20, 2021?
3. What is persistent diarrhea in Raya’s case? • May 21, 2021
• When a child has several episodes of diarrhea • May 24, 2021
accompanied by high temperature a day • May 25, 2021
• When a child frequently has bouts of loose bowel • No follow up checkup at the health center.
movements for more than a week with signs of
dehydration Correct answer: May 25, 2021
• When a child has an episode of diarrhea
lasting more than two weeks, which may lead 9. After giving health instructions to the caregiver,
to dehydration and malnutrition the patient came back the next day saying that blood in
• When a child frequently has diarrhea over a the stool had developed. What is the drug of choice and
period of one month, and is ill as a result. the right frequency of this classification according to the
IMCI guidelines?
Correct answer: When a child has an episode of • Give cotrimoxazole for 5 days.
diarrhea lasting more than two weeks, which may lead • Give oral amoxicillin for 5 days.
to dehydration and malnutrition • Give ciprofloxacin for 3 days.
• No medications required since this is normal for
patients with diarrhea.

4. When the nurse starts to pinch the skin of the Correct answer: Give ciprofloxacin for 3 days.
abdomen for further assessment, the skin goes back
10. The following are correct statements from the • “I will not give another dose of Vitamin A to the
nurse when instructing caregiver on treating diarrhea patient since she does not have any measles.”
at home, except: • “I will give 200,000 IU of Vitamin A as part of
• “Give your child frequent sips of water, ORS or the treatment plan.”
breastfeed your baby as much as she would
take.” Correct answer: “I will give 200,000 IU of Vitamin A as
• “If your child vomits, wait for 5 minutes then part of the treatment plan.”
continue giving ORS but this time, more
slowly.” 15. With a positive malaria test for Plasmodium
• “Please give your child 50 to 100 ml of fluids after falciparum, the nurse would consider giving anti-malarial
each loose stool.” medications as part of the treatment plan. What is the
• “Dissolve one packet of ORS to 250ml of clean correct drug combination for this type of malaria?
water or according to package instructions” • Chloroquine (Day 1 to 3) and primaquine (Day 4)
• Chloroquine (Day 1 to 3) and primaquine (Day 4-
Correct answer: “If your child vomits, wait for 5 17)
minutes then continue giving ORS but this time, more • Artemether-lumefantrine (Day 1 to 3) and
slowly.” primaquine (Day 4)
• Artemether-lumefantrine (Day 1 to 3) and
Scenario #2: A mother came to the clinic with her 2 chloroquine (Day 4)
years old daughter, Rabiya. The mother said that her
child has been feeling feverish for the past two days. Correct answer: Artemether-lumefantrine (Day 1 to 3)
She cannot get the exact temperature since they did and primaquine (Day 4)
not have any thermometer at home. Along with the
fever, the nurse noticed the child’s red eyes and 16. On the other hand, if the patient is tested positive
generalized rash. The nurse then assesses the child for Plasmodium vivax, the correct drug combination for
for possible malaria, dengue and ear problems. this type of malaria?
• Chloroquine (Day 1 to 3) and primaquine (Day 4)
11. In a different scenario, supposing there is no • Chloroquine (Day 1 to 3) and primaquine (Day
generalized rash or red eyes on the assessment findings. 4-17)
When the nurse asks about travel history or living • Artemether-lumefantrine (Day 1 to 3) and
situation, the mother confidently says that they did not primaquine (Day 4)
have any travel history for the past 3 weeks and there are • Artemether-lumefantrine (Day 1 to 3) and
no cases of malaria in their area. What would the nurse chloroquine (Day 4)
write as a classification on the case recording form?
• Malaria Correct answer: Chloroquine (Day 1 to 3) and
• Measles with eye or mouth complications primaquine (Day 4-17)
• Fever: No Malaria
• Fever 17. The following are indications that the patient has
severe dengue hemorrhagic fever, except:
Correct answer: Fever • Bleeding from the nose or gums
• Skin petechiae of more than 15 on the
12. After several assessments of the nurse, Rabiya’s antecubital fossa after a Rumpel-Leede’s test
temperature via axillary is 39 C. There is no stiff neck • Capillary refill of more than 3 seconds
and any form of bleeding. What is the nurse’s next • Persistent vomiting
step upon identifying the classification according to
IMCI guidelines? Correct answer: Skin petechiae of more than 15 on
• Apply a tepid sponge bath to the child. the antecubital fossa after a Rumpel-Leede’s test
• Give one dose of paracetamol in the health
center. 18. The following statements from the caregiver need
• Give Vitamin A treatment. correction from the nurse regarding health teachings on
• Give appropriate antibiotics. dengue hemorrhagic fever, except?
• “I will follow up in 5 days if my child’s fever
Correct answer: Give one dose of paracetamol in the persists or there are signs of bleeding.”
health center. • “When my child’s temperature is 38. 5 C or above,
I could give aspirin to reduce pain and fever.”
13. Given the situation above, what classification will • “I will give an oral rehydration solution to
the nurse write on the initial visit case recording form for prevent dehydration.”
patient Rabiya? • “It is okay to give chocolate prevent hypoglycemia
• Fever and green leafy vegetables to strengthen the
• Measles immune system.”
• Measles with eye or mouth complications
• Very severe febrile disease Correct answer: “I will give an oral rehydration
solution to prevent dehydration.”
Correct answer: Very severe febrile disease
19. When assessing the patient for ear problems, the
14. During the treatment plan, the nurse will be giving nurse knows that an ear infection reported for less than
a dose of Vitamin A for the child. What is a correct two weeks is called?
statement from the nurse? • Acute ear infection
• “I will not give another dose of Vitamin A to the • Mastoiditis
patient since she had her recent dosage 4 weeks • Chronic ear infection
ago.” • Middle ear infection
• “I will give 100,000 IU of Vitamin A since she is
more than 3 years old.” Correct answer: Acute ear infection
25. If the mid-upper arm circumference is 110cm,
what does this finding tell the nurse?
20. What is the appropriate antibiotic for this type of • Georgia has severe acute malnutrition.
ear infection on a 2-year-old child? • Georgia is healthy.
• Cotrimoxazole 2.5ml BID for 3 days • Georgia has moderate acute malnutrition and
• Ciprofloxacin 500mg BID for 3 days needs advice on feeding recommendations.
• Amoxicillin 10ml BID for 5 days • Georgia should be referred urgently to a hospital.
• Ampicillin Sodium 3ml and Gentamicin 2.0 ml via
IM route, one dose each Correct answer: Georgia has severe acute
malnutrition.
Correct answer: Amoxicillin 10ml BID for 5 days

Scenario #3: Georgia is a 10-month-old child with a 26. With Georgia’s findings on the palmar pallor, her
weight of 8 kilograms and a length of 85 cm. His additional classification to be written in the case recording
temperature via axillary is 37 C. There is no bipedal form is identified as:
edema, WFL z-score is -3, the mid upper arm • No anemia
circumference is 110cm. During the appetite test, he • Anemia
was able to finish ¾ of the RUTF. There is no medical • Severe anemia
complication present. His palms are paler than that of • None of the above
the nurse’s palms.
Correct answer: Anemia
21. With the given situation above, what is the
classification of Georgia’s problem that the nurse should
write on the case recording form? 27. Upon identifying the classification of palmar
• Moderate acute malnutrition pallor, what should be part of the treatment plan of the
• Uncomplicated severe acute malnutrition nurse?
• No acute malnutrition • Refer the patient urgently to the hospital.
• Complicated severe acute malnutrition • Give iron and anti-helminthic medication.
• Advise the caregiver on feeding
Correct answer: Uncomplicated severe acute recommendations.
malnutrition • No treatment plan needed.

Correct answer: Give iron and anti-helminthic


22. Which of the following are part of the nurse’s medication.
treatment plan?
i.Give the first dose of appropriate antibiotics.
ii.Give oral amoxicillin for 5 days 28. One of the causes of anemia is the presence
iii.Give RUTF if available. parasites including hookworms or whipworms. As part of
iv.Assess for possible TB infection. the treatment plan, health care providers give specific
v.Keep the child warm. medications for these cases. Which statement is true
vi.Treat the child to prevent low blood sugar. when giving this medication?
vii.Follow up in 30 days. • Give oral quinine 300 mg as a single dose.
viii.Give feeding recommendations. • Give mebendazole 500mg as a single dose.
ix.Give micronutrient powder supplement. • Give tetracycline ointment four times daily.
x.Refer urgently to the hospital
• Give ferrous fumarate 100mg once daily.
• ii, iii, iv, vii
• i, v, vi, x Correct answer: Give mebendazole 500mg as a single
• ii, iii, iv dose.
• ii, iv, viii, ix, vi

Correct answer: ii, iii, iv


29. How many days will the nurse instruct the
caregiver in giving iron supplements to a patient with
23. Which criterion belongs to the classification of “No uncomplicated severe acute malnutrition?
acute malnutrition”? • 14 days
• WFL between -3 and -2 z-scores • 30 days
• MUAC of 130mm • 7 days
• MUAC of 120 and was able to finish RUTF portion • Not anymore since the patient is receiving
• WFH of less than -3 z-score RUTF.

Correct answer: MUAC of 130mm Correct answer: Not anymore since the patient is
receiving RUTF.

24. Georgia shows clinical signs of marasmus, which


of the following is not a sign of this condition?
• Extremely thin body 30. Which is true in the following health instructions
• Distented belly from a nurse about micronutrient powder?
• Swelling of ankles and feet • MNP is given before meals to enhance
• Lack of fat on buttocks and arms absorption.
• Add MNP to food before and during cooking.
Correct answer: Swelling of ankles and feet • MNP is given to children 3 months and up.
• Mix MNP into complementary food before
feeding.
36. If Alex had been confirmed infected with HIV, how
Correct answer: Mix MNP into complementary food will she take an anti-tuberculosis prophylaxis if her weight
before feeding. is 5 kilograms?
• 300mg for 12 months
Scenario #4: Alex was born a month ago from an HIV • 100mg for 6 months
positive mother. A serological test has been done to • 50 mg for 6 months
the baby with a positive result. Since the mother has • 30 mg for 12 months
been on antiretroviral treatment medications for
months, she chose to exclusively breastfeed Alex. Correct answer: 50 mg for 6 months

31. On the initial follow up at the health center, what


diagnosis will the nurse write on the case recording form 37. According to WHO Pediatric Staging for HIV
for baby Alex? Infection, when a patient develops enlarged liver or
• Confirmed HIV infection spleen and recurrent sinusitis, ear infection and tonsillitis,
• HIV exposed what stage will we consider these clinical signs?
• HIV infection • Asymptomatic
• HIV infection unlikely • Mild Disease
• Moderate Disease
Correct answer: HIV exposed • Severe Disease/AIDS

Correct answer: Mild Disease


32. With the knowledge of mother-to-child-
transmission (MTCT), what is the overall risk of a mother
transmitting HIV to her child during breastfeeding if 38. How would the nurse counsel Alex’s mother with
nothing has been done to prevent HIV transmission regards to recommended feeding practice if the child has
including ART prophylaxis? not yet been confirmed with HIV infection?
• 35% • Exclusive breastfeeding until 12 months of life
• 20% • Breastfeeding and also formula feeding, in order
• 15% to provide additional nutrition
• 7% • Commercial infant formula only to avoid HIV
transmission
Correct answer: 15% • Exclusive breastfeeding for a 6 weeks

Correct answer:
33. To ensure that Alex is either a confirmed HIV
infected or an HIV negative, what would the nurse do if
there are no available HIV PCR tests? 39. Which of the following statements regarding HIV
• Instruct the mother to stop breastfeeding for 6 is true?
weeks before a repeat serological test. • HIV attacks CD5 cells and uses it to replicate.
• Instruct the mother to continue breastfeeding and • HIV can usually weaken or destroy the immune
repeat serological test after 18 weeks. system much for rapidly in adults than in children.
• Repeat serological test immediately. • Exclusive breastfeeding increases the risk of HIV
• Give ART treatment regardless of the result. transmission.
• • HIV is considered to have progressed to AIDS
when serious sicknesses occur.

Correct answer: Correct answer: HIV is considered to have


progressed to AIDS when serious sicknesses occur.

34. The following prophylaxis will be part of the


management plan for Alex, except?
• Ciprofloxacin 40. When does a child eligible for ART treatment?
• Nevirapine and Zidovudine • Positive serological test in a 3-year-old child.
• Isoniazid • Negative virological test in a 2-year-old child.
• Trimethoprim-sulfamethoxazole • Negative HIV tests in a 4-year-old child.
• No tests available yet for the child.
Correct answer: Ciprofloxacin
Correct answer: Positive serological test in a 3-year-
old child.

35. Which of the following classification and age Care of a Well-Child


qualifies for antibiotic prophylaxis to start as soon as
possible? 41. How often should infants/children below 6 months
• 6 years of age old breastfeed?
• a 1 month old, confirmed HIV infection • At least 10 times in 48 hours
• 2 years old, HIV Exposed • At least 6 times in 24 hours
• 3 years old , confirmed HIV infection • At least 12 times in 48 hours
• At least 8 times in 24 hours
Correct answer: 2 years old, HIV Exposed
Correct answer: At least 8 times in 24 hours

42. Which of the following situations need counseling


from a health care provider to address feeding problems?
• Mother feeds 2-month-old child breastmilk “I will not let my children get medications for their
from a cup. grandmother’s use.”
• Mother feeds mashed potatoes with
breastmilk to her 5-month-old child. Correct answer: “I can hold my infant while I am
• Mother gives 18 months old child his own serving cooking.”
of food during dinner time.
• Mother hears slow, deep sucks from her child 48. Another aspect that we asses on a well-child is
who is breastfeeding. their growth and development. Which defines child
growth?
Correct answer: Mother feeds mashed potatoes with • Gradual unfolding of capacities including learning
breastmilk to her 5-month-old child. how to walk and talk.
• Increase in physical size, composition and
43. The nurse assesses the vaccination status of a 7- distribution of tissue.
week-old child who visited the health center. She then • The capacity to explore things around them
asks for the child health book from the mother. As she • Learning how to communicate interests and
compares the child’s vaccination record and the needs
recommended schedule, she notices that there are
vaccinations not yet given except for the initial vaccination Correct answer: Increase in physical size,
given at birth. What will the nurse administer at this time? composition and distribution of tissue.
• Measles Vaccine
• BCG and Hepatitis B vaccines 49. The following are used to measure child growth,
• Pentavalent 1, PCV 1, Rotavirus 1 and OPV 1 except:
• Pentavalent 2, PCV 2, Rotavirus 2 and OPV 2 • Weight-for-age
• Length/height-for-age
Correct answer: Pentavalent 1, PCV 1, Rotavirus 1 • Weight-for-length/height
and OPV 1 • Mid-upper arm circumference

44. When will the nurse administer measles vaccine Correct answer: Mid-upper arm circumference
to a child?
• At 10 weeks 50. What situation will the nurse use “length” as a
• At birth measurement to a child being assessed for growth?
• At 9 months • A 3-year-old child with some dehydration.
• At 14 weeks • A 5-year-old child with pneumonia
• A 4-year-old child with severe muscle wasting
Correct answer: At 9 months and cannot stand alone.
• A 2-year-old child who can stand by himself
45. The following children are not qualified to receive independently.
routine/specific immunizations, except:
• A child who is known to have AIDS scheduled for Correct answer: A 4-year-old child with severe
BCG muscle wasting and cannot stand alone.
• A child with uncomplicated severe acute
malnutrition.
• A child who had convulsions a day after the first
dose of Pentavalent vaccine. NCM 109 Lab (Myrtle and Leahbonds)
• A child who has active neurological disease like 1. The nurse explains to new parents that infants
epilepsy are at greater risk for fluid and electrolyte
imbalance than older children. Which parent
Correct answer: comment would indicate that further education is
needed?
46. Which of the following nurse’s action should be a. "Compared to an adult, an infant has
corrected? little body water for reserve."
• The nurse gives Vitamin A supplementation to b. "Infants maintain their temperature by
a 4-month-old child. losing heat through their heads."
• The nurse administers 200,000 IU Vitamin A to a c. "Infants have a higher metabolic rate
2-year-old sick child. than older children do."
• The nurse gives another dose of Vitamin A to a d. "Infants lose water through their skin,
child with measles who was given 6 weeks ago. and they have a larger proportion of skin
• The nurse administers 100,000 IU Vitamin A to a surface area than older children do."
9-month-old with dysentery. • Correct answer: b. "Infants
maintain their temperature
Correct answer: The nurse gives Vitamin A by losing heat through their
supplementation to a 4-month-old child. heads."
2. A parent asks the nurse why her children get
47. As part of the care of well children, health care fewer ear infections as they grow older. The
providers should give counseling regarding prevention of nurse bases on her answer on which aspects of
childhood accidents. The following statements of the the infant's Eustachian tube?
caregiver indicate understanding, except for one? . It is shorter, wider, and more horizontal than an
“I will not let my 4-year-old child caretaking my 1-year-old older child's Eustachian tube.
child when I am away for a while.” a. It is shorter, wider, and more diagonal than an
“I will watch my children closely whenever they swim in older child's Eustachian tube.
their inflatable pool.” b. It is shorter, narrower, and more diagonal than
“I can hold my infant while I am cooking.” an older child's Eustachian tube.
c. It is shorter, narrower, and more horizontal than two days. A pelvic exam reveals a closed cervix.
an older child's Eustachian tube. What type of abortion does this indicate?
• Correct answer: a. It is . Imminent
shorter, wider, and more a. Missed
horizontal than an older b. Threatened
child's Eustachian tube. c. Incomplete
3. An infant has acute otitis media. Which would be • Correct answer: c. Threatened
the most important instruction for the nurse to 8. The patient asks for information about ectopic
teach the parents? pregnancy. The nurse correctly responds by
. Administer acetaminophen to relieve discomfort. saying ectopic pregnancy is caused by: (Select
a. Keep the baby in a flat position during sleep. all that apply.)
b. Administer a decongestant. . Pelvic inflammatory disease (PID)
c. Place the baby to sleep with a pacifier. a. Endometriosis
• Correct answer: a. Administer b. Presence of an IUD
acetaminophen to relieve c. History of previous ectopic pregnancy
discomfort. • Correct answer:
4. An 8-year-old child is diagnosed with viral • Pelvic inflammatory disease
pneumonia and sent home from the clinic with (PID)
no antibiotic prescription. The symptoms • Endometriosis
worsen, and the child returns to the clinic a week • Presence of an IUD
later with signs of a higher fever, listlessness, • History of previous ectopic
and a harsh, productive cough. The child's pregnancy
mother states, "I knew a prescription for 9. The nurse is caring for a laboring patient with
antibiotics was needed." Which response by the Type I diabetes. What signs and symptoms
nurse is the most appropriate? would the nurse assess if hypoglycemia was
. "It is better to wait to make sure so we don't use suspected?
antibiotics unnecessarily. This approach also saves . Headache and anorexia
healthcare dollars.” a. Dry skin and blurred vision
a. "Antibiotics are not effective for viral pneumonia. b. Frequent urination and headache
Bacteria can grow later in the course of the illness, c. Diaphoresis and disorientation
requiring the need for antibiotics at that time." • Correct answer: d.
b. "You do not want to expose your child to Diaphoresis and
medication unnecessarily. Now it is necessary, because disorientation
it is bacterial pneumonia." 10. The nurse is providing prenatal care to an
c. "Sometimes we just do not know. I'm glad you asymptomatic HIV-infected patient. Which
came back in." nursing intervention should take priority?
• Correct answer: b. . Taking her temperature
"Antibiotics are not a. Performing a hearing test
effective for viral b. Performing a vision test
pneumonia. Bacteria can c. Assessing reflexes
grow later in the course of • Correct answer: a. Taking her
the illness, requiring the temperature
need for antibiotics at that 11. During delivery of a first twin, a very tight nuchal
time." cord is reduced from the baby’s neck by
5. A child is diagnosed with severe combined clamping and dividing it. After this, the second
immunodeficiency deficiency syndrome. The twin (as yet unborn) develops several fetal
nurse's priority interventions are directed toward distress. Of the following, what is the most likely
which objective? mechanism for the distress in the second twin?
. Prevention of infection . A twin-to-twin transfusion before birth
a. Maintenance of skin integrity a. The second twin may no longer be connected to
b. Management of body image concerns its placenta
c. Maintenance of cardiac function b. Placenta previa in the second twin
• Correct answer: a. Prevention c. Amniotic fluid embolism
of infection d. Uterine rupture
6. A patient at 36 weeks gestation is admitted to • Correct answer: b. The
the labor and birth unit. Her chief complaint is second twin may no longer
abdominal cramping with a sudden gush of clear be connected to its placenta
fluid. What is the priority nursing diagnosis for 12. Pelvic inflammatory disease (PID) occurs in
this prenatal patient? women because of which of the
. Risk for ineffective coping related to unknown following characteristics of the fallopian tube?
outcome of pregnancy. . It is a conduit from the peritoneal space to the
a. Knowledge deficit related to unfamiliarity with uterine cavity
loss of vaginal fluids. a. It is found in the utero-ovarian ligament
b. Risk for infection related to premature rupture of b. It has five separate parts
membranes. c. It is attached to the ipsilateral ovary by the
c. Impaired physical mobility related to strict bed mesosalpinx
rest. d. It is entirely extraperitoneal
• Correct answer: c. Risk for • Correct answer: a. It is a
infection related to conduit from the peritoneal
premature rupture of space to the uterine cavity
membranes. 13. A 47-year-old G0P0 patient presents with history
7. A prenatal patient at 16 weeks gestation of irregular menses, infertility, and currently
presents to the clinic with unexplained, bright red increasingly heavy bleeding when it occurs. Her
bleeding, cramping, and backache for the past examination is remarkable for obesity, mild
hypertension, and a clinical finding consistent c. Is classified as having persistent diarrhea
with polycystic ovarian syndrome (PCOS). An • Correct answer:
endometrial biopsy is done and shows • b. Is less than 2 years old
endometrial hyperplasia. Which of the following • c. Is classified as having
is its best histological description? anemia, very low weight and
. Endometrial glands scattered throughout an or growth faltering
atrophic-appearing uterine muscle 19. When a mother is advised to return to a health
a. Increased number of glands with a piling up of worker for her child, it is necessary to tell her
their cells and decreased intervening stroma when to return for a follow-up visit and when to
b. Tightly spiraled endometrial glands with return immediately. When to return for a follow-
eosinophilic cytoplasm surrounding the arterioles up visit depends on the child’s classification; for
c. Tortuous glands with a loose, edematous stroma example, in case of acute ear infection child
d. Endometrial glands surrounding a fibrovascular needs to return in 5 days. The mother needs to
stroma, often with a characteristic central blood vessel return to the clinic immediately if her child (select
• Correct answer: b. Increased all that apply):
number of glands with a . Is drinking eagerly
piling up of their cells and a. Is not able to drink or breastfeed
decreased intervening b. Develops fever
stroma c. Is coughing often
14. Which of the following pathologic features is d. Becomes sicker
most helpful in distinguishing complete • Correct answer:
hydatidiform mole from normal placenta? • b. Is not able to drink or
. Trophoblastic proliferation breastfeed
a. Absence of blood vessels • c. Develops fever
b. Hydropic degeneration of villi • e. Becomes sicker
c. Cellular atypia 20. If a child has had ear pain and pus draining from
d. Sex chromatin positivity the ear for 10 days, and no tender swelling
• Correct answer: b. Absence behind the ear, you will classify this child as
of blood vessels having:
15. During a pelvic examination on a patient that is . Acute ear infection
approximately 8 weeks’ gestation by dates and a. Chronic ear infection
pelvic examination, one adnexa is found to be b. Mastoiditis
slightly enlarged. This is most commonly due to c. Not enough signs to classify this child
which of the following? • Correct answer: a. Acute ear
. Corpus luteum cyst infection
a. Ectopic pregnancy 21. Early detection of an ectopic pregnancy is
b. Follicular cyst paramount in preventing a life- threatening
c. Ovarian neoplasm rupture. Which symptoms should alert the nurse
d. Parovarian cyst to the possibility of an ectopic pregnancy?
• Correct answer: a. Corpus . abdominal pain, vaginal bleeding ad positive
luteum cyst pregnancy test
16. What is a child’s classification if he is 10 months a. hyperemesis and weight loss
old, has had a cough that lasted two days, has a b. amenorrhea and negative pregnancy test
breathing rate of 46 breaths per minute and c. copious discharge of clear mucus and prolonged
chest indrawing? epigastric pain
. No pneumonia: cough or cold • Correct answer: a. abdominal
a. Pneumonia pain, vaginal bleeding ad
b. Severe pneumonia or very severe disease positive pregnancy test
c. Cough, diarrhea, fever, ear problem 22. All are the possible causes of ectopic
• Correct answer: c. Severe pregnancy, except?
pneumonia or very severe . history of surgery
disease a. history of PID
17. A 3-year-old child has fever and diarrhea. He b. light vaginal bleeding
has no danger signs and he does not have a c. congenital abnormality of fallopian tube
cough or difficult breathing. The child has had • Correct answer: c. light
diarrhea for 2 weeks and there is no blood in vaginal bleeding
stool. The child is restless and irritable, but is not 23. You are taking care of a client who is 35 weeks
drinking eagerly. His eyes are not sunken. A skin pregnant experiencing bleedinf caused by
pinch goes back slowly. He does not have an placenta previa. You monitored the fetal heart
ear problem. The child’s diarrhea should be sound, and it is normal, the client is not in labor.
classified as (select all that apply) Which nursing intervention should the nurse
. No dehydration perform?
a. Some dehydration . Monitor the amount of vaginal blood loss.
b. Severe dehydration a. Allow the client to ambulate with assistance.
c. Severe persistent diarrhea b. Perform a vaginal examination to check for
• Correct answer: cervical dilation.
• b. Some dehydration c. Notify the physician.
• d. Severe persistent diarrhea • Correct answer: a. Monitor
18. Feeding should be assessed in a child who the amount of vaginal blood
(select all that apply): loss.
. Need urgent referral 24. A nurse is caring for a client whose membranes
a. Is less than 2 years old ruptured prematurely 12 hours ago. When
b. Is classified as having anemia, very low weight assessing this client, the nurse’s highest priority
and or growth faltering is to evaluate?
. Cervical effacement and dilation. b. the patient will need to have a c-section and
a. White blood cell count. cannot deliver vaginally.
b. Maternal vital signs and fetal heart rate. c. the woman should report any bleeding
c. Frequency and duration of contraction. immediately to the doctor.
• Correct answer: c. Maternal • Correct answer: c. the patient
vital signs and fetal heart will need to have a c-section
rate. and cannot deliver
25. A nurse is providing care for a client who is vaginally.
diagnosed with a marginal abruptio placentae. 31. Which of the following would the nurse have
The nurse is aware that which of the following readily available for a client who is receiving
findings are the risk factors for developing the magnesium sulfate to treat severe
condition? Select all that apply. preeclampsia?
. Fetal position . Potassium chloride
a. Cocaine use a. Calcium gluconate
b. Blunt abdominal trauma b. Ferrous sulfate
c. cigarette smoking c. Calcium carbonate
• Correct answer: d. None of the above
• b. Cocaine use • Correct answer: b. Calcium
• c. Blunt abdominal trauma gluconate
• d. cigarette smoking 32. Your patient just gave birth to a baby. She was
26. A nurse is caring for a client that is at 36 weeks diagnosed with gestational diabetes throughout
of gestation with suspected placenta previa. her pregnancy. She has now normal blood
Which of the following manifestation should the glucose levels. When should the patient first
nurse expect? follow-up with her physician for blood glucose
. abdominal pain with scanty red vaginal bleeding testing?
a. increasing abdominal pain with a non-relaxed . 1-3 years
uterus. a. 6-12 weeks postpartum
b. painless red vaginal bleeding b. 1 year postpartum
c. intermittent abdominal pain following passage of c. Not applicable since this condition has resolved
bloody mucus. and only occurs during pregnancy
• Correct answer: c. painless d. None of the above
red vaginal bleeding • Correct answer: b. 6-12 weeks
27. The nurse is assessing a multigravida, 36 weeks postpartum
gestation for symptoms of pregnancy- induced 33. A client with preeclampsia is being treated with
hypertension and preeclampsia. The nurse bed rest and intravenous magnesium sulfate.
should give priority to assessing the client for? The drug classification of this medication is a:
. facial swelling . diuretic
a. pulse deficits a. tocolytic
b. ankle edema b. anticonvulsant
c. diminished reflexes c. antihypertensive
• Correct answer: a. facial d. None of the above
swelling • Correct answer: c.
28. The nurse is caring for a client receiving anticonvulsant
intravenous magnesium sulfate must closely 34. A woman with severe preeclampsia is receiving
observe for side effects associated with drug a magnesium sulfate infusion. The nurse
therapy. An expected side effect of magnesium becomes concerned after assessment when the
sulfate is: woman exhibits:
. decreased urinary output. . a sleepy, sedated affect.
a. hot flashes and episode of sleepiness a. a respiratory rate of 10 breaths/min.
b. absence of knee jerk reflex b. deep tendon reflexes of 2+
c. decreased respiratory rate. c. absent ankle clonus.
• Correct answer: b. hot flashes • Correct answer: b. a
and episode of sleepiness respiratory rate of 10
29. The client with preeclampsia is admitted to your breaths/min.
unit with an order for magnesium sulfate. Which 35. You are doing your patient education with your
action by the nurse indicates understanding of clinical instructor in the OB ward about
the possible side effects of magnesium sulfate. Gestational diabetes. You discuss the role of
. the nurse places a sign over the bed not to insulin in the body. Select all the CORRECT
check blood pressure in the right arm. statements that you are going to tell the patients
a. the nurse places a padded tongue blade at the about the role of insulin in the body.
bedside. . “Insulin is a type of cell that provides glucose to
b. the nurse inserts a foley catheter. the body from the blood.”
c. the nurse darkens the room. a. “Insulin is a hormone secreted by the beta cells
• Correct answer: c. the nurse of the pancreas.”
inserts a foley catheter. b. “Insulin influences cells by causing them to
30. A woman, who is 22 weeks pregnant, has a uptake glucose from the blood.”
routine ultrasound performed. The ultrasound c. "Insulin is a protein that helps carry glucose into
shows that the placenta is located at the edge of the cell for energy."
the cervical opening. As the nurse you know that • Correct answer: c. “Insulin
which statements is False about the finding. influences cells by causing
. this is known as the marginal placenta previa. them to uptake glucose
a. the placenta may move upward as the from the blood.”
pregnancy progresses and needs to be re-evaluated 36. The physician is considering HIV to patient X.
with another ultrasound at about 32 weeks gestation. You are aware that the screening test for HIV?
. Viral load • Correct answer: c. severe
a. Viral culture pneumonia or very severe
b. ELISA disease
c. CD4/CD8 count 45. Where can the IMCI case management
• Correct answer: c. ELISA guidelines be used?
37. Your client diagnosed with AIDS understands . in the inpatient ward of a hospital
her illness when she tells you: a. in a neonatal ward
. I need to eat yogurt every day to provide needed b. in the outpatient ward of a hospital and first-level
calcium for my bones. health facilities
a. I should peel and cook fruits before eating them. c. at the household level
b. I can enjoy foods from the salad bar at my local • Correct answer: c. in the
restaurant. outpatient ward of a
c. I might have to floss my teeth more often to hospital and first-level
prevent gum disease. health facilities
• Correct answer: b. I should 46. If a child has measles now or has had it within
peel and cook fruits before the last three months, and has fever and any
eating them. general danger sign, he or she will be classified
38. Which of the following movements of the lower as having:
chest describes chest indrawing? . Uncomplicated malaria
. Inward movement during inspiration a. severe complicated measles; very severe febrile
a. Inward movement during expiration disease or severe malaria
b. Outward movement during inspiration b. measles with eye or mouth complication
c. Outward movement during expiration c. measles
• Correct answer: a. Inward • Correct answer: b. severe
movement during complicated measles; very
inspiration severe febrile disease or
39. What is "fast breathing" in a 3-month-old child? severe malaria
. 20 breaths per minute or more 47. A child with fever plus any general danger sign
a. 30 breaths per minute or more should be classified as:
b. 40 breaths per minute or more . uncomplicated malaria
c. 50 breaths per minute or more a. acute ear infection
• Correct answer: d. 50 breaths b. measles
per minute or more c. very severe febrile disease or severe malaria
40. Which of the following signs are "general danger d. mastoiditis
signs" in a sick child who is 11 months old? • Correct answer: d. very
. Axillary temperature ≥ 39.0°C severe febrile disease or
a. Lethargy severe malaria
b. Blood in the stool 48. Your patient with a preeclampsia (severe) is
c. 10 loose, watery stools in a single day admitted with BP 170/100, with proteinuria, with
• Correct answer: b. Lethargy severe pitting edema Grade III. Which of the
41. This essential vitamin during pregnancy is following you would include in making your
critical in the absorption, distribution and storage patient’s plan of care?
of Calcium. . Daily weights
. Vitamin C a. Right lateral positioning
a. Vitamin D b. Seizure precautions
b. Ferrous Sulfate c. Stress reduction
c. Iron • Correct answer: c. Seizure
• Correct answer: b. Vitamin D precautions
42. Excessive nutrition during pregnancy can result 49. For the nutritional needs of a pregnant woman
in: for her growing fetus, which of the following
. Fetal Macrosomia and Post-partum Weight minerals are needed and responsible for fetal
Retention bone and teeth formation?
a. Fetal Defects . Iron and Folic Acid
b. Fetal Microsomia a. Calcium and Phosphorous
c. Fetal Death b. Vitamin D and E
• Correct answer: a. Fetal c. Sodium and Potassium
Macrosomia and Post- • Correct answer: b. Calcium
partum Weight Retention and Phosphorous
43. You can classify that a child aged 2 years has 50. Which of the following statements best describes
fast breathing if he has a respiratory rate of hyperemesis gravidarum?
. 60 bpm or more . Severe anemia leading to electrolyte, metabolic
a. 50 bpm or more and nutritional imbalances in the absence of other
b. 40 bpm or more medical problems
c. Any of the above a. Severe nausea and vomiting leading to
• Correct answer: c. 40 bpm or electrolyte, metabolic and nutritional imbalances in the
more absence of other medical problems
44. What is a child’s classification if he is 10 months b. Loss of appetite and continuous vomiting that
old, has had a cough that lasted two days, has a commonly results in dehydration and ultimately
breathing rate of 46 breaths per minute and decreasing maternal nutrients
chest indrawing? c. Severe nausea and diarrhea that can cause
. no pneumonia: cough or cold gastrointestinal irritation and possibly internal bleeding
a. pneumonia • Correct answer: b. Severe
b. severe pneumonia or very severe disease nausea and vomiting
c. very severe febrile disease leading to electrolyte,
metabolic and nutritional . Send the mother a brochure of things she can
imbalances in the absence try to assist the child to sleep independently.
of other medical problems a. Advise the mother that this is a normal behavior
51. To ensure efficient workflow of the clinic and that will eventually pass with time.
maximize available expertise, which task should b. Suggest that the child be put back into her own
be assigned to the experienced Licensed bed and allowed to cry herself to sleep.
Practical Nurse (LPN)? c. Schedule an appointment with the APN student
. Perform triage for walk-in clients. for assessment and management.
a. Perform physical assessment of walk-in clients. • Correct answer: Advise the
b. Give routine immunizations. mother that this is a normal
c. Obtain weight and height measurements. behavior that will eventually
• Correct answer: c. Give pass with time.
routine immunizations. 56. Six-year-old Billy woke last night with dyspnea,
52. A mother brings her 12-month-old child to the restlessness, wheezing, and cough. Mother and
clinic for an influenza vaccination. The RN tells child spent the night in a reclining chair. His
the mother that the child is also due for doses of mother declares, “He is having an asthma
measles-mumps-rubella, varicella, and hepatitis attack. We are both exhausted. I’m tired of
A vaccines. The mother declines the nurse's waiting forever to see the doctor!” What is the
advice because “he has already had enough of priority nursing concern?
those.” What is the priority action? . Billy's poor sleep quality and restlessness
. Encourage a follow-up appointment and notify a. Billy's ongoing shortness of breath
Child Protective Services. b. Mother's report of feeling exhausted
a. Assess the mother's concerns and current level c. Mother's frustration with health care system
of knowledge about immunization. • Correct answer: Billy's
b. Emphasize the benefits of immunization; explain ongoing shortness of
the purpose and schedule. breath
c. Respect the mother's decision and alert the 57. As the nurse approaches Billy, which
pediatrician to the situation. presentation would be of most concern and
• Correct answer: Assess the require immediate intervention?
mother's concerns and . Alert and irritable, lying recumbent on the
current level of knowledge examination table
about immunization. a. Awake and nervous, sitting upright and crying,
53. A 9-month-old child arrives at the health center skin pale and dry
with his mother for immunizations. The child is b. Agitated, sweating, and sitting upright with
fussy with rhinorrhea and has an axillary shoulders hunched forward
temperature of 100.4°F (38°C). The pediatrician c. Asleep in a side-lying position breathing through
has determined that the child has open mouth
nasopharyngitis. What is the priority action? • Correct answer: Agitated,
. Administer half of the immunizations and sweating, and sitting
reschedule a subsequent appointment for the other half. upright with shoulders
a. Advise the mother that fever is a contraindication hunched forward
for immunization and reschedule the appointment. 58. Which assessment finding for Billy is the most
b. Administer acetaminophen to reduce fever and urgent and requires immediate intervention and
apply an anesthetic cream to the injection site. notification of the pediatrician?
c. Advise the mother that the child will likely need . Sudden increase in respiratory rate and
an antibiotic and reschedule the appointment. decreased breath sounds
• Correct answer: Administer a. Rattling cough productive of frothy, clear,
acetaminophen to reduce gelatinous sputum
fever and apply an b. Crackles auscultated on inspiration in the lower
anesthetic cream to the lung fields
injection site. c. Restlessness and wheezing auscultated at the
54. A parent calls in for advice because her 18- end of expiration
month-old toddler has stumbled and bumped his • Correct answer: Sudden
head on the coffee table. Which symptom is increase in respiratory rate
cause for the greatest concern? and decreased breath
. A swelling the size of a golf ball that is tender to sounds
the touch 59. As the nurse cares for Billy and addresses his
a. Two episodes of vomiting a small amount of asthmatic condition, in what sequence should
undigested food the following prescribed actions be
b. Continuous crying for 2 hours, unrelieved by implemented?
familiar comfort measures 1. Give nebulized albuterol now and
c. Gaping 1.5-inch (4-cm) laceration on the every 30 minutes.
forehead, with bleeding controlled by pressure 2. Administer humidified oxygen to
• Correct answer: Continuous maintain saturation above 90%.
crying for 2 hours, 3. Administer IV methylprednisolone.
unrelieved by familiar 4. Obtain a chest radiograph and a
comfort measures complete blood count (CBC).
55. A parent calls in for advice because “Missy is 5 5. Contact the hospital about
years old, and she just won’t sleep in her own admission.
bed. For the past 4 months, she wakes and 6. Teach about measuring peak
comes to sleep with me and my husband. She expiratory flow rate to determine
cries and cries if we take her back to her own personal best.
room.” What is the priority action? 7. Schedule a radioallergosorbent test.
Correct answer:
• Give nebulized albuterol now 4. Terry is 7 months old; he rubs at
and every 30 minutes. both of his ears, acts fussy, refuses
• Administer humidified oxygen to suck, and has a temperature of
to maintain saturation above 101.2°F (38.4°C). He has had three
90%. episodes of otitis media in the past.
• Administer IV Social history includes being bottle-
methylprednisolone. fed and having parents who are both
• Obtain a chest radiograph smokers.
and a complete blood count Correct answer:
(CBC). • James is 3 years old. He
• Contact the hospital about awakened last night with a sore
admission. throat, difficulty 668 swallowing,
• Teach about measuring peak and a fever. He is flushed,
expiratory flow rate to anxious, and drooling. The
determine personal best. nurse observes a thick, muffled
• Schedule a quality to his voice and slow,
radioallergosorbent test. quiet breathing. The nurse notes
60. Billy is to be transferred from the clinic to the that James looks sick.
hospital for his asthmatic condition. Which tasks are RN • Daisy is 4 years old; she is alert
responsibilities and should not be delegated or and irritable with pale, sweaty
performed by another member of the health care team? skin. An older neighbor who was
Select all that apply. temporarily watching Daisy
a. Give a report to the attending pediatrician at the reports that she was running
receiving hospital. around and playing, and then
b. Give a report to the charge nurse at the she got “grumpy.” Daisy has
receiving hospital. diabetes, but the neighbor “was
c. Help the parent and child to collect and bag up not sure how to give her the
personal items. insulin.”
d. Determine that the client's condition is stable • Sarah is 11 months old; she is
enough for transport to the hospital. dirty and crying, and her right
e. Assess the response to treatment and document arm is swollen and red. Sam is
the client's condition. Sarah's 2-year-old brother; he is
f. Assist the client to transfer to the ambulance dirty and hungry and reaches
stretcher. out to be picked up. Ms. A, their
• Correct answer: mother, is 19 years old and
• Give a report to the charge single. She is thin and
nurse at the receiving disheveled and seems
hospital. somewhat confused. She is
• Assess the response to having trouble answering the
treatment and document the nurse's questions. Ms. A says,
client's condition. “Those kids play too rough! The
61. In the afternoon, several clients come to the older one is always pushing the
clinic for walk-in care. Prioritize the following clients in baby off the bed.”
the order in which they should be seen to ensure safe • Terry is 7 months old; he rubs at
care and efficiently manage client load. both of his ears, acts fussy,
1. James is 3 years old. He awakened refuses to suck, and has a
last night with a sore throat, difficulty temperature of 101.2°F
668 swallowing, and a fever. He is (38.4°C). He has had three
flushed, anxious, and drooling. The episodes of otitis media in the
nurse observes a thick, muffled past. Social history includes
quality to his voice and slow, quiet being bottle-fed and having
breathing. The nurse notes that parents who are both smokers.
James looks sick. 62. The pediatrician examines James and
2. Daisy is 4 years old; she is alert and determines that he should be taken immediately to the
irritable with pale, sweaty skin. An Children's Hospital emergency department (ED). The
older neighbor who was temporarily child is breathing slowly and quietly; humidified oxygen
watching Daisy reports that she was is being administered. What is the priority action?
running around and playing, and a. Instruct the parents to drive the child to the
then she got “grumpy.” Daisy has hospital immediately and call the ED.
diabetes, but the neighbor “was not b. Contact a private ambulance service and
sure how to give her the insulin.” prepare the client for transport.
3. Sarah is 11 months old; she is dirty c. Call 911, ask for advanced emergency medical
and crying, and her right arm is services (EMS), and monitor the child.
swollen and red. Sam is Sarah's 2- d. Assist the pediatrician to intubate the child and
year-old brother; he is dirty and then arrange for transport.
hungry and reaches out to be picked • Correct answer: Call 911, ask
up. Ms. A, their mother, is 19 years for advanced emergency
old and single. She is thin and medical services (EMS), and
disheveled and seems somewhat monitor the child.
confused. She is having trouble 63. Daisy's mother arrives at the clinic, and she is
answering the nurse's questions. relieved to find Daisy happy and smiling, but the mother
Ms. A says, “Those kids play too bursts into tears and begins to yell at her neighbor and
rough! The older one is always the nursing staff for “not taking care of her!” What is the
pushing the baby off the bed.” best way to handle her anger and tears?
. Remind the mother that the child is okay and b. Oxygen therapy
that the neighbor was doing what she thought was best c. Flutter valve with huffing
based on the information that she had. • Correct answer: Flutter valve
a. Allow the mother to express her feelings and with huffing
then take the neighbor aside and explain that the mother 69. The nurse reviews Rebecca's growth chart and
is just temporarily upset. determines her weight and BMI have consistently
b. Teach the mother about ways to communicate remained below the 5th percentile. Which collaborative
the child's needs to all caregivers and help her make a intervention is the nurse most likely to anticipate when
list of specific instructions. planning teaching?
c. Direct the mother to a private area and . Preparing the child and family for potential
encourage her to ventilate feelings; then gently assess gastrostomy tube placement
how she typically manages Daisy's diabetes. a. Focusing on increasing the child's intake of
• Correct answer: Direct the protein and calories
mother to a private area and b. Suggesting an increased dosage of pancreatic
encourage her to ventilate enzymes
feelings; then gently assess c. Preparing the child and family for potential total
how she typically manages parenteral nutrition
Daisy's diabetes. • Correct answer: Preparing the
64. What is the priority nursing concern for Terry, child and family for
who is rubbing at his ears, acting fussy, refusing to suck, potential gastrostomy tube
and has a temperature of 101.2°F (38.4°C)? placement
. Pain 70. Which laboratory value is consistent with
a. Poor nutrition clubbing of Rebecca's finger and toe nails?
b. Recurrent ear infections . Elevated white blood cell (WBC) count
c. Elevated temperature a. Elevated red blood cell (RBC) count
• Correct answer: Pain b. Decreased hematocrit
65. The pediatric social worker has just informed c. Decreased mean corpuscular volume
Ms. A that Child Protective Services has been notified • Correct answer: Elevated red
and that a representative will arrive shortly to speak with blood cell (RBC) count
her about the family's situation. Ms. A starts to cry and
threatens to leave. What is the priority action?
. Obtain an “against medical advice” (AMA) form
and have her sign it.
a. Notify the pediatrician of the mother's intent to
leave.
b. Inform the mother that the police will be notified
if she leaves.
71. Which nursing assessment is a priority?
c. Encourage Ms. A to remain and to express
a. Obtain a rectal temperature.
feelings and fears.
b. Auscultate the lungs.
• Correct answer: Encourage
c. Obtain a blood pressure.
Ms. A to remain and to
d. Auscultate the heart.
express feelings and fears.
• Correct answer: Auscultate
Rebecca is a 6-year-old girl with cystic fibrosis. She the heart.
arrives at the pediatric cystic fibrosis clinic for her routine 72. Which pharmacologic intervention should the
3-month appointment. The nurse obtains and calculates nurse anticipate at this time?
the following growth parameters: Weight, 33 lbs (15 kg) . IV methylprednisolone
(< 5%); height, 42 in. (106 cm) (5%); and body mass a. IV immunoglobulin
index (BMI), 13 (< 5%). Her parents report specks of b. IV ibuprofen
blood in her sputum after chest physiotherapy. Her c. IV infliximab
forced expiratory volume in 1 second (FEV1) has • Correct answer: IV
decreased 25% from her last visit 3 months ago. immunoglobulin
Rebecca has clubbing of both her finger and toe nails. 73. After a 14-day hospitalization, Bobby is
discharged home on a regimen of long-term aspirin
66. How would the nurse best interpret Rebecca's therapy. What advice should the nurse provide regarding
decreased FEV1? influenza vaccination?
a. Decreased oxygenation . Influenza vaccine is indicated for children 4
b. Increased obstruction months to 18 years of age.
c. Presence of infection a. Influenza vaccine should be postponed for 11
d. Pulmonary remodeling months after treatment.
• Correct answer: Increased b. Influenza vaccine is indicated for children
obstruction receiving long-term aspirin therapy.
67. What is the most likely cause of the specks of c. Influenza vaccine should be postponed in
blood in Rebecca's sputum? children receiving long-term aspirin therapy.
. A sign of pulmonary infection • Correct answer: Influenza
a. A sign of bronchial remodeling vaccine is indicated for
b. A sign of gastric irritation children receiving long-term
c. A sign of gastrointestinal bleeding aspirin therapy.
• Correct answer: A sign of
pulmonary infection Eight-year-old Charlie had a laparoscopic
68. The nurse establishes that Rebecca has poor appendectomy. During surgery, the appendix perforated.
airway clearance. Which intervention is most important Charlie arrives on the pediatric unit from the operating
for the nurse to implement for this problem? room. His weight on admission was 46 lbs (21 kg). He
. Increased fluid requirements has a peripheral IV line in the left basilic vein with D5W
a. Inhaled corticosteroids
and 20 mEq/L (20 mmol/L) of KCl running at 70 mL/hr. A b. A 32-year-old G4P3 woman at 27 weeks’
nasogastric tube (NG) is attached to low suction. gestation who noted vaginal bleeding today after
intercourse
74. Which assessment should be most concerning c. A 27-year-old G2P1 woman at 37 weeks’
to the nurse? gestation who experienced spontaneous rupture of
a. Oral temperature of 100.4°F (38°C) membranes 30 minutes ago but feels no contractions
b. Decreased bowel sounds in all quadrants • Correct answer: A 22-year-old
c. Urine output of 160 mL over 4 hours G3P2 woman at 38 weeks’
d. Respiratory rate of 15 breaths per minute gestation with contractions
• Correct answer: Respiratory every 3 minutes who is
rate of 15 breaths per requesting to go to the
minute bathroom to have a bowel
75. Charlie's parents ask the nurse if the NG tube movement
can be removed because it is irritating Charlie's nose. 79. A 19-year-old gravida 1, para 0 patient at 40
What is the nurse's best response? weeks’ gestation who is in labor is being treated with
. “The NG tube is necessary to prevent aspiration magnesium sulfate for seizure prophylaxis in
of the stomach contents into the lungs.” preeclampsia. Which are priority assessments with this
a. “The NG tube is necessary because Charlie will medication? Select all that apply.
need to have feedings through it.” . Check deep tendon reflexes.
b. “The NG tube is necessary to keep Charlie's a. Observe for vaginal bleeding
stomach empty, allowing the intestines to rest.” b. Check the respiratory rate.
c. “The NG tube is necessary to prevent swallowed c. Note the urine output.
air from building up in the stomach.” d. Monitor for calf pain.
• Correct answer: “The NG tube • Correct answer:
is necessary to keep • Check deep tendon reflexes.
Charlie's stomach empty, • Check the respiratory rate.
allowing the intestines to
• Note the urine output.
rest.”
80. Which action would best demonstrate evidence-
76. A 30-year-old woman with type 1 diabetes
based nursing practice in the care of a patient who is 1
mellitus comes to the clinic for preconception care. What
day postpartum and reporting nipple soreness while
is the priority education for her at this time?
breast feeding?
. Her insulin requirements will likely increase
. Assess the mother–baby couplet for nursing
during the second and third trimesters of pregnancy.
position and latch and correct as indicated.
a. Infants of mothers with diabetes can be
a. Advise the use of a breast pump until nipple
macrosomic, which can result in more difficult delivery
soreness resolves.
and higher likelihood of cesarean section
b. Advise alternating breast and bottle feedings to
b. Breast feeding is highly recommended, and
avoid excess sucking at the nipple
insulin use is not a contraindication.
• Correct answer: Assess the
c. Achievement of optimal glycemic control at this
mother–baby couplet for
time is of utmost importance in preventing congenital
nursing position and latch
anomalies.
and correct as indicated.
• Correct answer: Achievement
81. A 24-year-old gravida 2, para 1 woman is being
of optimal glycemic control
admitted in active labor at 39 weeks’ gestation. What
at this time is of utmost
prenatal data would be most important for the nurse to
importance in preventing
address at this time?
congenital anomalies.
. Hemoglobin level of 11 g/dL (110 g/L) at 28
77. Which task could be appropriately delegated to
weeks’ gestation
the unlicensed assistive personnel (UAP) working with
a. Positive result on test for group B streptococci at
the nurse at the obstetric clinic?
36 weeks’ gestation
. Checking the blood pressure of a patient who is
b. Urinary tract infection with Escherichia coli
36 weeks’ pregnant and reports a headache
treated at 20 weeks’ gestation
a. Removing the adhesive skin closure strips of a
c. Elevated level on glucose screening test at 28
patient who had a cesarean section 2 weeks ago
weeks’ gestation followed by normal 3-hour glucose
b. Giving community resource information and
tolerance test results at 29 weeks’ gestation
emergency numbers to a prenatal patient who may be
• Correct answer: Positive
experiencing domestic violence
result on test for group B
c. Dispensing a breast pump with instruction to a
streptococci at 36 weeks’
lactating patient having trouble with milk supply 4 weeks
gestation
postpartum
82. The telephone triage nurse in the prenatal clinic
• Correct answer: Checking the
receives the following calls. Which telephone call would
blood pressure of a patient
require immediate notification of the health care
who is 36 weeks’ pregnant
provider?
and reports a headache
. Patient reports leaking vaginal fluid at 34 weeks’
78. Several patients have just come into the
gestation
obstetric triage unit. Which patient should the nurse
a. Patient reports nausea and vomiting at 8 weeks’
assess first?
gestation.
. A 17-year-old gravida 1, para 0 (G1P0) woman
b. Patient reports pedal edema at 39 weeks’
at 40 weeks’ gestation with contractions every 6 minutes
gestation.
who is crying loudly and is surrounded by anxious family
c. Patient reports vaginal itching at 20 weeks’
members
gestation
a. A 22-year-old G3P2 woman at 38 weeks’
• Correct answer: Patient
gestation with contractions every 3 minutes who is
reports leaking vaginal fluid
requesting to go to the bathroom to have a bowel
at 34 weeks’ gestation
movement
83. The nurse in the labor and delivery unit is caring d. Assisting the mother with ambulation
for a 25-year-old gravida 3, para 2 patient in active labor. • Correct answer:
The nurse has identified late fetal heart decelerations • Providing the mother with an
and decreased variability in the fetal heart rate and ordered abdominal binder
notified the health care provider (HCP) on call, who • Taking the mother's vital
thinks that the pattern is acceptable. What would be the signs
priority action at this time? • Assisting the mother with
. Advise the patient that a different HCP will be ambulation
called because the first HCP's response was not 88. Which action by a newly graduated RN during a
adequate delivery complicated by shoulder dystocia would require
a. Discuss the concerns with another labor and immediate correction by the nurse who is orienting her?
delivery nurse . Applying fundal pressure
b. Document the conversation with the HCP a. Applying suprapubic pressure
accurately, including the HCP's interpretation and b. Requesting immediate presence of the
recommendation, and continue close observation of the neonatologist
fetal heart rate. c. Flexing the maternal legs back across the
c. Document the conversation with the HCP maternal abdomen
accurately, including the HCP's interpretation and • Correct answer: Applying
recommendation, and continue close observation of the fundal pressure
fetal heart rate. 89. Which statements by a new father indicate that
• Correct answer: Document additional discharge teaching is needed for this family,
the conversation with the who had their first baby 24 hours ago? Select all that
HCP accurately, including apply.
the HCP's interpretation and . “We have a crib ready for our baby with lots of
recommendation, and stuffed animals and two quilts that my mother made.”
continue close observation a. “My wife wants to receive the flu shot before she
of the fetal heart rate. goes home.”
84. What would be the appropriate first nursing b. “We will bring our baby to the pediatrician in 3
action when caring for a 20- year-old gravida 1, para 0 weeks.”
woman at 39 weeks’ gestation who is in active labor and c. “I will give the baby formula at night so my wife
for whom an assessment reveals mild variable fetal heart can rest. She will breast feed in the daytime.”
rate decelerations? d. “We will always put our baby to sleep in a face-
. Change the maternal position. up position.”
a. Notify the provider. • Correct answer:
b. Prepare for delivery. • “We have a crib ready for our
c. Readjust the fetal monitor. baby with lots of stuffed
• Correct answer: Change the animals and two quilts that
maternal position. my mother made.”
85. A 24-year-old gravida 1, para 0 patient, who is • “We will bring our baby to the
receiving oxytocin, is in labor at 41 weeks gestation. pediatrician in 3 weeks.”
Which are appropriate nursing actions in the presence of • “I will give the baby formula
late fetal heart rate decelerations? Select all that apply. at night so my wife can rest.
. Discontinue the oxytocin. She will breast feed in the
a. Decrease the maintenance IV fluid rate. daytime.”
b. Administer oxygen to the mother by mask. 90. The charge nurse in the labor and delivery unit
c. Place the woman in high Fowler position. needs to assign two patients to one of the RNs because
d. Notify the health care provider. of a staffing shortage. Normally the unit has nurse-
• Correct answer: patient ratio of 1:1. Which two patients should the charge
• Discontinue the oxytocin. nurse assign to the RN?
• Administer oxygen to the . A 30-year-old gravida 1, para 0 (G1P0) woman,
mother by mask. 40 weeks, 2 cm/90% effaced/–1 station
• Notify the health care a. A 25-year-old G3P2 woman, 38 weeks, 8
provider. cm/100% effaced/0 station
86. A pregnant woman at 12 weeks’ gestation tells b. A 26-year-old G1P1 woman who delivered via
the nurse that she is a vegetarian. What would be the normal vaginal delivery 15 minutes ago
first appropriate nursing action? c. A 17-year-old G1P0 woman with premature
. Recommend vitamin B12 and iron rupture of membranes, no labor at 35 weeks
supplementation. d. A 40-year-old G6P5 woman with contractions at
a. Recommend consumption of protein drinks daily. 28 weeks who has not yet been evaluated by the health
b. Obtain a 24-hour diet recall history. care provider
c. Determine the reason for her vegetarian diet. • Correct answer:
• Correct answer: Obtain a 24- • A 30-year-old gravida 1, para
hour diet recall history. 0 (G1P0) woman, 40 weeks,
87. A 26-year-old gravida 1, para 1 patient who 2 cm/90% effaced/–1 station
underwent cesarean section 24 hours ago tells the nurse • A 17-year-old G1P0 woman
that she is having some trouble breast feeding. Which with premature rupture of
tasks could be appropriately delegated to the unlicensed membranes, no labor at 35
assistive personnel (UAP) on the postpartum floor? weeks
Select all that apply. 91. While assessing a 29-year-old gravida 2, para 2
. Providing the mother with an ordered abdominal patient who had a normal spontaneous vaginal delivery
binder 30 minutes ago, the nurse notes a large amount of red
a. Assisting the mother with breast feeding vaginal bleeding. What would be the first priority nursing
b. Taking the mother's vital signs action?
c. Checking the amount of lochia present . Check vital signs.
a. Notify the health care provider. d. Decrease the IV fluid rate.
b. Firmly massage the uterine fundus. • Correct answer:
c. Put the baby to breast. • Turn the patient to a lateral
• Correct answer: Firmly position.
massage the uterine • Notify the anesthesiologist.
fundus. 96. A 17-year-old gravida 1, para 0 woman at 40
92. A 30-year-old gravida 1, para 0 woman at 39 weeks’ gestation is in labor. She has chosen natural
weeks’ gestation experienced a fetal demise and has childbirth with assistance from a doula. Her mother and
just delivered a female infant. Her husband is at the her boyfriend are at the bedside. What nursing action
bedside. Which are appropriate nursing actions at this can help the patient achieve her goal of an unmedicated
time? Select all that apply. labor and birth?
. Offer the option of autopsy to the parents . Encourage the patient to stay in bed.
a. Stay with the parents and offer supportive care. a. Allow the patient's support people to provide
b. Place the infant on the maternal abdomen. labor support and minimize nursing presence.
c. Clean and wrap the baby and offer the infant to b. Assess the effectiveness of the labor support
the parents to view or hold when desired. team and offer suggestions as indicated.
d. Ask the parents if there are any special rituals in c. Offer pain medication on a regular basis so the
their religion or culture for a baby who has died that they patient knows it is available if desired
would like to have done. • Correct answer: Assess the
• Correct answer: effectiveness of the labor
• Stay with the parents and support team and offer
offer supportive care. suggestions as indicated.
• Clean and wrap the baby and 97. A 25-year-old gravida 2, para 1 patient has
offer the infant to the come to the obstetric triage room at 32 weeks’ gestation
parents to view or hold reporting painless vaginal bleeding. The nurse is
when desired. providing orientation for a new RN on the unit. Which
• Ask the parents if there are statement by the new RN to the patient would require
any special rituals in their the nurse to promptly intervene?
religion or culture for a . “I’m going to check your vital signs.”
baby who has died that they a. “I’m going to apply a fetal monitor to check the
would like to have done. baby's heart rate and to see if you are having
93. A 27-year-old patient underwent a primary contractions.”
cesarean section because of breech presentation 24 b. “I’m going to perform a vaginal examination to
hours ago. Which assessment finding would be of the see if your cervix is dilated.”
most concern? c. “I’m going to feel your abdomen to check the
. Small amount of lochia rubra position of the baby.”
a. Temperature of 99°F (37.2°C) • Correct answer: “I’m going to
b. Slight redness of the left calf perform a vaginal
c. Pain rated as 3 of 10 in the incisional area examination to see if your
• Correct answer: Slight cervix is dilated.”
redness of the left calf 98. A 30-year-old gravida 6, para 5 woman at 12
94. A 22-year-old gravida 1, para 0 woman is being weeks’ gestation has just begun prenatal care, and her
given an epidural anesthetic for pain control during labor initial laboratory work reveals that she has tested
and birth. Which are appropriate nursing actions when positive for human immunodeficiency virus (HIV)
epidural anesthesia is used during labor? Select all that infection. What would be priority evidence-based nursing
apply education for this patient today?
. Request the anesthesiologist to discontinue the . Medication for HIV infection is safe and can
epidural anesthetic when the patient's cervix is greatly reduce transmission of HIV to the infant.
completely dilated to allow the patient to sense the urge a. Breast feeding is still recommended due to the
to push great benefits to the infant.
a. Insert an indwelling catheter because the b. Pregnancy is known to accelerate the course of
woman is likely to be unable to void. HIV disease in the mother
b. Encourage pushing efforts when the cervix is c. Cesarean section is not recommended because
completely dilated in the absence of an urge to push. of the increased risk of HIV transmission with the
c. Encourage the patient to turn from side to side bleeding at surgery.
during the course of labor. • Correct answer: Medication
d. Teach the patient that pain relief can be for HIV infection is safe and
expected to last 1 to 2 hours. can greatly reduce
• Correct answer: transmission of HIV to the
• Insert an indwelling catheter infant.
because the woman is likely 99. A 22-year-old woman is 6 weeks postpartum. In
to be unable to void. the clinic, she admits to crying every day, feeling
• Encourage the patient to turn overwhelmed, and sometimes thinking that she may hurt
from side to side during the the baby. What would be the priority nursing action at
course of labor. this time?
95. A 36-year-old gravida 1, para 0 patient has . Advise the patient of community resources,
received an epidural anesthetic. Her cervix is 6 cm parent groups, and depression hotlines
dilated. Her blood pressure is currently 60/38 mm Hg. a. Counsel the mother that the “baby blues” are
Which would be appropriate priority nursing actions? common at this time and assess her nutrition, rest, and
Select all that apply availability of help at home.
. Place the patient in high Fowler position. b. Contact the health care provider to evaluate the
a. Turn the patient to a lateral position. patient before allowing her to leave the clinic.
b. Notify the anesthesiologist.
c. Prepare for emergency cesarean section.
c. Advise the woman that she cannot use . "This is a normal reaction to parenthood in the
medication for depression because she is breast first trimester."
feeding. a. "This would be the best time to consider an
• Correct answer: Contact the abortion or adoption."
health care provider to b. "This is a sign of depression and I'd like you to
evaluate the patient before see a mental health specialist."
allowing her to leave the c. "This is an abnormal reaction, and I'd like you to
clinic. speak with Family Services."
100. A 23-year-old gravida 1, para 0 patient at 10 • Correct answers: a. "This is a
weeks’ gestation states that she exercises 5 days a normal reaction to
week. The nurse has discussed exercise in pregnancy parenthood in the first
with her. Which statement by the patient indicates that trimester."
more teaching of evidence- based principles is needed? 5. The nurse is teaching a parenting class to
. “I will continue to exercise 5 days a week.” prospective fathers. The nurse correctly teaches
a. “I will reduce my exercise at this time in my that couvade refers to the:
pregnancy to reduce the risk of miscarriage but will . Expectant father's fear of hurting the unborn
increase it in the second trimester.” baby during intercourse.
b. “I will drink more fluid before and after a. Expectant father's transition from nonparent to
exercising.” parent.
c. “I will stop playing football while I am pregnant.” b. Development of the physical symptoms of
• Correct answer: “I will reduce pregnancy in the father of the baby.
my exercise at this time in c. Development of attachment and bonding
my pregnancy to reduce the behaviors in the father of the baby.
risk of miscarriage but will • Correct answers: c.
increase it in the second Development of the
trimester.” physical symptoms of
pregnancy in the father of
the baby.
6. A 20-year-old woman arrives at a family
NCM 109 Maternal Lec
planning clinic seeking emergency contraception
MIDTERMS 48 hours after she had unprotected sexual
intercourse. The nurse correctly responds by
1. A nurse is teaching a group of first-trimester saying:
prenatal patients about the discomforts of . “Come right in so we can get you started.”
pregnancy. A patient asks the nurse, "What a. “You must wait 72 hours before the pill will
causes my nausea and vomiting?" The nurse work.”
knows the primary contributing factor to first- b. “You need to wait until you have missed your
trimester emesis is: period.”
a. Human chorionic gonadotropin. c. “The pills must be started the morning after
b. Estrogen intercourse.”
c. Progesterone • Correct answers: a. “Come
d. Prostaglandins right in so we can get you
• Correct answers: a. Human started.”
chorionic gonadotropin. 7. A newborn is born at 38 weeks gestation
2. The nurse is taking an initial history of a prenatal weighing 2250 grams. Which is the most
patient. Which sign would first indicate a appropriate nursing diagnosis?
positive, or diagnostic, sign of pregnancy? . Ineffective airway clearance.
. Fetal movement at 20 weeks gestation a. Risk for altered body temperature.
a. Visualization of fetal heart movement at 21 b. Acute pain.
weeks gestation c. Altered nutrition: More than body requirements.
b. Fetal heartbeat with Doppler at 19 weeks • Correct answers: b. Risk for
gestation altered body temperature
c. Fetal heartbeat with fetoscope at 18 weeks 8. Which response by the school nurse to a 16-
gestation year-old would be appropriate when the boy tells
• Correct answers: d. Fetal the nurse he smokes?
heartbeat with fetoscope at . "If you continue to smoke, you will get lung
18 weeks gestation cancer and die."
3. The nurse in the prenatal clinic assesses a 26- a. "We need to discuss how smoking can affect
year-old patient at 13 weeks gestation. Which your health."
presumptive (subjective) signs and symptoms of b. "You are hurting your family with your
pregnancy should the nurse anticipate? secondhand smoke."
. Hegar's sign and quickening c. "Did you know that smoking can stunt your
a. Ballottement and positive pregnancy test growth?"
b. Chadwick's sign and uterine souffle • Correct answers: b. "We need
c. Excessive fatigue and urinary frequency to discuss how smoking
• Correct answers: d. can affect your health."
Excessive fatigue and 9. The nurse teaches parents about pediatric
urinary frequency immunizations. What is considered an absolute
4. The nurse is taking a history from a prenatal contraindication to pediatric immunizations?
patient at seven weeks gestation. The patient . Respiratory illness with low-grade fever
states, "I don't know if I want this baby. How will a. Soreness, redness, and swelling at the previous
I know if I'll be a good mother?" What is the most injection site.
appropriate response by the nurse? b. Febrile seizure 1 month after the previous
injection of vaccine.
c. Anaphylactic reaction to previous immunization. child’s nutritional status, the nurse should ask
• Correct answers: d. the parents which question?
Anaphylactic reaction to . Do you think your child eats enough?
previous immunization a. Is your child picky eater?
10. A 6-year-old child is to receive regularly b. What did your child eat for breakfast?
scheduled immunizations. The parent states that c. d. Has your child always been so thin?
the child is not feeling well and asks the nurse to • Correct answers: c. What did
defer the immunizations until next week. What is your child eat for
the best response by the nurse? breakfast?
. Ask if the child has missed school. 17. A four-year-old patient, whose family follows a
a. Give the parent an immunization appointment for strict vegetarian diet, undergoes a
next week. prekindergarten physical examination. The
b. Check the child's temperature. pediatric nurse is alert to the patient's potential
c. Ask if the child has ever had a reaction to deficiency of:
immunizations. . Vitamin B12
• Correct answers: c. Check the a. Vitamin b6
child's temperature. b. Thiamin
11. A 35-year-old female is currently pregnant with c. Niacin
twins. She has 10-year-old triplets who were • Correct answers: a. Vitamin
born at 32 weeks gestation, and a 16-year-old B12
who was born at 41-week gestation. Twelve 18. Around what age should a child be able to build
years ago, she had a miscarriage at 19 weeks a tower of three building blocks?
gestation. What is her GTPAL? . 12 months
. G=4, T=1, P=1, A=1, L=4 a. 6 months
a. G=3, T=2, P=1, A=1, L=4 b. 3 months
b. G=4, T=1, P=1, A=1, L=3 c. 18 months
c. G=4, T=1, P=2, A=1, L=4 • Correct answers: d. 18
• Correct answers: a. G=4, T=1, months
P=1, A=1, L=4 19. When administering an I.M. injection to an infant,
12. During a prenatal visit a patient tells you her last the nurse in charge should use which site?
menstrual period was November 3, 2020. She . Vastus lateralis
told you that her menstruation is a 28 days a. Deltoid
cycle, when is the estimated due date of her b. Dorsogluteal
baby? c. Ventrogluteal
. August 10, 2020 • Correct answers: a. Vastus
a. August 11, 2021 lateralis
b. August 10, 2021 20. To meet the emotional needs of a 10-year-old
c. August 11, 2021 patient who is dying, the most appropriate
• Correct answers: c. August nursing action is to:
10, 2021 . encourage the patient to write in a journal
13. During a prenatal visit a patient tells you her last a. answer questions honestly and frankly
menstrual period was August 28, 2016. Based b. avoid interruptions by coordinating nursing
on the Naegele's Rule, when is the estimated actions
due date of her baby? c. provide opportunities for the patient to interact
. May 4, 2018 with children of the same age
a. June 5, 2017 • Correct answers: b. answer
b. May 4, 2017 questions honestly and
c. June 4, 2017 frankly
• Correct answers: d. June 4, 21. A woman spontaneously delivers a baby girl who
2017 is immediately handed to the nurse. Which
14. A 35-year-old female is currently pregnant with action is of highest priority for the nurse?
twins. She has 10-year-old triplets who were . Do an Apgar assessment.
born at 32 weeks gestation, and a 16-year-old a. Check neonatal heart rate.
who was born at 41-week gestation. Twelve b. Apply identification bracelets.
years ago, she had a miscarriage at 19 weeks c. Clear the nasopharynx.
gestation. What is her GTPAL? • Correct answers: d. Clear the
. G=3, T=2, P=1, A=1, L=4 nasopharynx.
a. G=4, T=1, P=1, A=1, L=4 22. Which area of health teaching will a new mother
b. G=4, T=1, P=1, A=1, L=3 be most responsive to during the taking in phase
c. G=4, T=1, P=2, A=1, L=4 of the postpartum period?
• Correct answers: b. G=4, T=1, . Family planning
P=1, A=1, L=4 a. Newborn care
15. During a prenatal visit a patient tells you her last b. Community support group
menstrual period was January 30, 2016. Based c. Perineal care
on the Naegele's Rule, when is the estimated • Correct answers: d. Perineal
due date of her baby? care
. November 6, 2016 23. A nurse is collecting data during an admission
a. October 6, 2016 assessment on a client who is pregnant with
b. October 7, 2016 twins. The client also has a 5-year-old child. The
c. November 6, 2017 nurse would document which gravida and para
• Correct answers: a. status on this client?
November 6, 2016 . Gravida III, Para II
16. Nurse Sunshine suspects that a child, age 4, is a. Gravida II, Para II
being neglected physically. To best assess the b. Gravida I, Para I
c. Gravida II, Para I 30. You are assigned in an outpatient department
• Correct answers: d. Gravida and one of your clients asks you what example
II, Para I of methods used in natural family planning.
24. A pregnant client is seen in the health care clinic . Calendar method and billings method
for a regular prenatal visit. The client tells the a. Calendar method and Condom
nurse that she is experiencing irregular b. Contraceptive pills and cervical mucus
contractions. The nurse determines that the c. Tubal ligation and vasectomy
client is experiencing Braxton hicks contractions. • Correct answers: a. Calendar
Based on the finding, which nursing action is method and billings
appropriate? method
. Instruct the client to maintain bed rest for the 31. Fundal height of a pregnant woman is measured
remainder of the pregnancy. from?
a. Instruct the client that these are common and . The top of the pubic bone to the top of the
may occur throughout the pregnancy. uterus.
b. Contact the physician. a. The middle of the pubic bone to the bottom of
c. Call the maternity clinic. the uterus.
• Correct answers: b. Instruct b. The bottom of the pubic bone to the top of the
the client that these are uterus.
common and may occur c. The top of the pubic bone to the middle of the
throughout the pregnancy. uterus.
25. A nursing instructor asks a student nurse to • Correct answers: a. The top of
describe the process of quickening. Which of the the pubic bone to the top of
following statements, if made by the student the uterus.
indicates an understanding of this term? 32. Which of the following statement is/are true?
. “It is the irregular, painless contractions that There is increasing concern about the
occur throughout pregnancy.” importance of adequate nutrition in the pre-
a. “It is the soft blowing sound that can be heard conception period to support a successful
when the uterus is auscultated.” pregnancy. This is because:
b. “It is the fetal movement that is felt by the . In the embryonic period the fetus is vulnerable to
mother.” external factors in its environment.
c. “It is the thinning of the lower uterine segment.” a. The embryonic period often occurs before
• Correct answers: c. “It is the woman knows she is pregnant.
fetal movement that is felt b. Interventions in late pregnancy may too late to
by the mother.” be effective.
26. A pregnant client asks the nurse in the clinic c. All of the options are correct.
when she will be able to start feeling the fetus to • Correct answers: b. The
move. The nurse responds by telling the client embryonic period often
that the fetal movement will be noted between: occurs before woman
. 6 to 8 weeks of gestation knows she is pregnant.
a. 8 to 10 weeks of gestation 33. Which of the following nutrients supplements
b. 10 to 12 weeks of gestation should a woman consume during pregnancy?
c. 14 to 16 weeks of gestation . Folic acid and Vitamin D
• Correct answers: d. 14 to 16 a. Iron
weeks of gestation b. Vitamin A
27. After Tetanus toxoid 3 vaccination a mother is c. Vitamin C
said to be protected to tetanus by around? • Correct answers: a. Folic acid
. 80% and Vitamin D
a. 99% 34. Which of these exercises do doctors recommend
b. 85% during pregnancy?
c. 90% . Yoga
• Correct answers: d. 90% a. Brisk walking
28. A 12-month-old received immunization at his b. Running
well-child visit. Fourteen days later, the mother c. All of the above
reports that the child has a red maculopapular • Correct answers: d. All of the
rash on the trunk and extremities. This rash is above
most likely due to which of the following 35. Gender of external genitalia of fetus becomes
vaccines? clearly distinguished at what weeks of gestation?
. DPT . 10 weeks
a. OPV a. 16 weeks
b. MMR b. 12 weeks
c. Hib c. 20 weeks
• Correct answers: c. MMR • Correct answers: d. 20 weeks
29. After the baby is delivered, the cord was cut 36. After receiving an immunization for diphtheria,
between two clamps using sterile scissors or tetanus and pertussis, a patient develops
blade, then the baby is placed at the? swelling and tenderness at the injection site, a
. Mother’s breast for the baby to feed and low-grade fever, and malaise. The nurse informs
bonding. the patient’s parents that:
a. Mother’s side. . The reaction is severe enough that they should
b. Give it to the grandmother. bring the patient to an emergency department.
c. Baby’s own mat or bed. a. They should delay future immunizations until the
• Correct answers: a. Mother’s patient sees a doctor.
breast for the baby to feed b. This is a mild reaction, and teaches them how to
and bonding. manage it.
c. This is an appropriate reaction, and instructs 43. For the nutritional needs of a pregnant woman
them to do nothing. for her growing fetus, which of the following
• Correct answers: c. This is a minerals are needed and responsible for fetal
mild reaction, and teaches bone and teeth formation?
them how to manage it. . Iron and Folic Acid
37. A nurse instructs parents who are concerned a. Calcium and Phosphorous
about the spread of illness at their children’s b. Vitamin D and E
daycare centers to inquire about the facilities: c. Sodium and Potassium
. CPR training for staff. • Correct answers: B. Calcium
a. Infection control practices. and Phosphorous
b. Reported cases of diarrhea during the previous 44. By the end of which of the following would the
year. nurse most commonly expect a child’s birth
c. Staff to child ratios. weight to triple?
• Correct answers: b. Infection . 4 months
control practices. a. 7 months
38. In preparing a preschool-age patient for an b. 9 months
injection, the most appropriate nursing c. 12 months
intervention is to: • Correct answers: D. 12
. Allow the patient to administer an injection to a months
doll. 45. In Leopold’s Maneuver step #1, you palpated a
a. Coordinate the patient watching a peer receive soft broad mass that moves with the rest of the
an injection. mass. The correct interpretation of this finding is:
b. Have the parents explain the process to the . The mass palpated at the fundal part is the head
patient. part.
c. Suggest diversionary activities like singing. a. The presentation is breech.
• Correct answers: b. b. The mass palpated is the back.
Coordinate the patient c. The mass palpated is the buttocks.
watching a peer receive an • Correct answers: D. The mass
injection. palpated is the buttocks.
39. When a 12-year-old child participates in a 46. Which of the following should the nurse do when
research study, the nurse’s primary concern is to a primipara who is lactating tells the nurse that
ensure that the: she has sore nipples?
. Parent or guardian has given verbal consent for . Tell her to breast feed more frequently.
the patient’s participation. a. Administer a narcotic before breast feeding.
a. Quality of care that the patient receives will not b. Encourage her to wear a brassiere.
be affected if the patient chooses to withdraw from the c. Use soap and water to clean the nipples.
study. • Correct answers: A. Tell her
b. Research meets the developmental needs of the to breast feed more
patient frequently
c. Research will directly benefit the patient. 47. To differentiate as a female, the hormonal
• Correct answers: b. Quality of stimulation of the embryo that must occur
care that the patient involves which of the following?
receives will not be affected . Increase in maternal estrogen secretion.
if the patient chooses to a. Decrease in maternal androgen secretion.
withdraw from the study. b. Secretion of androgen by the fetal gonad.
40. 40.What is the normal heart rate at birth? c. Secretion of estrogen by the fetal gonad.
. 120-160/min • Correct answers: D. Secretion
a. 100-120/min of estrogen by the fetal
b. 160-180/min gonad
c. Less than 100 48. When teaching a client about contraception.
• Correct answers: a. 120- Which of the following would the nurse include
160/min as the most effective method for preventing
41. Patient Sue is in her last trimester now and tells sexually transmitted infections?
the nurse “I’m most of the time having . Spermicides
constipation.” Which of the following should the a. Diaphragm
nurse recommend to the patient? b. Condoms
. Laxatives c. Vasectomy
a. Daily Enemas • Correct answers: C.
b. Increased fiber intake Condoms
c. Decreased fluid intake 49. Heartburn and flatulence, common in the second
• Correct answers: C. Increased trimester, are most likely the result of which of
fiber intake the following?
42. Your patient with a preeclampsia (severe) is . Increased plasma HCG levels
admitted with BP 170/100, with proteinuria, with a. Decreased intestinal motility
severe pitting edema Grade III. Which of the b. Decreased gastric acidity
following you would include in making your c. Elevated estrogen levels
patient’s plan of care? • Correct answers: C.
. Daily weights Decreased gastric acidity
a. Right lateral positioning 50. On which of the following areas would the nurse
b. Seizure precautions expect to observe chloasma?
c. Stress reduction . Breast, areola and nipples
• Correct answers: C. Seizure a. Chest, neck, arms and legs
precautions b. Abdomen, breast and thighs
c. Cheeks, forehead and nose
• Correct answers: D. Cheeks, VI. Be sensitive to the cultural needs of
forehead and nose and differences among children.
51. A newborn infant born prior to which of the VII. Head to toe assessment.
following gestational ages are considered • Correct answers: All of the
premature. above
. 37 weeks 59. Understanding Erickson’s stages of
a. 39 Weeks psychosocial development can help Pediatric nurses to:
b. 40 weeks I. Analyze patient’s symptomatic
c. 42 weeks behavior in the context of traumatic past
• Correct answers: 37 weeks experiences and struggles with current
52. Patency of the nares must be determined at birth developmental task.
because. II. Avoid conflict.
. Mouth breathing will exhaust the newborn. III. Communicate with a pediatric
a. The baby will have nasal flaring if not patent. patient, when you can anticipate the
b. Newborns are nose breathers. child’s reaction, it becomes easier to
c. Of the possibility of the polyps. modify your response to improve
• Correct answers: Newborns communication.
are nose breathers. IV. Be strict with pediatric patients.
53. Moments of hand hygiene are the following • Correct answers: I, II, III
except. only
. Before and after touching the patient. 60. Pediatric drug dosage is calculated according to:
a. After body fluid exposure risk. a. Age in weeks
b. After touching patient surroundings. b. Height in centimeters
c. Before any clean or aseptic procedure. c. Body weight in kilograms
d. After wearing a pair of gloves. d. All of the above
• Correct answers: After • Correct answers: Body weight
wearing a pair of gloves in kilograms
54. Personal Protective Equipment are strictly and 61. The nurse is caring for a client in labor. Which
routinely worn in the Labor and Delivery Rooms assessment finding indicates to the nurse that the client
during this Pandemic. True or false. The correct is beginning the second stage of labor?
sequence for Donning PPE is the following: . The contractions are regular.
gloves, mask/N95respirator, goggles/face shield, a. The membranes have ruptured.
gown. b. The cervix is completely dilated.
• Correct answers: False c. The client starts to expel clear vaginal fluid.
55. Referring back to no. 54 question, in what order • Correct answers: The cervix
would you doff (remove) the PP? is completely dilated.
. Gloves then mask 62. A client arrives at the birthing center in active
a. Goggles then mask labor. Her membranes are still intact, and the health care
b. Gloves, gown, goggles/faceshield, mask provider prepares to perform an amniotomy. What will
c. Gloves, mask, goggles then gown the nurse relay to the client as the most likely outcome of
• Correct answers: Gloves, the amniotomy?
gown, goggles/faceshield, . Less pressure on her cervix
mask a. Decreased number of contractions
56. A 3-year-old male child in Pediatric ward is b. Increased efficiency of contractions
noted with rapid shallow breathing, RR – 56 c. Increased maternal blood pressure
breaths/min, with on and off non-productive • Correct answers: Increased
cough. Upon Auscultation, coarse crackles are efficiency of contractions
heard on both lungs more on lower lobes. Which 63. A postpartum nurse is taking the vital signs of a
of the following nursing diagnosis a priority? client who delivered a healthy infant 4 hours ago. The
. Ineffective Breathing Pattern nurse notes that the client’s temperature is 38 degrees
a. Impaired Gas Exchange Celsius. Which of the following action would be
b. Ineffective Airway Clearance appropriate?
c. Risk for Infection . Notify the physician
• Correct answers: Ineffective a. Retake the temperature
Airway Clearance b. Refer to the Infection control link nurse
57. The age range for pediatric patients. c. Increase hydration by encouraging oral fluids
. Birth to puberty • Correct answers: Increase
a. Birth to 18 years hydration by encouraging
b. Infancy to 18 years oral fluids
c. One month to puberty 64. The ward nurse is providing instructions to a
• Correct answers: Birth to 18 client after Caesarian section delivery. The nurse
years instructs the client that she should expect normal bowel
58. The Nurses’ general approaches to Pediatric elimination to return which of the following?
physical assessment includes: . 3 days postpartum
I. Application of knowledge of growth a. 7 days postpartum
and development. b. On the day of delivery
II. Involve parents in the examination of c. 2 weeks postpartum
the child. • Correct answers: 3 days
III. Allow child to handle safe and clean postpartum
instruments. 65. A nurse is planning care for a postpartum client
IV. Establish trust. who had a vaginal delivery 1 hour ago. The client had a
V. Provide a quiet, private and safe midline Episiotomy. What is the priority nursing diagnosis
environment. for this client?
. Disturbed body image
a. Acute pain • Correct answers: Change the
b. Impaired bowel elimination client's position
c. Risk for imbalanced fluid volume 73. The nurse in charge is caring for a postpartum
• Correct answers: Acute pain vaginal delivery with a midline episiotomy. Which nursing
66. A newborn’s weight is important to help diagnosis takes priority for this client?
determine maturity and establish baseline data but it . Risk for deficient fluid volume related to
varies depending on which of the following? hemorrhage
. Race and genetic a. Risk for infection related to the type of delivery
a. Intrauterine b. Pain related to the type of incision
b. Nutritional c. Urinary retention related to peri urethral edema
c. All of the above • Correct answers: Risk for
• Correct answers: All of the deficient fluid volume
above related to hemorrhage
67. The following are the therapeutic dependent 74. Which changes would the nurse identify as a
nursing interventions the nurse should include to a progressive physiological change in postpartum period.
patient with Bacterial Pneumonia except for one: . Lactation
. Bronchodilators a. Lochia
a. Tepid Sponge Bath b. Urine evolution
b. Antipyretics c. Diuresis
c. Antibiotics • Correct answers: Lochia
• Correct answers: Tepid 75. A patient is in second stage of labor, during this
Sponge Bath stage, how frequently should the nurse in charge assess
68. Nurse Nica is assessing a patient with Viral her uterine contraction.
Pneumonia who has a Hypersensitivity type 1 immune . Every 5 minutes
response. Which of the following factors immediate a. Every 15 minutes
triggers to develop it except for one: b. Every 30 minutes
. Exposure may be by ingestion c. Every 60 minutes
a. Exposure may be by inhalation • Correct answers: Every 15
b. Exposure may be by direct contact minutes
c. Exposure may be blood borne 76. While caring for a patient who is hospitalized for
• Correct answers: Exposure acute gastroenteritis and dehydration, a pediatric nurse
may be blood borne notes the parent keeping packets of herbs by the
69. Which of the following is a way to protect patient's bedside. Suspecting that the parent may be
yourself from Dengue Hemorrhagic fever except? administering the herbs to the patient, the nurse's first
. Wear long sleeve shirts and long pants action is to.
a. Use mosquito repellent lotions . Ask the parent in a nonjudgmental manner about
b. Get rid of water containers the herbs
c. Put off mosquito net in an area with mosquitoes a. Coordinate a nursing care conference to discuss
• Correct answers: Put off the patient's plan of care
mosquito net in an area with b. Discuss the risks of using alternative therapies
mosquitoes with the parent
70. A nurse develops a plan of care for a woman c. Refer the family to a social worker for possible
with HIV and her newborn. The nurse includes which nonadherence with the healthcare regimen
intervention in the plan of care? • Correct answers: Ask the
. Monitor the newborn’s vital signs routinely parent in a nonjudgmental
a. Maintain standard precautions at all times while manner about the herbs
caring for the newborn 77. During the first two years of life, patients with
b. Initiate referral to evaluate the infection bronchopulmonary dysplasia requiring rehospitalization
c. Advise mother to have her infant’s antiviral are most likely to be diagnosed with:
infections yearly . Electrolyte disturbances
• Correct answers: Maintain a. Immune deficiencies
standard precautions at all b. Pulmonary hypertension
times while caring for the c. Respiratory tract infections
newborn • Correct answers: Respiratory
71. A client who is in labor and delivery has the tract infections
following assessment findings: Gravida 2 para 1, 78. When a patient participates in a research study,
estimated 40nweeks gestation, contractions 2 minutes the pediatric nurse's primary concern is to ensure that
apart, lasting 45 seconds, vertex + 4 station. Which of the:
the following would be the priority at this time? . Parent or guardian has given verbal consent for
. Placing the client in bed to begin fetal the patient's participation
monitoring a. Quality of care that the patient receives will not
a. Preparing for immediate delivery be affected if the patient chooses to withdraw from the
b. Checking for ruptured membrane study
c. Providing comfort measures b. Research meets the developmental needs of the
• Correct answers: Preparing patient
for immediate delivery c. Research will directly benefit the patient
72. Nurse Jenny Kim is caring for a client in labor. • Correct answers: Parent or
The external fetal monitor shows a pattern of variable guardian has given verbal
deceleration in fetal heart rate. What should the nurse do consent for the patient's
first? participation
. Prepare for emergency delivery 79. A 14-year-old boy with chronic asthma, who has
a. Check for placenta previa been hospitalized several times during the winter with
b. Administer oxygen severe asthmatic exacerbations, confides, "I wish I could
c. Change the client's position stay here in the hospital because every time that I go
home, I get sick again!" The pediatric nurse's best . Triceps
response is: a. Deltoid
. "I think that you should consider participating on b. Gluteal
a swim team to improve your pulmonary function." c. Vastus Lateralis
a. "Let's talk about preventing and managing your • Correct answers: Vastus
asthma on a daily basis at home." Lateralis
b. "Why don't I speak with your parents about what 87. The nurse has assessed four newborns RR
they are doing at home to help control your asthma?" immediately following birth. which RR would require
c. "Your insurance company does not pay for any further assessment by the nurse?
additional days of hospitalization that are not medically . 60 bpm
necessary." a. 70 bpm
• Correct answers: "Let's talk b. 64 bpm
about preventing and c. 20 bpm
managing your asthma on a • Correct answers: 20 bpm
daily basis at home." 88. Nurse Pia is assessing infantile reflexes in a 9-
80. Which of the following is not a sign of severe month-old baby, which of the following would she identify
dehydration in children? as normal?
. Lethargic or unconscious . Bilateral parachute
a. Skin pinch goes back readily a. Persistent rooting
b. Unable to drink or drinking poorly b. Absent moro reflex
c. Sunken eyes c. Unilateral grasp
• Correct answers: Skin pinch • Correct answers: Bilateral
goes back readily parachute
81. You are observing a 5-month-old for 89. An appropriate teaching was carried out by the
developmental skills. Which of the following skills would nurse if the parent of a 3 year old patient with acute
concern you if the infant was not demonstrating? gastroenteritis are aware of avoiding which of the
. Follows objects with the eyes following?
a. Crawling . Banana
b. Sitting with support a. Apple juice
c. Grasps object b. Ice cream
• Correct answers: Follows c. Plain rice
objects with the eyes • Correct answers: Ice cream
82. When administering magnesium sulfate to a 90. You're assessing a 10-month-old infant. You
client with preeclampsia, the nurse understands that this note on examination the anterior fontanelle is open. The
drug is given to: nurse will:
. Slow the progress of labor . Document this as an abnormal finding because
a. Facilitate diuresis this fontanelle should close at 2 months.
b. Reduce blood pressure a. Document this as a normal finding because this
c. Prevent seizure fontanelle closes at about 18 months.
• Correct answers: Prevent b. Document this as a normal finding because this
seizure fontanelle closes at about 12 months.
83. A newborn has small, whitish, pinpoint spots c. Document this as an abnormal finding because
over the nose, which the nurse knows are caused by this fontanelle should close at 6 months.
retained sebaceous secretions. When charting this • Correct answers: Document
observation, the nurse identifies it as: Single choice. this as a normal finding
. Dandruff because this fontanelle
a. Lanugo closes at about 18 months.
b. Milia 91. An infant can move his head from side to side
c. Mongolian Spots while following a moving object, can lift his head from a
• Correct answers: Milia prone position 45° off the examining table, smiles when
84. You’re assessing the one-minute APGAR score encouraged, and makes cooing sounds. He cannot
of a newborn baby. On assessment, you note the maintain a seated position. The most likely age of the
following about your newborn patient: heart rate 130 infant is.
bpm, pink body and hands with cyanotic feet, weak cry, . 1 month
flexion of the arms and legs, active movement and crying a. 3 months
when stimulated. What is your patient’s APGAR score? b. 6 months
. APGAR 9 c. 9 months
a. APGAR 10 • Correct answers: 3 months
b. APGAR 8 92. A child is brought to the clinic for a routine
c. APGAR 5 examine. She can dress with help, can ride a tricycle,
• Correct answers: APGAR 8 knows her own age, and can speak in short sentences.
85. A nurse in delivery room is assisting in the She had difficulty in copying a square. The age of this
delivery of a newborn infant. After the delivery, the nurse child is most likely.
prepares to prevent heat loss in the newborn resulting . 1 year
from evaporation by: a. 2 years
. Warming the crib pad b. 3 years
a. Turning on the overhead radiant warmer c. 4 years
b. Closing the doors • Correct answers: 3 years
c. Drying the infant in a warm blanket 93. A mother brings an 18-month-old to the
• Correct answers: Drying the emergency center with the concern that the child may
infant in a warm blanket have ingested a substance. Which of the following is a
86. Vitamin K is prescribed for a neonate. A nurse contraindication to the use of ipecac in this child.
prepares to administer the medication in which muscle . Age less than 5 years
site? a. Breast-feeding
b. Ingestion of alkali • Correct answers: ENC steps
c. Ingestion of iron of skin-to-skin contact
• Correct answers: Ingestion of properly timed cord
alkali clamping/cutting maybe
94. During the health interview, the mother of a 4- interchanged in cesarian
month-old says, “I’m not sure my baby is doing what he section babies.
should be.” What is the nurse’s best response? 100. You are attending to a mother and newborn
. “I’ll be able to tell you more after I do his being moved from the delivery table to the stretcher,
physical." what is the best position for her baby to be in.
a. “Fill out this developmental screening . Supine on the mother’s abdomen.
questionnaire and then I can let you know.” a. Supine beside the mother’s trunk.
b. “Tell me more about your concerns.” b. Prone on the mother’s abdomen.
c. “All mothers worry about their babies. I’m sure c. Prone on the mother’s chest.
he’s doing well.” • Correct answers: Prone on
• Correct answers: “Tell me the mother’s chest.
more about your
concerns.”
95. A 15-month-old girl is having her first health FINALS
supervision visit at your facility. Her mother has not 1. Gaile delivers an 8-lb boy after a 14-hour labor. In
brought a copy of the child’s immunization record but the post partal period, assessing for which of the
believes she is fully immunized: “She had immunizations following is the priority?
3 months ago at the local health department.” Which A. Postpartal hemorrhage
would be the best action by the nurse? B. Endometritis
. Ask the mother to bring the records to the 18- C. Thrombophlebitis
month health supervision visit. D. Amniotic embolus
a. Start the “catch-up” schedule because there are
no immunization records. 2. Christine has reached 8cm dilatation, you noticed
b. Keep the child at the facility while the mother that her fetal heart rate suddenly slows. On perineal
returns home for the records. inspection, you observe the fetal cord has prolapsed.
c. Call the local health department and verify the Your first action would be to:
child’s immunization status.
• Correct answers: Ask the A. Turn her to left side.
mother to bring the records B. Place her in knee-chest position.
to the 18-month health C. Replace the cord in gentle pressure.
supervision visit. D. Cover the exposed cord with a dry
96. The newborn should be prepared for and sterile gauze.
breastfeeding by giving.
. Sterile Water 3. After an hour of oxytocin therapy, a woman in
a. Glucose labor states she feels dizzy and nauseated. Your best
b. Cow's Milk action would be to:
c. None of the above
• Correct answers: None of the A. Assess the rate of flow of the
above oxytocin infusion.
97. Which of the following vaccinations should be B. Administer oral orange juice for added
given in the newborn period? potassium.
. Anti-measles C. Assess her vaginally for full dilatation.
a. Anti-hepatitis B D. Instruct her to breathe in and out
b. Anti-hepatitis A rapidly.
c. Anti-diphtheria
• Correct answers: Anti- 4. Which of the following is recommended for all
hepatitis B women during the childbearing age?
98. You are called to attend to a newborn about to
be delivered in a taxi outside the health facility. Your first A. Additional B vitamins
appropriate action should be. B. Additional vitamin A
. Milk the cord towards the newborn C. Folic acid supplement
a. Do skin-to-skin contact D. Vitamin C supplement
b. Clamp the cord immediately
c. Dry the baby thoroughly 5. Hanna has developed dystocia. You would
• Correct answers: Dry the explain that this term means:
baby thoroughly
99. You are called to attend a baby born by A. High blood pressure related to difficult
cesarean section. The mother requests that her newborn labor.
receives Essential Newborn Care (ENC). Which of the B. Difficult or abnormal labor.
following is an appropriate statement about ENC for C. Potential for placental detachment.
cesarean- delivered babies? D. Muscle weakness related to prolonged
. ENC follows the same sequence of steps as in labor.
vaginally born babies.
a. ENC is not doable for babies born by caesarian 6. The assessment findings for a client with an
section. incompetent cervix should contain which of the following?
b. ENC steps of skin-to-skin contact properly timed
cord clamping/cutting maybe interchanged in cesarian A. Anxiety and fear R/T situational self-
section babies. esteem.
c. None of the above B. History of one of his sisters in law with
the same diagnosis.
C. History of repeated, spontaneous
second trimester termination of A. A dark red discharge on a 2-day
pregnancy. postpartum client
D. The need to discuss the importance B. A pink to brownish discharge on a
of cervical test. client who is 5 days postpartum
C. Almost colorless to creamy discharge
7. Ann, a gravida 8, para 7 is in Ramos General on a client 2 weeks after delivery
Hospital only 15 minutes when she begins to deliver D. A bright red discharge 5 days after
precipitously. The fetal head begins to deliver as you walk delivery
into the labor room. Your best action would be to:
14. The Santos couple will undergo testing for
A. Place a hand gently on the fetal infertility. Infertility is said to be exist when:
head to guide delivery.
B. Ask her to push with next contraction A. A woman has no uterus.
so delivery is rapid. B. A woman has no children.
C. Assess BP and PR to detect placental C. A couple has been trying to
bleeding. conceive for one year.
D. Attach a fetal monitor to determine D. A couple has wanted a child for six
fetal status. months.

8. Faith, a pregnant mother with cardiac problem. 15. Kaye’s cousin on the other hand, knowing
When teaching the pregnant woman with Class II heart Marshal’s specialization asks what artificial insemination
disease, the nurse should advise her to: by donor entails. Which would be your best answer if you
were Nurse Marshal?
A. Gain at least 30 lbs.
B. Increase calcium in the diet. A. Donor sperm are introduced
C. Refrain strenuous activity. vaginally into the uterus or cervix
D. Inform her of the need t to limit fluid B. Donor sperm are injected intra-
intake to one liter. abdominally into each ovary.
C. Artificial sperm are injected vaginally
9. You assess that a fetus is in breech presentation. to test tubal patency.
Where will you auscultate for fetal heart sounds? D. The husband’s sperm is administered
intravenously weekly.
A. High in the abdomen
B. Left lateral abdomen. 16. Karla, a postpartal client calls you into her room
C. Low in the abdomen because she is having a heavy lochia flow containing
D. Right lateral abdomen large clots. Your first action would be to:

10. Which of the following is safe step to take during A. Assess her BP.
an induction of labor? B. Palpate her fundus.
C. Turn to her left side
A. Avoid the continuous use of both D. Assess her perineum.
uterine and fetal heart rate monitoring.
B. Observe the contractions are less than 17. Why are postpartal women prone to urinary
5 minutes apart and less than 70 seconds retention?
in duration.
C. Prepare oxytocin as piggyback A. Catheterization at the time of
solution as prescribed, being delivery reduces bladder tonicity.
extremely careful of the dose used. B. Decreased bladder sensation results
D. Urge the woman to remain on her back from edema due to pressure of birth.
during labor to offer a good blood supply C. Frequent partial voiding never relieves
to the uterine muscle. the bladder pressure.
D. Mild dehydration causes a
11. Which of the following is the most frequent reason concentrated urine volume in the urine.
for post partal hemorrhage?
18. You administer methylergometrine (Methergine),
A. Endometritis 0.2 mg, to a postpartal woman with uterine subinvolution.
B. Uterine atony Which of the following assessment should you make
C. Perineal laceration before administering medication?
D. Deciminated intravascular coagulation
A. Her urine output is more than 50
12. A postpartal woman has a fourth-degree ml/hour.
laceration. Which of the following physician’s order would B. Her BP is below 140/90mmHg.
you question? C. She can walk without experiencing
dizziness.
A. Urging her to drink all the milk on D. Her hematocrit level is higher than
her tray. 45%.
B. Administration of acetaminophen for
pain. 19. You are caring for Irene who has had a baby by
C. Administration of sitz bath cesarean birth. Which of the following would be the most
D. Administration of enema. important assessment to make?

13. The nurse assesses the postpartum vaginal A. Whether her abdomen is soft or not.
discharge (lochia) on four clients. Which of the following B. Whether her perineum is edematous.
assessments would warrant notification of the physician?
C. If her breast fill with milk by the third C. Bricanyl (Terbutaline)
day. D. Betamethasone
D. If she wants to breastfeed or not.
27. Following a cesarean birth, Beah has 3,000 ml of
20. On the second day postpartum following a IVF ordered. You anticipate in your plan of care that she
cesarean birth, at which of the following locations would will be kept NPO except for minimal ice chips until which
you expect to palpate the woman’s fundus? time?

A. Two fingers above the umbilicus. A. Until 24-hr post procedure.


B. At the umbilicus. B. Until 48-hr post procedure.
C. Two fingers below the umbilicus. C. Until bowel sounds have returned.
D. Four fingers below the umbilicus. D. Until her bladder tone has returned.

21. Which assessment on the third postpartal day 28. To prevent thrombophlebitis following a cesarean
would make you evaluate Preggy as having uterine birth, which of the following would be most important to
subinvolution? implement?

A. Her uterus is 2 cm above the A. Urge the client to cough and take
symphysis pubis. breaths.
B. Her uterus is 3 fingerwidth under the B. Encourage to ambulate.
umbilicus. C. Urge the client not to dislodge the IVF
C. Her uterus is at the level of the line.
umbilicus. D. Instruct the client to press inward on
D. She experiences “pulling” pain while her abdomen periodically.
breastfeeding.
29. Which of the following women will most likely not
22. A postpartal woman with thrombophlebitis tells be a candidate to attempt a vaginal birth a after having
you that her legs are very painful. Which of the following had a previous cesarean section?
actions would be most appropriate to relieve this pain?
A. Woman who had a cesarean section
A. Massage the calf of the legs. because of placenta previa.
B. Keep covers off the legs. B. Woman who had a cesarean section
C. Apply ice to her leg above the knee. because of cephalopelvic disproportion.
D. Urge her to walk to relieve muscle C. Woman who has a gynecoid-shaped
spasm. pelvis.
D. Woman who had a cesarean section
23. The type of assisted reproductive method. Which because of a breech presentation.
is done by instilling ova and sperm into the fallopian tube
for fertilization is 30. Dinah expresses concern about sexual activity
during pregnancy. She has heard that intercourse is
A. IVF harmful to a fetus and that, besides, it is uncomfortable for
B. GIFT pregnant women. An appropriate reply by Dinah’s nurse
C. ZIFT would be to:
D. Cloning
A. tell her that intercourse should be
24. Beah is scheduled to have epidural anesthesia avoided during the last month of
for a cesarean birth. Which of the following would you pregnancy.
anticipate including in the preoperative plan of care while B. reassure her that intercourse during
she waits for the anesthetic? pregnancy is neither harmful nor
uncomfortable.
A. Encourage her to ambulate. C. ask whether she and her partner are
B. Administering an oral antacid. having sexual problems.
C. Administering morphine sulfate IM. D. state that her fears are normal and
D. Keeping her turned on her side. suggest she and her partner try various
positions for intercourse.
25. Beah asks you if she will have any difficulty
breastfeeding following a cesarean birth. Your best 31. Before assessing the postpartum client’s uterus
response would be that: for firmness and position in relation to the umbilicus and
midline, which of the following should the nurse do first?
A. You do not recommend she try to
breastfeed following a cesarean birth. A. Assess the vital signs.
B. Although she can try, it is hard to find B. Administer analgesia.
a comfortable position to hold a newborn C. Ambulate her in the hall.
to breastfeed. D. Assist her to urinate.
C. She will need too much analgesia
postoperatively to make breastfeeding 32. Which of the following should the nurse do when
safe. a primipara who is lactating tells the nurse that she has
D. You will help her find a comfortable sore nipples?
position for breastfeeding her infant.
A. Tell her to breast feed more frequently.
26. Which of the following drugs does the nurse B. Administer a narcotic before breast
expect to be ordered if preterm delivery is inevitable? feeding.
C. Encourage her to wear a nursing
A. Yutopar (Ritodrine) brassiere.
B. Pitocin (Oxytocin)
D. Use soap and water to clean the A. Post term delivery
nipples. B. Epidural anesthesia
C. Grand multiparity
33. The nurse assesses the vital signs of a client, 4 D. PROM
hours’ postpartum that are as follows: BP 90/60;
temperature 38.5ºC; pulse 100 weak, thready; RR- 20 per 40. Mrs. Lovely. A 37-year-old post-partum client is at
minute. Which of the following should the nurse do first? risk of thrombophlebitis. Which of the following nursing
intervention decreases her chance of developing
A. Report the temperature to the thrombophlebitis?
physician
B. Recheck the blood pressure with A. Breastfeeding the newborn
another cuff. B. Early ambulation
C. Assess the uterus for firmness and C. Administration of anticoagulant.
position. D. Immobilization and elevation of legs
D. Determine the amount of lochia.
41. The medical record of a client reveals a condition
34. A postpartum client has a temperature of 38ºC, in which the fetus cannot pass through the maternal
with a uterus that is tender when palpated remains pelvis. The nurse interprets this as:
unusually large, and not descending as normally
expected. Which of the following should the nurse assess A. Contracted pelvis
next? B. Cephalopelvic pelvic disproportion
C. Cervical insufficiency
A. Lochia D. Maternal disproportion
B. Breasts
C. Incision 42. A positive Homan’s sign indicates which of the
D. Urine following?

35. The nurse would anticipate a cesarean birth for a A. Possible mastitis
client who has its infection at the onset of labor? B. Probable UTI
C. Possible post-partum hemorrhage
A. Toxoplasmosis D. Probable deep vein thrombosis
B. Herpes-simplex virus
C. Human papilloma virus 43. Mrs. Makerot has been diagnosed with puerperal
D. Hepatitis (postpartum) infection. Which of the following events most
likely contributes to the development of puerperal
36. The rationale for using a prostaglandin gel for a infection?
client prior to the induction of labor is to:
A. Prolonged 1st stage of labor more than
12 hours
A. Soften and efface the cervix. B. Prolonged rupture of membranes
B. Numb cervical pain receptors more than 24 hours
C. Prevent cervical lacerations. C. Midline episiotomy
D. Stimulate uterine contractions. D. Multiparity

37. Another client named Kaye is diagnosed as 44. There are number of factors that lead to male
having endometriosis. This condition interferes with infertility. Which of the following factors prevent sperm
fertility because: motility or movements?

A. Endometrial implants can block the A. Spermatogenesis


fallopian tubes. B. Changes in the seminal fluid
B. The uterine cervix becomes inflamed C. Ejaculation problems
and swollen. D. Obstruction of the seminiferous
C. The ovaries stop producing adequate tubules
estrogen.
D. Pressure in the pituitary leads to 45. Which of the following is the most common cause
decreased FSH levels. of Syphilis?

38. Kaye’s cousin on the other hand, knowing A. Escherichia coli


Marshal’s specialization asks what artificial insemination B. Neisseria gonorrheae
by donor entails. Which would be your best answer if you C. Staphylococcus aureus
were Nurse Marshal? D. Treponema pallidium

A. Donor sperm are introduced vaginally 46. Which drug would the nurse expect to administer
into the uterus or cervix. to a woman with infertility secondary to anovulation to
B. Donor sperm are injected intra- suppress the hypothalamus?
abdominally into each ovary.
C. Artificial sperm are injected vaginally A. Clomipine (Clomid)
to test tubal patency. B. Bromocriptine (Parlodel)
D. The husband’s sperm is administered C. Levothyroxine (Synthroid)
intravenously weekly. D. Testosterone Cypionate

39. Which of the following findings on a newly 47. Collection of semen for analysis, and must be
delivered woman’s chart would indicate she is at risk for repeated after 2-3 months is:
developing postpartum hemorrhage?
A. After 2-4 days of sexual abstinence.
B. After 1 month of sexual abstinence. A. 1, 2, 6
C. After sexual contact. B. 3 & 5
D. After sexual arousal C. 4 & 6
D. All of the above
48. Which of the following is unlikely to happen when
the fetal membranes rupture? 56. Beta-adrenergic drugs that decrease effect of
calcium on muscle activation to slow or stop uterine
A. imminent onset of labor contractions.
B. risk of infection
C. risk of cord prolapsed. A. Terbutaline and Ritodrine
D. maternal hypovolemia B. Nifedipine
C. Magnesium sulfate
49. When Mrs. Dimatantu is receiving discharge D. Indomethacin
instructions from the nurse, she asks the nurse when she
can resume sexual intercourse. The nurse states that 57. Which of the following clients are at risk of post-
sexual intercourse can be resumed: partum hemorrhage?
1. Mothers with macrosomia baby
A. 8 weeks after giving birth. 2. Polyhydramnios
B. With doctor approval 3. Precipitous labor
C. When the lochia has stopped. 4. Obese
D. At the man’s discretion 5. Multiparity

50. When Mrs. Dimatantu is receiving discharge A. 1,2


instructions from the nurse, she asks the nurse when she B. 3,4
can resume sexual intercourse. The nurse states that C. 4,5
sexual intercourse can be resumed: D. All of the above

A. 8 weeks after giving birth. 58. Which of the following is not a common cause of
B. With doctor approval early postpartum bleeding?
C. When the lochia has stopped.
D. At the man’s discretion A. Atony
B. Laceration
51. Following are steps in in vitro fertilization 1. C. Subinvolution
Capture of ova 2. Ovulation 3. Fertilization of ova and D. Hematoma
growth 4. Insertion of fertilized ova
59. Mrs. Dimagiba has continued to breast-feed her
A. 2,1,3,4 infant every 3-4 hours except night. About 8 hours after
B. 3,4,1,2 delivery, she complains that her breast has become
C. 4,3,2,1 larger, firmer and tender. Nurse Dee recognizes that Mrs.
D. 1,2,3,4 Dimagiba is probably experiencing:

52. Which of the following symptoms is least A. Mastitis


important in characterizing postpartum “blues”? B. Engorgement
C. Let-down reflex
A. Crying easily and feeling despondent. D. Involution
B. Loss of appetite and anxiety
C. Altered body image. 60. A feeling of ambivalence about the pregnancy is:
D. Difficulty sleeping and poor
concentration. A. a sign of unwanted pregnancy
B. normal in early pregnancy
53. The most susceptible veins for thrombophlebitis C. rare at any stage of pregnancy
are the following except: D. typical in late pregnancy

A. pelvic 61. The risk of postpartum thromboembolism occurs


B. hip because of physiologic increase in:
C. upper extremities
D. lower leg A. Coagulation factors
B. Heart rate
54. In reviewing discharge instructions with Mrs. C. Diuresis
Dimatantu, Nurse Veah is aware that the higher incidence D. Blood pressure
of postpartum “blues” can result from:
62. What should the nurse expect as a possible
A. Fatigue cause of bleeding after 30 minutes after birth wherein the
B. Subinvolution fundus is firm and increasing amount of lochia rubra and
C. Neonatal jaundice a few large clots?
D. Pregnancy-induced hypertension
A. Prolonged 2nd stage of labor
55. Methods used to initiate induction of labor. B. Primiparous status
1. Oxytocin C. Inadequate amount of oxytocin
2. Amniotomy D. Retained placental fragments.
3. Cesarean section
4. Terbutaline 63. Which option below is considered a positive
5. Internal Examination Homan's Sign for the assessment of a deep vein
6. Prostaglandin gel suppository form thrombosis (DVT)?
A. The patient reports pain when the foot C. Schedule follow-up care every 2
is manually plantarflexed. weeks.
B. The patient experiences pain when the D. Use the stairs to increase activity level.
leg is extended.
C. the patient experiences pain when the
leg is flexed. 71. A mother receiving medications for pregnancy
D. The patient reports pain when the foot induced hypertension should have her diastolic blood
is manually dorsiflexed. pressure maintained in the range of 90 to 100 mmHg to:

64. The lower leg of a client with thrombophlebitis A. avoid causing fetal anoxia.
looks different than that of the unaffected leg with the 3 B. ensure progression of labor.
symptoms. C. prevent premature contractions.
1. Cool extremity D. present sudden elevations in pulse.
2. Redness
3. Warm extremity 72. Which of the following would be a priority
4. Swelling intervention for a client with the prolapsed cord?
5. Cyanosis
6. Pain A. Cover the cord with a dry sterile tower.
B. Monitor the mother’s vital signs.
A. 2,3,4 C. Place the woman in the
B. 1,2,3 Trendelenburg position
C. 4,5,6 D. Start medication as ordered.
D. All of the above
73. A gravida 3 para 3 patient delivers a term
65. Which of the factors below regarding a deep vein newborn. The patient is blood group O and her infant is
thrombosis is not included in Virchow's Triad? blood group A. the baby begins to develop jaundice at 12
hours of age. This newborn’s jaundice is most likely
A. Hypercoagulability caused by:
B. Increased venous blood flow.
C. Stasis of venous blood A. Sepsis neonatorum
D. Endothelial damage B. Physiologic jaundice
C. Rh incompatibility
66. Which assessment relates most directly to D. ABO blood group incompatibility
rupture membranes and release of amniotic fluid?
74. Which of the following is true about Intra
A. Bloody show Cytoplasmic Sperm Injection?
B. Fluid with a pH of 7.0 to 7.5 with
nitrazine test A. Sperm is directly introduced into ovum
C. Fluid with a pH of 5.0 with nitrazine in a culture medium in the laboratory and
test then the zygote/embryo is transferred in
D. Woman complains of urge to push. the fallopian tube or uterus.
B. Transfer of ovum collected from donor
67. In evaluating the effects if oxytocin after delivery, into fallopian tube of another female.
the nurse should monitor for: C. Semen either collected from the
husband or a donor is artificially
A. effective breastfeeding. introduced either into a vagina or into the
B. engorged breasts. uterus of the female.
C. relief of pain.
D. firmness of the uterus. 75. A patient in labor would require a cesarean
delivery if she had:
68. Which of the following contraceptive methods
also offers protection against sexually transmitted A. Syphilis
infections? B. Toxoplasmosis
C. Cytomegalovirus
A. Abstinence D. Genital herpes
B. Coitus interruptus
C. Fertility awareness methods 76. Based on Mrs. Aniametten twin delivery, the most
D. Oral contraceptives important intervention the nurse should implement is to:

69. Which assessment most closely relates to a A. Assess fundal tone and lochia.
diagnosis of ectopic pregnancy? B. Apply cold pack to the perineal area.
C. Administer analgesic as ordered.
A. Brownish red, tapioca-like vesicles D. Encourage voiding by offering
B. Elevated temperature bedpan.
C. Spotting or bleeding 2 to 3 weeks after
a missed menstrual period 77. Turucan reads a Doctor’s Order of Cefazolin1g
D. Sudden absence of fetal movement IVP q 120 ANST. The nurse is correct if he interprets that:

70. Which nursing intervention would be appropriate A. The patient needs to be skin tested
for a client who has a diastolic blood pressure of more first before giving the medication
than 20 mmHg on the “roll-over” test? S intravenously.
B. The medication will be given through
A. Increase intake of oral fluids Intravenous Push.
B. Rest on left side as much as possible C. The medication will be given every 12
hours following the initial administration.
D. The patient has negative skin testing. 84. Day with preterm rupture of membranes has
blood work ordered daily. The results of which of the
78. When taking an obstetrical history on a pregnant following would be most important to assess?
client who estates, “I had a son born at 38 weeks’
gestation, a daughter born at 30 weeks’ gestation, and I A. Hemoglobin and hematocrit
lost a baby at about 8 weeks’,” the nurse should record B. Sodium and potassium level
her obstetrical history as which of the following? C. Serum creatinine
D. White blood count
A. G2 T2 P0 A0 L2
B. G4 T1 P1 A1 L2 85. Which of the following nursing diagnosis would be
C. G3 T1 P1 A0 L2 most appropriate for a woman diagnosed with PIH?
D. G3 T2 P0 A0 L2
A. Deficient fluid volume related to
79. The nurse records the progress of labor of jail. vasospasm of arteries.
One contraction started at 8:40 and ended 8:41. The next B. Ineffective tissue perfusion related to
contraction started at 8:45 and ended 8:46. Which of the poor heart contraction.
following refers to the period between 8:41 to 8:45? C. Risk for injury related to fetal distress.
D. Imbalanced nutrition related to
A. Intensity decreased sodium levels.
B. Duration
C. Interval 86. Zesa is being admitted to TGH for severe
D. Frequency preeclampsia. When deciding on where to place her, the
most appropriate area for Zesa is?
80. A public health nurse would instruct a pregnant
woman to notify physician immediately if which of the A. Near the nursery so she can maintain
following symptoms occur during pregnancy? hope she will have a child.
B. Near the elevator so she can be
A. Presence of dark color of the neck. transported easily.
B. Increased vaginal discharge. C. Near the nurse’s station so she can be
C. Swelling of the face. observed closely.
D. Breast tenderness. D. In the back hallway where there is a
quiet, private room.
81. Paula asks her nurse about the use of vitamin and
mineral supplements during the second and third 87. Mrs. Reyal was admitted to the OB ward in active
trimesters of pregnancy. Which of the following labor. During contraction, the nurse observes a 15 beats
recommendations would be most appropriate for the per minute deceleration of the FHR below the baseline
nurse to make? rate. What would be the most appropriate action?

A. Pregnant women with an adequate A. Prepare for immediate delivery


diet do not need vitamin or mineral because the fetus is in distress.
supplements. B. Call the physician immediately and
B. All pregnant women should take a await the orders.
multivitamin capsule daily. C. Turn the client on her left side to
C. Pregnant women should supplement increase venous return.
their diet with a minimum of 30 mg of iron D. Record this normal fetal response to
and 100 mg of vitamin C daily. contraction on the chart
D. Pregnant women should supplement
their diet with a minimum of 30 mg of iron 88. Which of the following is the most common sign
and 0.2 mg of folate daily. of preeclampsia?

82. Mrs. Nagdurugo is at risk for abruption placenta. A. Proteinuria and hypertension
Which of the following assessment would most likely lead B. Hypertension and hyporeflexia
you to suspect that this has happened? C. Glucosuria and proteinuria
D. Angle edema and glucosuria
A. Sharp fundal pain and discomfort
between contraction. 89. Mars is admitted with a diagnosis of Ectopic
B. Painless vaginal bleeding and a fall in pregnancy. For which of the following would you
blood pressure. anticipate beginning preparation?
C. Pain in the lower quadrant and
increase pulse rate. A. Bed rest for the next 4 weeks.
D. An increase blood pressure and B. Intravenous administration of tocolytic.
oliguria. C. Immediate surgery.
D. Internal uterine monitoring
83. What helpful strategies will help you as a nurse
and the patient in condition like Mrs. Matulungin with a 90. Which of the following would alert the nurse that
diagnosis of placenta previa? the client maybe about to experience seizure in PIH
client?
A. Therapeutic counseling and your
presence. A. Decreased contraction intensity.
B. Listening to the fetal heart sounds and B. Decreased temperature.
emphatic caring words. C. Epigastric pain
C. Reassurance your presence and D. Hyporeflexia
conversation.
D. Attend to her, diet and hygiene needs.
91. In performing a routine fundal assessment, the 98. Which finding is most characteristic of perineal
nurse finds Mrs. Dimawari’s fundus to be boggy. The hematoma?
nurse should first:
A. Fever
A. Call the doctor B. Lethargy
B. Massage the fundus C. Positive homan’s sign
C. Assess lochia flow. D. Severe vulvar pain
D. Start giving oxytocin.
99. Which of the following is the most common cause
92. Which of the following situations is more likely to of mastitis?
predispose a patient to postpartum hemorrhage?
A. Escherichia coli
A. Birth of 3,175 grams B. Neisseria gonorrheae
B. Birth of twins C. Staphylococcus aureus
C. Prolonged 1st stage of labor D. Treponema pallidum
D. PIH
100. Which of the following signs and symptoms
93. How the nurse determines the frequency of indicated postpartum client is experiencing cystitis?
uterine contractions?
A. Urinary retention
A. Time from the beginning of one B. Burning sensation on urination.
contraction to the beginning of the next C. Dribbling of urine
contraction. D. Difficulty in starting urinary stream.
B. Time from the beginning of one
contraction to the end of the next
contraction. NCM 109 Pedia Lec
C. Time from the peak of one contraction
Midterms
to the peak of the next contraction.
D. Time from the end of one contraction
1.SITUATION: Demy a 3 year old patient brought in TPH
to the beginning of the next contraction.
emergency room with cystic fibrosis. Lynn was assigned
in pediatric ward. 1. A sputum specimen is collected from
94. In the event of precipitous labor, which of the
Demy for culture purposes, the primary reason being to
following is the most important action?
learn. Single choice.
a. whether the child's cystic fibrosis is under control.
A. Call the doctor
b. what organism is responsible for the child's
B. Remain with the patient and coach
disease.
her.
c. if there is blood present in the child's lungs
C. Push the fetal back until equipment
tracheobronchial tree
are assembled.
d. which portion of the child's lungs is affected.
D. Ask the patient to do shallow, rapid
breathing.
Correct answers: what organism is responsible for the
child's disease.
95. Which of the following characteristics is most
commonly associated with LGA?
2. Which of the following statements best defines cystic
fibrosis? Single choice.
A. Weight under 4000 grams
a. an acquired lung condition that occurs at birth due
B. Hypothermia
to oxygen deficiency 1
C. Risk for birth injury
b. a congenital anomaly occurring during fetal
D. Dysmorphic features
development as a result of dietary deficiencies of the
mother
96. Which of the following factors is not typically lead
c. an autosomal recessive disorder that mainly
to male subfertility?
affects the exocrine glands and does multiple system
damage
A. Disturbance in spermatogenesis
d. a respiratory condition affecting people of
(production of sperm cells)
Mediterranean descent, involving mainly exchange at a
B. Obstruction in the seminiferous
microscopic level in the lungs
tubules, ducts, or vessels preventing
movement of spermatozoa.
Correct answers: an autosomal recessive disorder that
C. problems of ova transport through the
mainly affects the exocrine glands and does multiple
fallopian tubes to the uterus.
system damage
D. ejaculation problems
3. Which of the following represents the life expectancy of
97. The client is on her 12th week of pregnancy. She
a child with cystic fibrosis?. Single choice.
has been diagnosed to have ruptured ectopic pregnancy.
a. 21 year old
Which of the following signs and symptoms are
b. 30 year old
characteristics of the condition?
c. 12 year old
d. 7 year old
A. Profuse bright red vaginal bleeding.
B. Spotting, abdominal pain that radiates
Correct answers: 30 year old
to the shoulder.
C. Elevated hemoglobin and hematocrit
4. Lynn can expect Demy's to have stools that are
levels.
typically. Single choice.
D. Leukopenia decreased body
a. bulky and foul odor
temperature.
b. dry and have the odor of ammonia
c. watery and sweet in odor 11.During assessment, a physical finding John is most
d. dark in color and have almost no odor likely to observe is that Matt has. Single choice.
a. an enlarged liver
Correct answers: bulky and foul odor b. a protuberant abdomen
c. a tender inguinal lymph nodes
5.During the warm weather, the nurse will assess children d. edema in the lower extremities
with cystic fibrosis to see if they need which of the
following supplements added to their diet?. Single choice. Correct answers: a protuberant abdomen
a. zinc
b. magnesium 12.The family of a child with celiac’s disease is receiving
c. salt discharge teaching in regard to diet. The nurse will teach
d. potassium the family to modify the diet in which of the following
ways?. Single choice.
Correct answers: salt a. Eliminate wheat, barley, rye, and oats and
substitute corn, rice, and millet.
6.Demy is to have postural (bronchial) drainage. The b. Increase bulk including raw vegetables, fruit, and
times in the day when it is best for Lynn to plan to carry a variety of cereals and nuts.
out postural drainage is shortly. Single choice. c. Eliminate fats but increase natural fish oils and
a. after meals olive oils.
b. before meals d. Eliminate lactose products such as milk, cheese,
c. before rest periods ice cream, and butter.
d. after rest periods
Correct answers: Eliminate wheat, barley, rye, and oats
Correct answers: before meals and substitute corn, rice, and millet.

7.In working with children with cystic fibrosis, the nurse 13.For how long is it likely that Matt will need to remain on
expects to carry out and provide teaching on which of the a special diet. Single choice.
following treatments?. Single choice. a. For the rest of his life.
a. dietary modification involving a low protein, high b. Until the disease is well controlled.
fat, low carbohydrate diet c. Until approximately the age of puberty.
b. range of motion and a highly regimented exercise d. For the period required to desensitize him to
program offending foods.
c. preparation for renal dialysis and strict intake and
output Correct answers: For the rest of his life.
d. eto answer
14.Matt's wears reusable diapers at night. His mother
Correct answers: asks what safety precautions she should take his wet,
soiled diapers. Which of the following statements should
8.Lynn should understand prior to evaluating the effects guide the nurse when she responds to Matt's mother? ..
of postural drainage that the primary reason for Demy's Single choice.
having this type of therapy is ton help to. Single choice. a. The diaper should be boiled after they are
a. dilate the bronchioles. washed.
b. promote better use of diaphragm. b. The diapers should be soaked in an antiseptic
c. improve circulation in the chest cavity. solution before they are washed
d. clean the lungs of mucopurulent material. c. It would be best to use disposable diapers until
the diarrhea is under control.
Correct answers: clean the lungs of mucopurulent d. There are no special precautions necessary in
material. the care of this infant's diaper.

9.SITUATION: Mr. John a new staff nurse was assigned Correct answers: There are no special precautions
in the pediatric ward that is caring for 18 months old with necessary in the care of this infant's diaper.
celiac disease. Matt is admitted to the pediatric unit with a
diagnosis of celiac disease. John who was assigned to 15.Which of the following makes children prone to
this child realizes that the typical stool of a child with this tonsillitis and pharyngitis?. Single choice.
disease will be:. Single choice. a. not brushing their teeth and flossing after meals
a. black and tarry and at bedtime
b. normal colored and pencil thin or ribbonlike in b. frequent upper respiratory tract infections and
appearance being around other children who may be infected
c. bulky, putty colored, foul smelling, greasy, and c. eating large amounts of sugar and drinking high
likely to float sugar content drinks that encourage growth of pathogens
d. white and loose d. not bathing properly or frequently

Correct answers: bulky, putty colored, foul smelling, Correct answers: frequent upper respiratory tract
greasy, and likely to float infections and being around other children who may be
infected
10.The nurse should expect that, because Matt has celiac
disease, his stools are most likely to be. Single choice. 16.The school nurse encourages the parents of a student
a. especially dark in color who has a severe sore throat to take the child to the
b. abnormally small in amount pediatrician. The nurse is especially concerned about the
c. unusually hard in consistency possibility of a streptococcal tonsillitis mainly because of
d. particularly offensive in odor which of the following?. Single choice.
a. If streptococcal tonsillitis is not treated adequately
Correct answers: particularly offensive in odor it may lead to scarlet fever, otitis media, or more serious
complications.
b. The student will miss a lot of school and fall
behind in schoolwork and homework assignments and 21.The nurse is assigned to care for a child who had a
may even possibly fail a grade. tonsillectomy yesterday. The child complains of a sore
c. Other children may also get sore throats if this throat and wants something cool. Which of the following
student is not aggressively treated. cool drinks or food would be acceptable to offer the child?.
d. If untreated, streptococcal tonsillitis may cause Single choice.
anemia, weight loss, and lethargy. a. lime sherbet
Correct answers: If streptococcal tonsillitis is not treated b. chilled tomato juice
adequately it may lead to scarlet fever, otitis media, or c. cold cola
more serious complications. d. cherry Jell-O

17.The mother of a two-and-a-half-year old child who has Correct answers: lime sherbet
had tonsillitis asks about the possibility of a tonsillectomy
for the child. The nurse informs the mother that 22.Which of the following signs or symptoms is the
tonsillectomies are usually performed when the child is earliest manifestation of bleeding in a child who has had
over three years old. The mother wants to know why a tonsillectomy?. Single choice.
children younger than three have to wait for a a. frequent swallowing
tonsillectomy. Which of the following is the nurse’s best b. spitting up blood
answer?. Single choice. c. coffee ground emesis
a. “Surgery is too difficult on a younger child d. complaint of thirst
because their mouths are so small, making it difficult to
get to the tonsils.” Correct answers:
b. “It is too scary for a younger child and they don’t
understand what is happening.” 23.Which of the following outcomes is the most important
c. “Younger children are more prone to excessive for a child who has had a tonsillectomy?. Single choice.
blood loss and/or the tonsils growing back.” a. no postoperative bleeding
d. “Separation from the parents or caregivers is b. bonding with the nursing staff
much more difficult in the child under three.” c. absence of pain
d. drinking sufficient water
Correct answers: “Younger children are more prone to
excessive blood loss and/or the tonsils growing back.” Correct answers: no postoperative bleeding

18.The nurse explains to parents why an adenoidectomy 24.The nurse assessing a child with a respiratory problem
is sometimes indicated with the tonsillectomy. Which of finds stridor, which is best described by which of the
the following is the nurse’s best explanation?. Single following?. Single choice.
choice. a. excursion of the chest wall beyond that normally
a. An adenoidectomy is justified when a child’s seen in the healthy child
enlarged adenoids block the flow of air through the nasal b. a high-pitched sound produced by an obstruction
passages.” of the trachea or larynx
b. “Some surgeons prefer to take out the adenoids c. Message for respondents who select this answer
at the same time as the tonsillectomy because it saves d. a triad of respiratory symptoms
the child from a second surgery.” e. substernal retractions and nasal flaring
c. Without the tonsils to screen out foreign proteins,
the adenoids are more easily infected.” Correct answers: a high-pitched sound produced by an
d. “It is hard to take out the tonsils without removing obstruction of the trachea or larynx
the adenoids because they often are adhered to each
other.” 25.Examination of the throat of a child with acute epiglottis
by depressing the tongue is. Single choice.
Correct answers: An adenoidectomy is justified when a a. a good idea because there is a characteristic
child’s enlarged adenoids block the flow of air through the patchy redness that aids in diagnosis
nasal passages.” b. contraindicated due to the possibility of causing
complete airway obstruction
19.Which of the following positions does the nurse use for c. impossible because the tongue is too swollen and
the child who has just returned to the room from surgery?. tender to allow it
Single choice. d. to be done carefully because of the possibility of
a. elevated head and legs causing additional pain
b. on the abdomen or side
c. on the back and flat Correct answers: contraindicated due to the possibility
d. on the back or sitting up of causing complete airway obstruction

Correct answers: on the abdomen or side 26.The nurse is assessing a child with croup. Which of the
following signs and/or symptoms would the nurse
20.A child who is in the first postoperative day after associate with croup and most expect to find?. Single
tonsillectomy starts to cough and wants to blow his nose. choice.
Which of the following actions by the nurse would be a. barklike cough, hoarseness, inspiratory stridor
best?. Single choice. b. expiratory stridor, irregular breathing, and weak
a. Offer the child some tissue and give instructions cough
on its disposal. c. Cheyne-Stokes respirations and substernal
b. Remind the child not to cough or blow his nose. retraction
c. Tell him he can cough but not to blow his nose. d. almost continual cough and complete laryngitis
d. Inform the child that he can blow his nose but not
cough. Correct answers: barklike cough, hoarseness,
inspiratory stridor
Correct answers: Tell him he can cough but not to blow
his nose.
27.Which of the following statements best describes Correct answers: raising the head of the child’s bed
croup?. Single choice.
a. a common childhood disease 33.Pneumonia in children appears:. Single choice.
b. an autoimmune-initiated infection a. in a more primitive form which is easier to cure
c. a life-threatening bacterial infection than that found in adults
d. a respiratory condition similar to a cold b. as a secondary disease
c. as a tertiary disease
Correct answers: an autoimmune-initiated infection d. as a primary disease or a complication of another
disease
28.The nurse is assessing the skin moisture of a child with
laryngotracheobronchitis. Which of the following explains Correct answers: as a primary disease or a complication
why the nurse assessed the skin moisture in this case?. of another disease
Single choice.
a. diaphoresis is associated with increased 34.Which of the following types of pneumonia would be
respiratory effort treated with antibiotics?. Single choice.
b. skin moisture is a relatively good measure of the a. viral
degree of infection b. Bacterial
c. increased skin moisture will signal danger of c. Aspiration
ketoacidosis d. adenoviral
d. to get some measure of how dehydrated the child
might become Correct answers: Bacterial

Correct answers: diaphoresis is associated with 35.The nurse is assessing a child with a diagnosis of
increased respiratory effort pneumonia. The child is taking shallow respirations.
Which of the following is the most likely explanation?.
29.The nurse working with a child with Single choice.
laryngotracheobronchitis is offering the child tiny medicine a. a deficiency of carbon dioxide
cups of fluid and giving stickers and prizes for meeting b. an attempt to minimize pain
goals related to intake of fluid. All this effort on the part of c. exhaustion and lack of energy
the nurse to get the child to drink more fluid is because d. the chest muscles are strained
the fluid is needed to:. Single choice.
a. prevent diarrhea Correct answers: a deficiency of carbon dioxide
b. loosen secretions
c. maintain electrolyte balance 36.For several days, the nurse has been working with the
d. keep kidneys functioning parents of a baby with cleft lip and cleft palate. The
parents seem to have a lot of fears related to the care of
Correct answers: loosen secretions their child and the child’s future. How could the nurse best
help these parents?
30.The nurse assessing a child with respiratory syncytial a. Talk to them about how their fears are
virus (RSV) bronchiolitis looks up the disorder as a review unnecessary and encourage them to stop being afraid.
and is most likely to find which of the following classic b. Tell the parents about any personal family
symptoms associated with this disorder? a. b. c. d.. Single experiences with cleft lip and or cleft palate or about
choice. experiences with other parents having a baby with similar
a. clubbing of the fingers, poor circulation of problems.
extremities, bluish color c. Show some before and after photographs of
b. barklike cough, thick yellow nasal discharge successful surgical repairs and offer to arrange for them
c. patchy rash on the body, rapid respirations, and to talk with other parents of children with a cleft lip and/or
exhaustion palate.
d. respiratory distress, drooling, fever, agitation, and d. Take over more of the care of the infant to give
lethargy the parents time to adjust to having a baby with
deformities.
Correct answers: respiratory distress, drooling, fever,
agitation, and lethargy Correct answers: c
37.The mother of a newborn with a cleft lip says to the
31.Which of the following therapies is the only specific nurse: “Well, I guess I will have to give up the idea of
therapy for respiratory syncytial virus (RSV) breastfeeding and do bottle feeding.” Which of the
bronchiolitis?. Single choice. following is the nurse’s best response?
a. ribavirin (Virazole) a. “How do you feel about giving up breastfeeding
b. chest percussion and going to bottle feeding?”
c. an oxygen tent b. “The breast will mold to the shape of the baby’s
d. Antibiotics lips, will fill the opening in the lip, and your baby will
probably have no more difficulty breastfeeding than any
Correct answers: ribavirin (Virazole) other baby.”
c. “It will help to not dwell on having to give up
32.The nurse is caring for a child with a diagnosis of breastfeeding and perhaps you can breastfeed your next
bronchiolitis who is being managed at home. Which of the baby.”
following strategies to foster adequate ventilation would d. “Let’s talk about all the advantages of bottle
be best for the nurse to suggest to the family?. Single feeding such as having someone else get up and give the
choice. baby the night bottle so you can catch up on needed rest
(1/1 Point) and sleep.”
a. raising the head of the child’s bed
b. administering oxygen therapy Correct answers: b
c. having the child take a walk twice a day
d. installing an attic fan 38.A mother wants to breastfeed her baby born with a cleft
lip and a cleft palate and asks the nurse if this will be
possible. Which of the following is the nurse’s best b. clean the suture line with cotton-tipped
response? applicators dipped in diluted hydrogen peroxide
a. “It will be easier on you and the baby will get c. gently clean with cotton balls saturated with
better nutrition if you bottle feed.” sterile normal saline solution or sterile water
b. “I wish you could breastfeed, but in this case it is d. use an Asepto syringe or a bulb syringe to flush
going to be impossible.” water thoroughly over the surgical site.
c. “It may be possible to breastfeed your baby. I will
help you. If it does not work out, you can use a breast Correct answers: d
pump and feed breast milk using a bottle and special
nipples.” 45.In assessing a newborn, the nurse notices a large
d. “It is done in rare cases where a mother is amount of fine frothy bubbles of mucus in the mouth. Even
determined to breastfeed and willing to keep offering the when the nurse suctions the bubbles, they soon return.
breast for several days even when the baby is not nursing The baby has a rattling sound to the respirations and has
well.” a choking episode and becomes a little cyanotic. Which of
the following conditions will the nurse suspect?
Correct answers: c a. cleft lip
b. cleft palate
39.When the caregiver uses standard nipples and bottles c. esophageal atresia
for a baby with cleft lip and cleft palate, the nurse will d. Intussusception.
teach the ESSR method. This method involves:
a. enter, swish, swallow, and recover Correct answers: c
b. engage, start, swallow, and restart
c. entertain, stimulate, swallow, and retry 46.The nurse is with a newborn who is having
d. enlarge, stimulate, swallow, and rest. radiographic studies to determine if the newborn has
esophageal atresia and if the newborn also has
Correct answers: d tracheoesophageal fistula. When the radiopaque
nasogastric tube is passed through the nose to the
40.When infants with a cleft lip and/or palate are nursing stomach, it stops at 10 cm and the radiographic studies
from a bottle, they will signal a need for a rest before they show air in the stomach. The nurse is aware that these
choke or gag. Which of the following is a signal from the finding indicate which of the following conditions?
infant to take a break? a. normal esophagus and stomach
a. elevating eyebrows and wrinkling the forehead b. abnormal esophagus and normal stomach
b. a slight lifting of the ears c. esophageal atresia without tracheosesophageal
c. puckering the lips tightly and blinking the eyes fistula
d. clenching the fists and a jerking of the arms. d. esophageal atresia with tracheoesophageal
fistula.
Correct answers: a
Correct answers: d
41.When a caregiver has tried standard nipples, preemie 47.Prior to the surgical repair of an esophageal atresia
nipples, and all kinds of special nipples without much with tracheoesophageal fistula, the nursing interventions
success, the best plan is to teach feeding with which of are mainly focused on which of the following things?
the following types of equipment? a. getting as much weight gain as possible
a. spoon b. family education regarding care
b. “sippy” cup c. preventing aspiration pneumonia
c. Asepto syringe with a rubber tip d. discharge planning.
d. special straw for drawing fluid up.
Correct answers: c
Correct answers: c
48.The nurse is caring for a baby who has just had a
42.Which of the following is the first question the nurse surgical repair of an esophageal atresia with
asks on accepting her assignment to care for a mother tracheoesophageal fistula. It is most important for the
and a baby who has a cleft lip and a cleft palate? nurse to position the gastrostomy tube in which of the
a. “Is my care affected by the child’s appearance?” following ways?
b. “How does the mother feel about this baby?” a. pinned to the bed sheet
c. “Is the father going to help with this baby’s care?” b. elevated
d. “How bad are the cleft lip and the cleft palate?”. c. even with the bed
d. Lowered.
Correct answers: a
Correct answers: b
43.In the period immediately after a baby has had surgery
for cleft lip repair, the nurse will do which of the following 49.Which of the following conditions is the most frequent
things? cause of intestinal obstruction in infants and young
a. Place the baby in a prone position lying flat. children?
b. Remove the elbow restraints. a. cancer
c. Hold the pain medication. b. Hirschsprung’s disease
d. Apply a Logan bow or a butterfly adhesive. c. benign tumors
d. Intussusception.
Correct answers: d
Correct answers: d
44.The nurse will teach the family of an infant who has
had a cleft lip repair how to clean the suture line after 50.A newborn has been diagnosed as having
feeding. The nurse will instruct the family to: intussusception. The mother is worried that she caused
a. use a damp, soft washcloth to gently wash the this by something she did or did not do during pregnancy.
suture line The nurse will tell the mother that in most cases the cause
of intussusception is due to:
a. unknown causes 57.Which of the following is the most current treatment for
b. polyps Hirschsprung’s disease?
c. Viruses a. a two-stage surgery using first a temporary
d. drug use in pregnancy. colostomy to provide bowel rest and secondly a pull-
through procedure
Correct answers: a b. a one-stage pull-through without a temporary
colostomy
51.Which of the following statements best describes c. the laparoscopic-assisted pull-through procedure
intussusception? with anal entry thus eliminating major abdominal surgery
a. a hernia of the small intestine into the abdominal d. a laser surgery that is done on an outpatient basis
cavity and requires no incision at all.
b. an outpouching of the bowel anywhere along the
entire bowel Correct answers: c
c. a condition in which one segment of the bowel
telescopes into the lumen of an adjacent segment 58.The nurse caring for the child who is going to have
d. a segment of the bowel is not innervated to any surgery related to Hirschsprung’s disease knows that
extent and it becomes inactive and for all purposes is assessment of the infant’s fluid and electrolyte status is
dead. necessary because of which of the following reasons?
a. There will be extensive bowel cleansing with
Correct answers: c repeated saline enemas.
b. The child’s extreme constipation will throw the
52.The treatment of choice and the safest treatment for electrolytes off.
intussusception is which of the following treatments? c. The child will be NPO for a very long period of
a. a three-stage surgery involving a temporary time.
colostomy opening d. Vomiting and diarrhea are not unusual in these
b. hydrostatic reduction with barium cases.
c. hydrostatic reduction with an air or a water-
soluble contrast agent Correct answers: a
d. external massage and rotation.
59.Which of the following anorectal malformations do not
Correct answers: c require surgery?
a. imperforate anus
53.Discharge instructions to the parents of an infant who b. anal stenosis
was treated for intussusception most need to include c. rectal atresia
information on observing for signs of: d. anal agenesis.
a. intestinal obstruction and reoccurrence
b. vitamin and mineral deficiencies Correct answers: b
c. adjustment to being at home
d. abnormal vital signs. 60.On assessment of a newborn, the nurse finds
meconium in the urine. The nurse realizes that this is
Correct answers: a indicative of which of the following conditions?
a. a bladder that is wrapped around the intestine
54.The nurse assessing newborn babies and infants b. imperforate anus
during their hospital stay after birth will notice which of the c. anal agenesis
following symptoms as a primary manifestation of d. a fistula between the bowel and the urinary tract.
Hirschsprung’s disease?
a. failure to pass meconium during the first 24 to 48 Correct answers: d
hours after birth
b. high grade fever 61.Which of the following interventions is most important
c. the skin turns yellow then brown over the first 48 when the baby has surgery to correct a low anorectal
hours of life malformation?
d. a fine rash over the trunk. a. meticulous skin care
b. push fluids
Correct answers: a c. keep anal area covered
d. high fiber diet.
55.Which of the following is the major cause of death in
Hirschsprung’s disease? Correct answers: a
a. widespread infection in the body
b. failure to take in enough nourishment 62.The nurse is providing some teaching to the parents of
c. bacterial endocarditis and heart failure a two-year-old child who has had surgery to correct an
d. enterocolitis, sepsis, or bowel perforation. anorectal malformation. The nurse will advise the parents
or caregivers that toilet training will:
Correct answers: d a. likely be delayed
b. require help from an expert
56.The nurse assessing an older infant or child will c. be no different than for other toddlers
suspect Hirschsprung’s disease when the child has a d. be easier.
history of which of the following symptoms?
a. chronic constipation Correct answers: a
b. blood in the stools
c. chronic vomiting 63.Which of the following conditions is the most common
d. clay-colored stools. esophageal disorder found in infants and the esophageal
disorder most frequently referred to a pediatric
Correct answers: a gastroenterologist?
a. hypertrophic pyloric stenosis
b. esophageal stricture
c. gastroesophageal reflux (GER) 68.The nurse is assessing a child admitted to the hospital
d. intestinal malrotation. for abdominal pain. Which of the following findings by the
nurse would be typical of those seen in appendicitis?
Correct answers: c a. anorexia, nausea, and vomiting preceded the
pain according to history provided by the caregivers
64.A nurse is working with a caregiver of an infant with b. a subnormal temperature for the last two days
gastroesophageal reflux. The infant has poor weight gain, c. pain that was vague and somewhat localized to
cries when awake, and is vomiting or regurgitating with the periumbilical area and gradually migrated to the right
feedings. Given these symptoms, which of the following lower quadrant
interventions would be best for the nurse to recommend d. night time chills for one week.
to the caregiver?
a. Give small, frequent feedings. Correct answers: c
b. Feed the infant with the infant’s body in a straight
position. 69.The nurse is working with a parent whose child had a
c. Hold the baby in an upright position for an hour perforation of the appendix. The nurse shares with the
after feedings. parents why appendicitis frequently progresses to
d. Thicken the formula with a little rice cereal. perforation in children by saying:
a. “The appendix is usually near the perforation
Correct answers: d state by the time children will say anything for fear they
have done something wrong.”
65.The parents of an infant with gastroesophageal reflux b. “Young children have a thinner appendiceal wall
(GER) ask the nurse about the advisability of the side- than adults so they progress to perforation much quicker
lying or supine position for prevention of sudden infant than adults.”
death syndrome for their infant. The nurse’s best c. “Children tolerate pain much better than adults so
response would be: they are in a lot of pain by the time they tell caregivers.”
a. “Yes, prevention of SIDS is extremely important d. “The appendix in children is smaller and therefore
and side-lying or supine is currently recommended.” much easier to rupture”.
b. “Currently prone or head elevated prone is
recommended for GER and with elimination of puffy Correct answers: b
bedding from the crib you can decrease chances of
SIDS.” 70.The nurse is planning interventions for a child who has
c. “Side-lying or supine will work just fine as long as inflammatory bowel disease (IBD) and a nursing
you elevate the head of the bed so that the infant is diagnosis of nutrition less than body requirements. Which
sleeping nearly sitting up.” of the following interventions will be most helpful in
d. “SIDS is something you cannot really protect resolving this nursing problem?
against for certain so get some baby monitors and let your a. special IBD diet (diet that has been proven
baby seek whatever position is most comfortable”. effective for treating IBD)
b. diet as tolerated with lactose hydrolyzed milk
Correct answers: b instead of milk products, and omission of highly seasoned
foods, and reduction of fiber
66.The nurse on the pediatric unit is assigned to an infant c. salt free diet high in potassium, vitamins, and
with gastroesophageal reflux. The infant has had minerals
episodes of pneumonia and is not gaining weight. The d. two large meals a day instead of several mini
infant has not responded well to six weeks of medical meals and snacks.
management and is being considered for surgery. At the
beginning of the shift, after receiving report on this infant, Correct answers: b
the nurse will first:
a. read the infant’s chart or computer record NCM 110 Nursing Informatics Lec
b. weigh the infant
c. assess respiratory status
d. check the medication administration record and
Midterms
set up medications.
1.SITUATION: Demy a 3 year old patient brought in TPH
Correct answers: c
emergency room with cystic fibrosis. Lynn was assigned
in pediatric ward. 1. A sputum specimen is collected from
67.The nurse is working with the parents of a child who
Demy for culture purposes, the primary reason being to
has chronic constipation. The nurse will teach the parents
learn. Single choice.
how to establish a regular pattern of defecation. The nurse
a. whether the child's cystic fibrosis is under control.
will evaluate that the parents understood the teaching
b. what organism is responsible for the child's
when they report back to the nurse with which of the
disease.
following statements?
c. if there is blood present in the child's lungs
a. “Our child has not been given any dessert unless
tracheobronchial tree
he has had a bowel movement the previous day.”
d. which portion of the child's lungs is affected.
b. “We take turns making sure he sits on the toilet
until he has a bowel movement even if it is an hour.”
Correct answers: what organism is responsible for the
c. “He gets to flush the toilet himself and wave bye-
child's disease.
bye to the stools in the toilet and we pat his head and say
good boy.”
2. Which of the following statements best defines cystic
d. “He sits on the toilet after a meal for 5–10 minutes
fibrosis? Single choice.
and usually has a bowel movement and then we give him
a. an acquired lung condition that occurs at birth due
a star for the prize chart”.
to oxygen deficiency 1
b. a congenital anomaly occurring during fetal
Correct answers: d
development as a result of dietary deficiencies of the
mother
c. an autosomal recessive disorder that mainly celiac disease. Matt is admitted to the pediatric unit with a
affects the exocrine glands and does multiple system diagnosis of celiac disease. John who was assigned to
damage this child realizes that the typical stool of a child with this
d. a respiratory condition affecting people of disease will be:. Single choice.
Mediterranean descent, involving mainly exchange at a a. black and tarry
microscopic level in the lungs b. normal colored and pencil thin or ribbonlike in
appearance
Correct answers: an autosomal recessive disorder that c. bulky, putty colored, foul smelling, greasy, and
mainly affects the exocrine glands and does multiple likely to float
system damage d. white and loose

3. Which of the following represents the life expectancy of Correct answers: bulky, putty colored, foul smelling,
a child with cystic fibrosis?. Single choice. greasy, and likely to float
a. 21 year old
b. 30 year old 10.The nurse should expect that, because Matt has celiac
c. 12 year old disease, his stools are most likely to be. Single choice.
d. 7 year old a. especially dark in color
b. abnormally small in amount
Correct answers: 30 year old c. unusually hard in consistency
d. particularly offensive in odor
4. Lynn can expect Demy's to have stools that are
typically. Single choice. Correct answers: particularly offensive in odor
a. bulky and foul odor
b. dry and have the odor of ammonia 11.During assessment, a physical finding John is most
c. watery and sweet in odor likely to observe is that Matt has. Single choice.
d. dark in color and have almost no odor a. an enlarged liver
b. a protuberant abdomen
Correct answers: bulky and foul odor c. a tender inguinal lymph nodes
d. edema in the lower extremities
5.During the warm weather, the nurse will assess children
with cystic fibrosis to see if they need which of the Correct answers: a protuberant abdomen
following supplements added to their diet?. Single choice.
a. zinc 12.The family of a child with celiac’s disease is receiving
b. magnesium discharge teaching in regard to diet. The nurse will teach
c. salt the family to modify the diet in which of the following
d. potassium ways?. Single choice.
a. Eliminate wheat, barley, rye, and oats and
Correct answers: salt substitute corn, rice, and millet.
b. Increase bulk including raw vegetables, fruit, and
6.Demy is to have postural (bronchial) drainage. The a variety of cereals and nuts.
times in the day when it is best for Lynn to plan to carry c. Eliminate fats but increase natural fish oils and
out postural drainage is shortly. Single choice. olive oils.
a. after meals d. Eliminate lactose products such as milk, cheese,
b. before meals ice cream, and butter.
c. before rest periods
d. after rest periods Correct answers: Eliminate wheat, barley, rye, and oats
and substitute corn, rice, and millet.
Correct answers: before meals
13.For how long is it likely that Matt will need to remain on
7.In working with children with cystic fibrosis, the nurse a special diet. Single choice.
expects to carry out and provide teaching on which of the a. For the rest of his life.
following treatments?. Single choice. b. Until the disease is well controlled.
a. dietary modification involving a low protein, high c. Until approximately the age of puberty.
fat, low carbohydrate diet d. For the period required to desensitize him to
b. range of motion and a highly regimented exercise offending foods.
program
c. preparation for renal dialysis and strict intake and Correct answers: For the rest of his life.
output
d. eto answer 14.Matt's wears reusable diapers at night. His mother
asks what safety precautions she should take his wet,
Correct answers: soiled diapers. Which of the following statements should
guide the nurse when she responds to Matt's mother? ..
8.Lynn should understand prior to evaluating the effects Single choice.
of postural drainage that the primary reason for Demy's a. The diaper should be boiled after they are
having this type of therapy is ton help to. Single choice. washed.
a. dilate the bronchioles. b. The diapers should be soaked in an antiseptic
b. promote better use of diaphragm. solution before they are washed
c. improve circulation in the chest cavity. c. It would be best to use disposable diapers until
d. clean the lungs of mucopurulent material. the diarrhea is under control.
d. There are no special precautions necessary in
Correct answers: clean the lungs of mucopurulent the care of this infant's diaper.
material.
Correct answers: There are no special precautions
9.SITUATION: Mr. John a new staff nurse was assigned necessary in the care of this infant's diaper.
in the pediatric ward that is caring for 18 months old with
15.Which of the following makes children prone to 19.Which of the following positions does the nurse use for
tonsillitis and pharyngitis?. Single choice. the child who has just returned to the room from surgery?.
a. not brushing their teeth and flossing after meals Single choice.
and at bedtime a. elevated head and legs
b. frequent upper respiratory tract infections and b. on the abdomen or side
being around other children who may be infected c. on the back and flat
c. eating large amounts of sugar and drinking high d. on the back or sitting up
sugar content drinks that encourage growth of pathogens
d. not bathing properly or frequently Correct answers: on the abdomen or side

Correct answers: frequent upper respiratory tract 20.A child who is in the first postoperative day after
infections and being around other children who may be tonsillectomy starts to cough and wants to blow his nose.
infected Which of the following actions by the nurse would be
best?. Single choice.
16.The school nurse encourages the parents of a student a. Offer the child some tissue and give instructions
who has a severe sore throat to take the child to the on its disposal.
pediatrician. The nurse is especially concerned about the b. Remind the child not to cough or blow his nose.
possibility of a streptococcal tonsillitis mainly because of c. Tell him he can cough but not to blow his nose.
which of the following?. Single choice. d. Inform the child that he can blow his nose but not
a. If streptococcal tonsillitis is not treated adequately cough.
it may lead to scarlet fever, otitis media, or more serious
complications. Correct answers: Tell him he can cough but not to blow
b. The student will miss a lot of school and fall his nose.
behind in schoolwork and homework assignments and
may even possibly fail a grade. 21.The nurse is assigned to care for a child who had a
c. Other children may also get sore throats if this tonsillectomy yesterday. The child complains of a sore
student is not aggressively treated. throat and wants something cool. Which of the following
d. If untreated, streptococcal tonsillitis may cause cool drinks or food would be acceptable to offer the child?.
anemia, weight loss, and lethargy. Single choice.
Correct answers: If streptococcal tonsillitis is not treated a. lime sherbet
adequately it may lead to scarlet fever, otitis media, or b. chilled tomato juice
more serious complications. c. cold cola
d. cherry Jell-O
17.The mother of a two-and-a-half-year old child who has
had tonsillitis asks about the possibility of a tonsillectomy Correct answers: lime sherbet
for the child. The nurse informs the mother that
tonsillectomies are usually performed when the child is 22.Which of the following signs or symptoms is the
over three years old. The mother wants to know why earliest manifestation of bleeding in a child who has had
children younger than three have to wait for a a tonsillectomy?. Single choice.
tonsillectomy. Which of the following is the nurse’s best a. frequent swallowing
answer?. Single choice. b. spitting up blood
a. “Surgery is too difficult on a younger child c. coffee ground emesis
because their mouths are so small, making it difficult to d. complaint of thirst
get to the tonsils.”
b. “It is too scary for a younger child and they don’t Correct answers:
understand what is happening.”
c. “Younger children are more prone to excessive 23.Which of the following outcomes is the most important
blood loss and/or the tonsils growing back.” for a child who has had a tonsillectomy?. Single choice.
d. “Separation from the parents or caregivers is a. no postoperative bleeding
much more difficult in the child under three.” b. bonding with the nursing staff
c. absence of pain
Correct answers: “Younger children are more prone to d. drinking sufficient water
excessive blood loss and/or the tonsils growing back.”
Correct answers: no postoperative bleeding
18.The nurse explains to parents why an adenoidectomy
is sometimes indicated with the tonsillectomy. Which of 24.The nurse assessing a child with a respiratory problem
the following is the nurse’s best explanation?. Single finds stridor, which is best described by which of the
choice. following?. Single choice.
a. An adenoidectomy is justified when a child’s a. excursion of the chest wall beyond that normally
enlarged adenoids block the flow of air through the nasal seen in the healthy child
passages.” b. a high-pitched sound produced by an obstruction
b. “Some surgeons prefer to take out the adenoids of the trachea or larynx
at the same time as the tonsillectomy because it saves c. Message for respondents who select this answer
the child from a second surgery.” d. a triad of respiratory symptoms
c. Without the tonsils to screen out foreign proteins, e. substernal retractions and nasal flaring
the adenoids are more easily infected.”
d. “It is hard to take out the tonsils without removing Correct answers: a high-pitched sound produced by an
the adenoids because they often are adhered to each obstruction of the trachea or larynx
other.”
25.Examination of the throat of a child with acute epiglottis
Correct answers: An adenoidectomy is justified when a by depressing the tongue is. Single choice.
child’s enlarged adenoids block the flow of air through the a. a good idea because there is a characteristic
nasal passages.” patchy redness that aids in diagnosis
b. contraindicated due to the possibility of causing
complete airway obstruction
c. impossible because the tongue is too swollen and
tender to allow it 31.Which of the following therapies is the only specific
d. to be done carefully because of the possibility of therapy for respiratory syncytial virus (RSV)
causing additional pain bronchiolitis?. Single choice.
a. ribavirin (Virazole)
Correct answers: contraindicated due to the possibility b. chest percussion
of causing complete airway obstruction c. an oxygen tent
d. Antibiotics
26.The nurse is assessing a child with croup. Which of the
following signs and/or symptoms would the nurse Correct answers: ribavirin (Virazole)
associate with croup and most expect to find?. Single
choice. 32.The nurse is caring for a child with a diagnosis of
a. barklike cough, hoarseness, inspiratory stridor bronchiolitis who is being managed at home. Which of the
b. expiratory stridor, irregular breathing, and weak following strategies to foster adequate ventilation would
cough be best for the nurse to suggest to the family?. Single
c. Cheyne-Stokes respirations and substernal choice.
retraction (1/1 Point)
d. almost continual cough and complete laryngitis a. raising the head of the child’s bed
b. administering oxygen therapy
Correct answers: barklike cough, hoarseness, c. having the child take a walk twice a day
inspiratory stridor d. installing an attic fan

27.Which of the following statements best describes Correct answers: raising the head of the child’s bed
croup?. Single choice.
a. a common childhood disease 33.Pneumonia in children appears:. Single choice.
b. an autoimmune-initiated infection a. in a more primitive form which is easier to cure
c. a life-threatening bacterial infection than that found in adults
d. a respiratory condition similar to a cold b. as a secondary disease
c. as a tertiary disease
Correct answers: an autoimmune-initiated infection d. as a primary disease or a complication of another
disease
28.The nurse is assessing the skin moisture of a child with
laryngotracheobronchitis. Which of the following explains Correct answers: as a primary disease or a complication
why the nurse assessed the skin moisture in this case?. of another disease
Single choice.
a. diaphoresis is associated with increased 34.Which of the following types of pneumonia would be
respiratory effort treated with antibiotics?. Single choice.
b. skin moisture is a relatively good measure of the a. viral
degree of infection b. Bacterial
c. increased skin moisture will signal danger of c. Aspiration
ketoacidosis d. adenoviral
d. to get some measure of how dehydrated the child
might become Correct answers: Bacterial

Correct answers: diaphoresis is associated with 35.The nurse is assessing a child with a diagnosis of
increased respiratory effort pneumonia. The child is taking shallow respirations.
Which of the following is the most likely explanation?.
29.The nurse working with a child with Single choice.
laryngotracheobronchitis is offering the child tiny medicine a. a deficiency of carbon dioxide
cups of fluid and giving stickers and prizes for meeting b. an attempt to minimize pain
goals related to intake of fluid. All this effort on the part of c. exhaustion and lack of energy
the nurse to get the child to drink more fluid is because d. the chest muscles are strained
the fluid is needed to:. Single choice.
a. prevent diarrhea Correct answers: a deficiency of carbon dioxide
b. loosen secretions
c. maintain electrolyte balance 36.For several days, the nurse has been working with the
d. keep kidneys functioning parents of a baby with cleft lip and cleft palate. The
parents seem to have a lot of fears related to the care of
Correct answers: loosen secretions their child and the child’s future. How could the nurse best
help these parents?
30.The nurse assessing a child with respiratory syncytial a. Talk to them about how their fears are
virus (RSV) bronchiolitis looks up the disorder as a review unnecessary and encourage them to stop being afraid.
and is most likely to find which of the following classic b. Tell the parents about any personal family
symptoms associated with this disorder? a. b. c. d.. Single experiences with cleft lip and or cleft palate or about
choice. experiences with other parents having a baby with similar
a. clubbing of the fingers, poor circulation of problems.
extremities, bluish color c. Show some before and after photographs of
b. barklike cough, thick yellow nasal discharge successful surgical repairs and offer to arrange for them
c. patchy rash on the body, rapid respirations, and to talk with other parents of children with a cleft lip and/or
exhaustion palate.
d. respiratory distress, drooling, fever, agitation, and d. Take over more of the care of the infant to give
lethargy the parents time to adjust to having a baby with
deformities.
Correct answers: respiratory distress, drooling, fever,
agitation, and lethargy Correct answers: c
37.The mother of a newborn with a cleft lip says to the b. “How does the mother feel about this baby?”
nurse: “Well, I guess I will have to give up the idea of c. “Is the father going to help with this baby’s care?”
breastfeeding and do bottle feeding.” Which of the d. “How bad are the cleft lip and the cleft palate?”.
following is the nurse’s best response?
a. “How do you feel about giving up breastfeeding Correct answers: a
and going to bottle feeding?”
b. “The breast will mold to the shape of the baby’s 43.In the period immediately after a baby has had surgery
lips, will fill the opening in the lip, and your baby will for cleft lip repair, the nurse will do which of the following
probably have no more difficulty breastfeeding than any things?
other baby.” a. Place the baby in a prone position lying flat.
c. “It will help to not dwell on having to give up b. Remove the elbow restraints.
breastfeeding and perhaps you can breastfeed your next c. Hold the pain medication.
baby.” d. Apply a Logan bow or a butterfly adhesive.
d. “Let’s talk about all the advantages of bottle
feeding such as having someone else get up and give the Correct answers: d
baby the night bottle so you can catch up on needed rest
and sleep.” 44.The nurse will teach the family of an infant who has
had a cleft lip repair how to clean the suture line after
Correct answers: b feeding. The nurse will instruct the family to:
a. use a damp, soft washcloth to gently wash the
38.A mother wants to breastfeed her baby born with a cleft suture line
lip and a cleft palate and asks the nurse if this will be b. clean the suture line with cotton-tipped
possible. Which of the following is the nurse’s best applicators dipped in diluted hydrogen peroxide
response? c. gently clean with cotton balls saturated with
a. “It will be easier on you and the baby will get sterile normal saline solution or sterile water
better nutrition if you bottle feed.” d. use an Asepto syringe or a bulb syringe to flush
b. “I wish you could breastfeed, but in this case it is water thoroughly over the surgical site.
going to be impossible.”
c. “It may be possible to breastfeed your baby. I will Correct answers: d
help you. If it does not work out, you can use a breast
pump and feed breast milk using a bottle and special 45.In assessing a newborn, the nurse notices a large
nipples.” amount of fine frothy bubbles of mucus in the mouth. Even
d. “It is done in rare cases where a mother is when the nurse suctions the bubbles, they soon return.
determined to breastfeed and willing to keep offering the The baby has a rattling sound to the respirations and has
breast for several days even when the baby is not nursing a choking episode and becomes a little cyanotic. Which of
well.” the following conditions will the nurse suspect?
a. cleft lip
Correct answers: c b. cleft palate
c. esophageal atresia
39.When the caregiver uses standard nipples and bottles d. Intussusception.
for a baby with cleft lip and cleft palate, the nurse will
teach the ESSR method. This method involves: Correct answers: c
a. enter, swish, swallow, and recover
b. engage, start, swallow, and restart 46.The nurse is with a newborn who is having
c. entertain, stimulate, swallow, and retry radiographic studies to determine if the newborn has
d. enlarge, stimulate, swallow, and rest. esophageal atresia and if the newborn also has
tracheoesophageal fistula. When the radiopaque
Correct answers: d nasogastric tube is passed through the nose to the
stomach, it stops at 10 cm and the radiographic studies
40.When infants with a cleft lip and/or palate are nursing show air in the stomach. The nurse is aware that these
from a bottle, they will signal a need for a rest before they finding indicate which of the following conditions?
choke or gag. Which of the following is a signal from the a. normal esophagus and stomach
infant to take a break? b. abnormal esophagus and normal stomach
a. elevating eyebrows and wrinkling the forehead c. esophageal atresia without tracheosesophageal
b. a slight lifting of the ears fistula
c. puckering the lips tightly and blinking the eyes d. esophageal atresia with tracheoesophageal
d. clenching the fists and a jerking of the arms. fistula.

Correct answers: a Correct answers: d


47.Prior to the surgical repair of an esophageal atresia
41.When a caregiver has tried standard nipples, preemie with tracheoesophageal fistula, the nursing interventions
nipples, and all kinds of special nipples without much are mainly focused on which of the following things?
success, the best plan is to teach feeding with which of a. getting as much weight gain as possible
the following types of equipment? b. family education regarding care
a. spoon c. preventing aspiration pneumonia
b. “sippy” cup d. discharge planning.
c. Asepto syringe with a rubber tip
d. special straw for drawing fluid up. Correct answers: c

Correct answers: c 48.The nurse is caring for a baby who has just had a
surgical repair of an esophageal atresia with
42.Which of the following is the first question the nurse tracheoesophageal fistula. It is most important for the
asks on accepting her assignment to care for a mother nurse to position the gastrostomy tube in which of the
and a baby who has a cleft lip and a cleft palate? following ways?
a. “Is my care affected by the child’s appearance?” a. pinned to the bed sheet
b. elevated
c. even with the bed 55.Which of the following is the major cause of death in
d. Lowered. Hirschsprung’s disease?
a. widespread infection in the body
Correct answers: b b. failure to take in enough nourishment
c. bacterial endocarditis and heart failure
49.Which of the following conditions is the most frequent d. enterocolitis, sepsis, or bowel perforation.
cause of intestinal obstruction in infants and young
children? Correct answers: d
a. cancer
b. Hirschsprung’s disease 56.The nurse assessing an older infant or child will
c. benign tumors suspect Hirschsprung’s disease when the child has a
d. Intussusception. history of which of the following symptoms?
a. chronic constipation
Correct answers: d b. blood in the stools
c. chronic vomiting
50.A newborn has been diagnosed as having d. clay-colored stools.
intussusception. The mother is worried that she caused
this by something she did or did not do during pregnancy. Correct answers: a
The nurse will tell the mother that in most cases the cause
of intussusception is due to: 57.Which of the following is the most current treatment for
a. unknown causes Hirschsprung’s disease?
b. polyps a. a two-stage surgery using first a temporary
c. Viruses colostomy to provide bowel rest and secondly a pull-
d. drug use in pregnancy. through procedure
b. a one-stage pull-through without a temporary
Correct answers: a colostomy
c. the laparoscopic-assisted pull-through procedure
51.Which of the following statements best describes with anal entry thus eliminating major abdominal surgery
intussusception? d. a laser surgery that is done on an outpatient basis
a. a hernia of the small intestine into the abdominal and requires no incision at all.
cavity
b. an outpouching of the bowel anywhere along the Correct answers: c
entire bowel
c. a condition in which one segment of the bowel 58.The nurse caring for the child who is going to have
telescopes into the lumen of an adjacent segment surgery related to Hirschsprung’s disease knows that
d. a segment of the bowel is not innervated to any assessment of the infant’s fluid and electrolyte status is
extent and it becomes inactive and for all purposes is necessary because of which of the following reasons?
dead. a. There will be extensive bowel cleansing with
repeated saline enemas.
Correct answers: c b. The child’s extreme constipation will throw the
electrolytes off.
52.The treatment of choice and the safest treatment for c. The child will be NPO for a very long period of
intussusception is which of the following treatments? time.
a. a three-stage surgery involving a temporary d. Vomiting and diarrhea are not unusual in these
colostomy opening cases.
b. hydrostatic reduction with barium
c. hydrostatic reduction with an air or a water- Correct answers: a
soluble contrast agent
d. external massage and rotation. 59.Which of the following anorectal malformations do not
require surgery?
Correct answers: c a. imperforate anus
b. anal stenosis
53.Discharge instructions to the parents of an infant who c. rectal atresia
was treated for intussusception most need to include d. anal agenesis.
information on observing for signs of:
a. intestinal obstruction and reoccurrence Correct answers: b
b. vitamin and mineral deficiencies
c. adjustment to being at home 60.On assessment of a newborn, the nurse finds
d. abnormal vital signs. meconium in the urine. The nurse realizes that this is
indicative of which of the following conditions?
Correct answers: a a. a bladder that is wrapped around the intestine
b. imperforate anus
54.The nurse assessing newborn babies and infants c. anal agenesis
during their hospital stay after birth will notice which of the d. a fistula between the bowel and the urinary tract.
following symptoms as a primary manifestation of
Hirschsprung’s disease? Correct answers: d
a. failure to pass meconium during the first 24 to 48
hours after birth 61.Which of the following interventions is most important
b. high grade fever when the baby has surgery to correct a low anorectal
c. the skin turns yellow then brown over the first 48 malformation?
hours of life a. meticulous skin care
d. a fine rash over the trunk. b. push fluids
c. keep anal area covered
Correct answers: a d. high fiber diet.
67.The nurse is working with the parents of a child who
Correct answers: a has chronic constipation. The nurse will teach the parents
how to establish a regular pattern of defecation. The nurse
62.The nurse is providing some teaching to the parents of will evaluate that the parents understood the teaching
a two-year-old child who has had surgery to correct an when they report back to the nurse with which of the
anorectal malformation. The nurse will advise the parents following statements?
or caregivers that toilet training will: a. “Our child has not been given any dessert unless
a. likely be delayed he has had a bowel movement the previous day.”
b. require help from an expert b. “We take turns making sure he sits on the toilet
c. be no different than for other toddlers until he has a bowel movement even if it is an hour.”
d. be easier. c. “He gets to flush the toilet himself and wave bye-
bye to the stools in the toilet and we pat his head and say
Correct answers: a good boy.”
d. “He sits on the toilet after a meal for 5–10 minutes
63.Which of the following conditions is the most common and usually has a bowel movement and then we give him
esophageal disorder found in infants and the esophageal a star for the prize chart”.
disorder most frequently referred to a pediatric
gastroenterologist? Correct answers: d
a. hypertrophic pyloric stenosis
b. esophageal stricture 68.The nurse is assessing a child admitted to the hospital
c. gastroesophageal reflux (GER) for abdominal pain. Which of the following findings by the
d. intestinal malrotation. nurse would be typical of those seen in appendicitis?
a. anorexia, nausea, and vomiting preceded the
Correct answers: c pain according to history provided by the caregivers
b. a subnormal temperature for the last two days
64.A nurse is working with a caregiver of an infant with c. pain that was vague and somewhat localized to
gastroesophageal reflux. The infant has poor weight gain, the periumbilical area and gradually migrated to the right
cries when awake, and is vomiting or regurgitating with lower quadrant
feedings. Given these symptoms, which of the following d. night time chills for one week.
interventions would be best for the nurse to recommend
to the caregiver? Correct answers: c
a. Give small, frequent feedings.
b. Feed the infant with the infant’s body in a straight 69.The nurse is working with a parent whose child had a
position. perforation of the appendix. The nurse shares with the
c. Hold the baby in an upright position for an hour parents why appendicitis frequently progresses to
after feedings. perforation in children by saying:
d. Thicken the formula with a little rice cereal. a. “The appendix is usually near the perforation
state by the time children will say anything for fear they
Correct answers: d have done something wrong.”
b. “Young children have a thinner appendiceal wall
65.The parents of an infant with gastroesophageal reflux than adults so they progress to perforation much quicker
(GER) ask the nurse about the advisability of the side- than adults.”
lying or supine position for prevention of sudden infant c. “Children tolerate pain much better than adults so
death syndrome for their infant. The nurse’s best they are in a lot of pain by the time they tell caregivers.”
response would be: d. “The appendix in children is smaller and therefore
a. “Yes, prevention of SIDS is extremely important much easier to rupture”.
and side-lying or supine is currently recommended.”
b. “Currently prone or head elevated prone is Correct answers: b
recommended for GER and with elimination of puffy
bedding from the crib you can decrease chances of 70.The nurse is planning interventions for a child who has
SIDS.” inflammatory bowel disease (IBD) and a nursing
c. “Side-lying or supine will work just fine as long as diagnosis of nutrition less than body requirements. Which
you elevate the head of the bed so that the infant is of the following interventions will be most helpful in
sleeping nearly sitting up.” resolving this nursing problem?
d. “SIDS is something you cannot really protect a. special IBD diet (diet that has been proven
against for certain so get some baby monitors and let your effective for treating IBD)
baby seek whatever position is most comfortable”. b. diet as tolerated with lactose hydrolyzed milk
instead of milk products, and omission of highly seasoned
Correct answers: b foods, and reduction of fiber
c. salt free diet high in potassium, vitamins, and
66.The nurse on the pediatric unit is assigned to an infant minerals
with gastroesophageal reflux. The infant has had d. two large meals a day instead of several mini
episodes of pneumonia and is not gaining weight. The meals and snacks.
infant has not responded well to six weeks of medical
management and is being considered for surgery. At the Correct answers: b
beginning of the shift, after receiving report on this infant,
the nurse will first:
a. read the infant’s chart or computer record
b. weigh the infant
c. assess respiratory status
d. check the medication administration record and
set up medications.
NCM 113 CHN2 Lab (Myrtle and Jomer)
Correct answers: c
MIDTERMS
1. Which of the following is defined as a process unimmunized, also, Mrs. Cruz verbalized presence of
whereby existing and potential health conditions mass in her left breast, and found out that their 18-year-
or problems of the family are determined? old daughter Ofelia is suffering from an eating disorder.
a. Second Level Assessment Nurse identified which among the problems of the Cruz
b. Third Level of Assessment family is/are classified as Stress Points/Foreseeable
c. Fourth Level of Assessment Crisis?
d. First Level Assessment . Mr. Cruz lost his Job, and still Jobless for two
• Correct answers: First Level months
Assessment a. 18-year-old daughter with eating disorder
2. You have been assigned as a community health b. B and C
nurse in barangay San Agustin, Victoria Tarlac c. A and D
providing primary health care services, you know d. Presence of mass in the left breast of Mrs. Cruz
that one of the following characterized Primary e. Unimmunized one and eight months old child
Health Care? • Correct answers: Mr. Cruz lost
1. Essential health care accessible and his Job, and still Jobless for
acceptable to individuals and families in the two months
community. 7. In implementing family centered care, the nurse
2. The cost of care is affordable to the community should perform which of the following?
and country. . Assists family members to assume dependent
3. The setting is in tertiary hospitals. roles
4. Require health personnel and specialized a. Provides his or her own beliefs on how to solve
training. problems
a. 3&4 b. Offers information about necessary self-care
b. 1, 2, 3 & 4 abilities
c. 1&2 c. Works with clients to help them accept blame for
d. 3&4 their interactions
• Correct answers: 1 & 2 • Correct answers: Offers
3. A client with severe arthritis is returning home information about necessary
after having had a colostomy. The client is unable to self-care abilities
perform the colostomy care. The nurse should first 8. 8.Situation: Nurse Stephen is conducting home
perform which of the following? visit to the Cruz family, while providing care to the family,
a. Inform the client that an alternate way of he found out that Mr. Cruz lost his job two months ago,
managing the colostomy must be learned and still jobless, he discovered that the youngest child
b. Investigate whether or not someone else in the who is already one year and eight months old is still
family or neighborhood will be able to do the colostomy unimmunized, also, Mrs. Cruz verbalized presence of
care mass in her left breast, and found out that their 18-year-
c. Refer the client to a colostomy self-help support old daughter Ofelia is suffering from an eating disorder. In
group relation to the situation given above, Nurse Stephen
d. Arrange for a private duty nurse to take care of identified the following health problems of the Cruz family,
the client along with the identified health problem stated, which of
• Correct answers: Investigate these is classified as a Health threat?
whether or not someone . 18-year-old daughter with eating disorder
else in the family or a. Mr. Cruz lost his Job, and still Jobless for two
neighborhood will be able to months
do the colostomy care b. Unimmunized one and eight months old child
4. Collection of genetic history in relation to health c. Presence of mass in the left breast of Mrs. Cruz
and illness is included in which of the following? d. A and D
. LIFESTYLE e. B and C
a. FAMILY HISTORY • Correct answers:
b. SOCIAL HISTORY Unimmunized one and eight
c. BIOGRAPHICAL months old child
• Correct answers: FAMILY 9. Typology/Category of health problem wherein the
HISTORY clinical or nursing judgment about a client transition form
5. On your Home visit to assess the family, you a specific level of wellness or capability to a higher level?
noticed a 90-year-old patient. During the health history, . Wellness Condition
the patient is unable to respond to questions in a logical a. Health Deficits
manner. As a nurse assessing the patient, your best b. Health Threats
action is to? c. Stress Points/Foreseeable Crisis
. Rephrase the questions slightly, and slowly • Correct answers: Wellness
repeat them in a lower voice. Condition
a. Ask the same questions in a louder and lower 10. This is the phase of the Nursing Process wherein
voice. putting the nursing care plan into action to achieve goals
b. Determine if the patient knows the name of the and outcomes?
current president. . Nursing Diagnosis
c. Ask the family member to answer the questions. a. Implementation
• Correct answers: Rephrase b. Assessment
the questions slightly, and c. Planning
slowly repeat them in a d. Evaluation
lower voice. • Correct answers:
6. Situation: Nurse Stephen is conducting home visit Implementation
to the Cruz family, while providing care to the family, he 11. This is the phase of the Nursing Process wherein
found out that Mr. Cruz lost his job two months ago, and you are Assessing the client’s response to nursing
still jobless, he discovered that the youngest child who is interventions and then comparing the response to the
already one year and eight months old is still goals?
. Implementation a. Death rate
a. Evaluation b. Mortality
b. Nursing Diagnosis c. Longevity
c. Planning • Correct answers: Mortality
d. Assessment 19. This is the phase of the Nursing Process wherein
• Correct answers: Evaluation in this phase the nurse sort, clusters and analyzes data?
12. For the following examples, which assessment . Nursing Diagnosis
views the “family as context?”. a. Assessment
. The client’s ability to understand and manage his b. Planning
own dietary needs c. Implementation
a. The family’s ability to support the client’s dietary d. Evaluation
and recreational needs • Correct answers: Nursing
b. The family’s demands on the client based on his Diagnosis
role performance 20. Intra-personal and inter-personal factors affecting
c. The adjustment of the client and family to the family member are included in the collections of which
changes in diet and exercise data?
• Correct answers: The client’s . FAMILY HISTORY
ability to understand and a. BIOGRAPHICAL
manage his own dietary b. SOCIAL HISTORY
needs c. LIFESTYLE
13. This is the phase of the Nursing Process wherein • Correct answers: SOCIAL
the expected outcomes are designed to resolve or HISTORY
minimize the identified problems of the client? 21. Once all nursing intervention actions have taken
. Nursing Diagnosis place, this phase of nursing process where in the nurse
a. Assessment determine if the goals for patient wellness have been met
b. Evaluation through re-assessment?
c. Planning . Implementation
d. Implementation a. Nursing Diagnosis
• Correct answers: Planning b. Evaluation
14. It defines the nature or type of nursing problem c. Assessment
that a family encounters in performing health tasks with d. Planning
respect to given health condition or problem and etiology • Correct answers: Evaluation
or barriers to the family’s assumption of the task? 22. Cultural assessment is also a part of taking the
. Fourth Level of Assessment health history of your patient in formulating your family
a. Second Level Assessment case study, you know that which of the following are true
b. First Level Assessment regarding cultural sensitivity?
c. Third Level of Assessment . Patient's response to signs and symptoms are
• Correct answers: Second independent of their cultural values
Level Assessment a. All members of one cultural group behave in
15. The consumption of alcohol, tobacco, caffeine, or exactly the same manner
herbal products is important in health history, these are b. Cultural and ethnic diversity have no impact in
included in which of the following? health care
. Sleep and Rest c. As a nurse, it is important to identify and examine
a. Illegal activity our own cultural and ethnic beliefs
b. Developmental • Correct answers: As a nurse, it
c. Bibliographical is important to identify and
d. Habits and lifestyle patterns examine our own cultural
e. Rest and recreation and ethnic beliefs
• Correct answers: Habits and 23. Genogram is a traditional family tree by allowing
lifestyle patterns the user to analyze hereditary patterns and psychological
16. A phase of the nursing process that is considered factors that punctuate relationships, which includes the
a critical step as it is used to determine the course of following rules, except?
treatment? . The oldest child is always at the left his family, the
. Assessment youngest child is always at the right his family
a. Nursing Diagnosis a. Spouse must always be closer to his/her first
b. Implementation partner, then the second partner (if any), third partner, and
c. Evaluation so on.
d. Planning b. The female parent is always at the left of the
• Correct answers: Nursing family and the male parent is always at the right of the
Diagnosis family.
17. Typology/Category of health problem that c. In the case of ambiguity, assume a male-female
includes Anticipated periods of unusual demand on the or female-female relationship.
individual or family in terms of adjustment/family • Correct answers: The female
resources? parent is always at the left of
. Stress Points/Foreseeable Crisis the family and the male
a. Wellness Condition parent is always at the right
b. Health Deficits of the family.
c. Health Threats 24. Which of the following features should the nurse
• Correct answers: Stress incorporate in her practice of community health nursing?
Points/Foreseeable Crisis 1. CHN is oriented to population.
18. You have a patient who died due to pulmonary 2. CHN emphasizes health.
tuberculosis, as a Nurse in the community, you will be 3. CHN involves inter disciplinary collaboration.
recording it to the list of? 4. CHN promotes client participation in
. Morbidity determining their own health.
a. 3&4 c. In CHN, the client is considered as a passive
b. 1, 2, 3 & 4 partner.
c. 1&2 • Correct answers: • In CHN, the
d. 3&4 client is considered as a
• Correct answers: • 1, 2, 3 & 4 passive partner.
25. Assessment of the healthy family should find 32. Base on the definition of Community Health
which of the following? Nursing, as a Nurse in the community, which among the
a. Minimum influence is exerted on the environment following is your main goal?
b. The structure is flexible enough to adapt to crises . To provide care for every family in the community
c. Change is viewed as detrimental to family a. To raise the level of citizenry by helping
processes communities and families to cope up with the
d. There is a passive response to stressors discontinuities of threats to health in such a way as to
• Correct answers: The maximize their potential for high-level of wellness
structure is flexible enough b. To enhance proper collaboration with others to
to adapt to crises the promotion of the client’s optimum level of functioning
26. Situation: Nurse Stephen is conducting home visit thru’ the use of secondary health care services.
to the Cruz family, while providing care to the family, he c. To eliminate health problems in the community by
found out that Mr. Cruz lost his job two months ago, and helping the family to become a passive recipient of care.
still jobless, he discovered that the youngest child who is • Correct answers: To raise the
already one year and eight months old is still level of citizenry by helping
unimmunized, also, Mrs. Cruz verbalized presence of communities and families to
mass in her left breast, and found out that their 18-year- cope up with the
old daughter Ofelia is suffering from an eating disorder. discontinuities of threats to
He identified several health problems of the Cruz family, health in such a way as to
which of the following is/are classified as Health deficit?. maximize their potential for
. Unimmunized one and eight months old child high-level of wellness
a. 18-year-old daughter with eating disorder 33. It is the blueprint of the care that the nurse
b. Mr. Cruz lost his Job, and still Jobless for two designs to systematically minimize or eliminate the
months identified health and family nursing problems through
c. Presence of mass in the left breast of Mrs. Cruz explicitly formulated outcomes of care (goals and
d. A and D objectives) and deliberately chosen of interventions,
e. B and C resources and evaluation criteria, standards, methods
• Correct answers: B and C and tools.
27. This is an element of a Community denoting or . Family Care Plan
relating to human society or any of its subdivisions? a. Daily Plan of Activities
. Physical Entity b. Plan of Action
a. Psycho-cultural Entity c. Patient Nursing Care Plan
b. Geographical Entity • Correct answers: Family Care
c. Social Entity Plan
• Correct answers: Social Entity 34. As a nurse in the community, you are fully aware
28. Which of the following resources that the Nurse that there are identified four clienteles of a professional
can use that is defined as physical, intellectual capabilities nurse in community health nursing, these are?
and finances from the clientele? . Individual, Families, Organizations, and Youth
. Government resources a. Families, Individuals, Partners, and Communities
a. Natural resources b. Groups, Families, Civilizations, and Individuals
b. Nurse resources c. Communities, Groups, Families, and Individuals
c. Family resources • Correct answers: •
d. Community resources Communities, Groups,
• Correct answers: Family Families, and Individuals
resources 35. Typology/Category of health problem wherein
29. This is an element of a Community wherein a their are conditions that are conducive to disease and
feature that occupies a position in space about which data accident, or may result to failure to maintain wellness or
describing the attributes of the entity? realize health potential?
. Physical Entity . Stress Points/Foreseeable Crisis
a. Social Entity a. Health Threats
b. Psycho-cultural Entity b. Wellness Condition
c. Geographical Entity c. Health Deficits
• Correct answers: • Correct answers: Health
Geographical Entity Threats
30. Which of the following nursing interventions 36. The following, except for one are
wherein the nurse prepares some family members to give Factors/conditions that may bring non-compliance or non-
similar care in her absence? acceptance of the family to take actions on its each health
. Supplemental problems?
a. Accumulative . The family has the necessary information but fails
b. Developmental to relate them to the problem condition
c. Facilitative a. There may be adherence to patterned behavior
• Correct answers: b. The family’s information may be complete or
Developmental accurate
31. The following statements about Community c. The family is not willing to face the reality of the
Health Nursing (CHN) are correct, except? situation
. The goal of CHN is achieved through Multi- d. The members may not be willing to oppose
Sectoral Efforts. family, peer or social pressure
a. The Community is the patient in CHN
b. The family is the unit of care
• Correct answers: The family’s c. Health Advocate
information may be d. Collaborator
complete or accurate • Correct answers: Health
37. Typology/Category of health problem wherein Advocate
instances of failure in health maintenance is present? 44. Which of the following nursing interventions
. Health Threats wherein the nurse improves family’s physical
a. Wellness Condition environment?
b. Health Deficits . Supplemental
c. Stress Points/Foreseeable Crisis a. Developmental
• Correct answers: Health b. Facilitative
Deficits c. Accumulative
38. In performing the nursing process, you know that • Correct answers: Facilitative
the planning phase of the nursing process includes which 45. This is a type of Nursing Diagnosis that focused
of the following activities? on promoting or enhancing a patient’s level of health?
. Evaluating goal achievement . ACTUAL
a. Setting goals and selecting interventions a. SUBSTANCIAL
b. Performing nursing actions and documenting b. COLLABORATIVE
them c. POTENTIAL
c. Assessing and diagnosing d. WELLNESS
• Correct answers: Setting • Correct answers: WELLNESS
goals and selecting 46. A goal that provides direct treatment to the patient
interventions is?
39. The nurse has recently been employed in a long- . Administration focused goal
term care facility and must learn gerontological principles a. Client focused goal
related to families. Which of the following is one of those b. Nurse focused goal
principles? c. Hospital focused goal
. Role reversal is usually expected and well- • Correct answers: Client
accepted by the elderly client. focused goal
a. Social support systems are likely to differ from 47. Which of the following nursing interventions
those for clients in younger age groups. wherein the nurse provides direct nursing care services to
b. Members of later-life families do not have to work the sick?
on developmental tasks. . Developmental
c. The care givers are often not members of the a. Facilitative
family. b. Accumulative
• Correct answers: • Social c. Supplemental
support systems are likely to • Correct answers:
differ from those for clients Supplemental
in younger age groups. 48. In completing a client’s family assessment, the
40. The following, except for one are Factors that nurse should begin by:
influence the goal formulation? . Determining the family’s form and attitudes
. Interpersonal relationship a. Testing the family’s ability to cope
a. Families individualize care to clients b. Gathering the health data from all family
b. Families felt need members
c. Families perception of the problem c. Evaluating communication patterns
• Correct answers: Families • Correct answers: Determining
individualize care to clients the family’s form and
41. This is an element of a Community relating to the attitudes
interaction of people in which individuals live with their 49. Which of the following resources that the Nurse
inner characteristics? can use that includes skills, knowledge, and time of the
. Geographical Entity health provider?
a. Social Entity . Community resources
b. Psycho-cultural Entity a. Family resources
c. Physical Entity b. Nurse resources
• Correct answers: Psycho- c. Natural resources
cultural Entity d. Government resources
42. The Nursing process is a scientific and • Correct answers: Nurse
systematized approach to health to care for individuals, resources
families, and illness prevention. The following are the 50. Nurse Theodore is a Nurse working in the
characteristics of the Nursing Process, except? community and reporting cases to the rural health unit of
. Open and Flexible Victoria Tarlac city, he is aware that according to the
a. Intrapersonal and collaborative Department of Health, the care being provided in a health
b. Patient centered center is?
c. Dynamic and cyclic . Secondary Health Care
d. Problem Oriented a. Tertiary Health Care
e. Goal directed b. Primary Health Care
• Correct answers: c. First-Line Care
Intrapersonal and • Correct answers: • Primary
collaborative Health Care
43. As a Nurse in the community, we have several 51. This term is defined as length of life a study, or
roles, and one of this role is defined as the one "who average lifespan?
speaks on behalf of the client".? . Death rate
. Health Educator a. Mortality
a. Facilitator b. Longevity
b. Clinician c. Morbidity
• Correct answers: • Morbidity 60. As a Nurse in the community, we have several
(pero Longevity yata) roles, and one of this role is defined as working with other
52. Which of the following resources that the Nurse health team member?
can use that includes RHU, clinics, Barangay Hall? . Health Advocate
. Family resources a. Collaborator
a. Natural resources b. Health Educator
b. Community resources c. Clinician
c. Government resources d. Facilitator
d. Nurse resources • Correct answers:
• Correct answers: Community Collaborator
resources
53. This is a Graphic representation of a family tree
that displays detailed data on relationships among
individuals?
. Initial Data Base
a. Family Assessment Guide
b. Ecomap
c. Home and Environment
d. Genogram FINALS
• Correct answers: Genogram 1. Which criterion in priority setting of health
54. A type of Nursing Diagnosis that identifies an problems is used only in community health care?
occurring health problem? a. Nature of the problem presented
. SUBSTANCIAL b. Modifiability of the problem
a. WELLNESS c. Magnitude of the health problem
b. POTENTIAL d. Preventive potential of the health problem
c. ACTUAL • Correct answers: Magnitude
d. COLLABORATIVE of the health problem
e. Correct answers: ACTUAL 2. Community health nursing is a field of nursing in
55. Another type of Nursing Diagnosis that identifies which it utilizes which of the following?
a high-risk health problem? 1. Knowledge about the history and
. POTENTIAL philosophy of public health practices.
a. WELLNESS 2. Concepts of epidemiology and bio-
b. ACTUAL statistics.
c. SUBSTANCIAL 3. Principles of leadership and
d. COLLABORATIVE management
• Correct answers: POTENTIAL 4. Principles of social change
56. You are the Nurse assigned to the RHU, you have • Correct answers: 1, 2, 3 & 4
observed individuals being sick due to illnesses in a 3. An indicator of success in community organizing
specified period of time, you know that this will be is when people are able to?
recorded as? a. Plan activities for the solution of the community
. Longevity problem
a. Death rate b. Identify the health problem as a common
b. Mortality concern
c. Morbidity c. Participate in community activities for the
• Correct answers: • Morbidity solution of a community problem
57. This is the phase of the Nursing Process wherein d. Plan activities for the solution of the community
the process of collecting, validating and recording data problem
about a client’s health status is being done? • Correct answers: Participate
. Evaluation in community activities for
a. Nursing Diagnosis the solution of a community
b. Implementation problem
c. Assessment 4. Is an official and periodic enumeration of the
d. Planning population. Demographic, economic, and social data are
• Correct answers: Assessment collected from a specified population group.
58. On this phase of the nursing process, the nurse . Census
considered each problem is assigned a clear, measurable a. Sample survey
goal for the expected beneficial outcome, as well as b. De Jure Method
strategies of action if being developed? c. De Facto Method
. Assessment • Correct answers: Census
a. Evaluation 5. Which of the following is an epidemiologic
b. Planning function of the nurse during an epidemic?
c. Nursing Diagnosis . Monitoring the condition of the cases affected by
d. Implementation the communicable disease
• Correct answers: • Planning a. Participating in the investigation to determine the
59. Which goal is being attained by teaching the client source of epidemic
or member of the family to perform a certain procedure? b. Teaching the community on preventive
. Nurse focused goal measures against the disease
a. Client focused goal c. Conducting assessment of suspected cases to
b. Administration focused goal detect the communicable diseases
c. Hospital focused goal • Correct answers:
• Correct answers: Nurse Participating in the
focused goal investigation to determine
the source of epidemic
6. Nurse Elsa is assessing the SOCIO-ECONOMIC depend largely on the community organizer's extent of
AND CULTURAL VARIABLES of barangay San integration with the people, her/his understanding of the
Sebastian. In assessing the Socio-Economic and events in the community?
Cultural Variables, she will include the following, except? . Entry Phase
1. Economic indicators a. Face-to-face
2. Environmental indicators b. Pre-entry Phase
3. Cultural factors c. Loading Phase
4. Leadership Factors d. Helping Phase
5. Social indicators e. Phase out
• Correct answers: f. Assigning Phase
Leadership Factors g. Mission Phase
7. According to Freeman and Heinrich, community • Correct answers: Entry Phase
health nursing is a developmental service. Which of the 12. Criteria for Nature of the Problem Presented
following best illustrates this statement? Scale are the following but one?
a. Health education and community organizing are . Health Deficit
necessary in providing community health services. a. Foreseeable Crisis
b. Community health nursing is intended primarily b. Environmental Health
for health promotion and prevention and treatment of c. Health Threat
disease. • Correct answers:
c. The community health nurse continuously Environmental Health
develops himself personally and professionally 13. In a mothers’ class, you discuss proper
d. The goal of community health nursing is to breastfeeding technique. Which is of these is a sign that
provide nursing services to people in their own places of the baby has “latched on” to the breast properly?
residence. . The mother does not feel nipple pain.
• Correct answers: Health a. The baby takes shallow, rapid sucks.
education and community b. Only the mother’s nipple is inside the baby’s
organizing are necessary in mouth
providing community health c. The baby’s mouth is only partly open.
services. • Correct answers: The mother
8. As a nurse in the community, you are fully aware does not feel nipple pain.
that there are identified four clientele of a professional 14. As a nurse in the community, you also have an
nurse in community health nursing, these are? active role in disease surveillance and epidemiology of
. Groups, Families, Civilizations, and Individuals disease in the community. Among others you also do
a. Individual, Families, Organizations, and Youth which of the following?
b. Communities, Groups, Families, and Individuals 1. Immediately inform the rural health
c. Families, Individuals, Partners, and physician of any unusual increases
Communities cases in the community disease.
• Correct answers: 2. Conduct epidemiological investigation
Communities, Groups, on all reported outbreaks.
Families, and Individuals 3. Request the midwife to graph weekly
9. As a rural health unit nurse, you also have an disease incidence cases in the
active role in disease surveillance and epidemiology of municipality.
disease in the community. Among others you also do the 4. Assist in the planning and
following. implementing disease control measures
1. Immediately inform the rural health during outbreak.
physician of any unusual increases a. 1, 2, 3 & 4
cases in the community disease. b. 2&4
2. Conduct epidemiological c. 1&4
investigation on all reported d. 1, 2 & 4
outbreaks. e. 3&4
3. Request the midwife to graph • Correct answers: 1, 2 & 4
weekly disease incidence cases in 15. Which of the following phases of the Community
the municipality. Organizing is also known as social preparation of the
4. Assist in the planning and community?
implementing disease control a. Phase out
measures during outbreak b. Assigning Phase
• Correct answers: 1, 2 & 4 c. Face-to-face
10. You are assessing the family households in your d. Mission Phase
assigned community. 50% of the family you have e. Pre-entry Phase
assessed is made up of parents, their children, and f. Helping Phase
relatives of one or both the parent. You know that this g. Loading Phase
type of family is called? h. Entry Phase
a. Gay/lesbian • Correct answers: Entry Phase
b. Single-parent Family 16. Which of the following features should the nurse
c. Communal incorporate in her practice of community health nursing?
d. Reconstituted 1. CHN is oriented to population.
e. Compound 2. CHN emphasizes health.
f. Nuclear Family 3. CHN involves inter disciplinary
g. Foster collaboration
h. Extended Family 4. CHN promotes client participation in
• Correct answers: Extended determining their own health.
Family a. 3 & 4 only
11. This phase of the Community Organizing is b. 1, 2, 3 & 4
considered crucial because the success of later activities c. 1 & 2 only
d. 2 & 4 only . Core group Formation
e. 1, 2 & 3 a. Home Visits
• Correct answers: 1, 2, 3 & 4 b. Follow-up
17. While conducting a community diagnosis, you c. Mobilization
have assessed several number of Dengue cases in the d. Community Organization
area. Dengue Fever is a viral infection transmitted • Correct answers: Community
through bite of vector Aedes Aegypti, this vector is Organization
characterized by? 23. Epidemiology is the study of occurrences and
a. Day biting Male Mosquito distribution of diseases as well as distribution and
b. Day biting female Mosquito determinants of health states or events in specified
c. Male biting mosquito population, and the application of this study to the
d. Night biting female mosquito control of health problems. Which of the following is a
e. Any mosquito function of epidemiology?
• Correct answers: Day biting . Evaluating the effectiveness of the
female Mosquito implementation of the Integrated Management of
18. You are assessing the family households in your Childhood Illness.
assigned community. Majority of the family you have a. Determining the efficacy of the antibiotic used in
assessed is made up of parents and one or more the treatment of the 3 year old client with pneumonia.
children living together. You know that this type of family b. Identifying the disease condition based on
is called? manifestations presented by a client.
. Extended Family c. Determining factors that contributed to the
a. Single-parent Family occurrence of pneumonia in a 3 year old.
b. Nuclear Family • Correct answers: Evaluating
c. Gay/lesbian the effectiveness of the
d. Compound implementation of the
e. Communal Integrated Management of
f. Reconstituted Childhood Illness
g. Foster 24. Which criterion in priority setting of health
• Correct answers: Nuclear problems is used only in community health care?Single
Family choice.
19. On your home visit at Family Cassano, a 7 . Preventive potential of the health problem
month old infant has Koplik Spots. As the community a. Modifiability of the problem
nurse, you are aware that this is a pathognomonic sign b. Nature of the problem presented
of measles Upon assessment you may see Koplik’s spot c. Magnitude of the health problem
by inspecting the? • Correct answers: Magnitude
. Buccal mucosa of the health problem
a. Nasal Mucosa 25. Community is defined as a group of people living
b. Skin on the abdomen in the same place or having a particular characteristic in
c. Skin on the antecubital surface common. While _________ pertains to the size,
• Correct answers: Buccal composition, and geographical distribution of the
mucosa population density?
20. Batang-Batang is your chosen community for . Demographic Variables
CHN 2. The perception of the existing problem is a. Community
important in the conduct of a Community Diagnosis. This b. Urban-Rural Distribution
refers to the perception of the population or the c. Population
community as they are affected by the problem and their d. Socio-Cultural Variables
readiness to act on the problem. • Correct answers:
. Social Reform Demographic Variables
a. Social Norm 26. You are assigned to Barangay Pokis as the
b. Socialization community nurse, you are aware of the different roles
c. Social Concern and functions of each member of the healthcare team
d. Social Agenda assigned on the said barangay. Being the Community
• Correct answers: Social Nurse on this area, you are aware that all of the
Concern following are not purposes of the home visit, except?
21. Environmental sanitation is defined as? . Team approach
. Study the pattern of occurrence of disease a. Endorsement done by a midwife to another
epidemiology. midwife
a. None of the choices b. Two-way referral system
b. A branch of public health that deals with the c. Giving health teaching regarding the prevention
study of preventing illnesses managing the environment and control of diseases.
and environmental hygiene. • Correct answers: Giving
c. Study all factors in man’s physical environment health teaching regarding
which may exercise a deleterious effect on his wellbeing the prevention and control
and survival. of diseases.
d. All of the choices 27. The following are the reasons of moving to
• Correct answers: A branch of Phase out phase of the community organizing, except?
public health that deals with . When the impact of the project has become
the study of preventing visible or change has been made
illnesses managing the a. When a participatory and politically responsive
environment and community starts to become individualistic
environmental hygiene. b. When a viable community-based organization
22. In conducting the community diagnosis, which has been established
step are plans formulated for solving community c. When the objectives have been attained
problems?
d. When the community resources can already be the component will alter an existing equilibrium to
maximized by the people increase or decrease the frequency of the disease. Label
e. When the members of the community can take the part of the Epidemiologic Triangle being asked: C is
over the planning, implementation, monitoring, and ___________________?
evaluation of the project.
• Correct answers: When a
participatory and politically
responsive community
starts to become
individualistic
28. A community diagnosis is a hypothesis or
statement of the outcome of the analysis and synthesis
of the data and information acquired during data
collection about a community. All of the following are . Host
sources of data: a. Agent
1. Population census b. Etiologic Factor
2. Indices of the health c. Environment
3. Statistics d. Nurse
4. Registration of vital data • Correct answers:
5. Health survey Environment
6. Studies and researches. 34. The public health nurse is responsible for
• Correct answers: 1, 4, 5 and presenting the municipal health statistics using graphs
6 and tables. To compare the frequency of the leading
29. It measures the frequency of occurrence of the causes of mortality in the municipality, which graph will
phenomena during a given period of time you prepare?
a. Incidence Rate . Pie
b. Incidence Death Rate a. Rows and Columns
c. Crude Death Rate b. Scatter diagram
d. Prevalence Rate c. Bar
• Correct answers: Incidence d. Line
Rate • Correct answers: Bar
30. Assessment provides an estimate of the degree 35. When the community health nurse visits a client
to which a family or community achieve the level of at home, the client states, “I haven’t slept at all the last
health, identifies specific deficiencies or guidance couple of nights.” Which response by the nurse
needed and estimates the possible effects of the nursing illustrates a therapeutic communication response to this
intervention. Arrange the following into its proper client?
sequence: . “Sometimes, I have trouble sleeping too.”
1-Initiate contact a. “You’re having difficulty sleeping?”
2-Mutual trust/ confidence b. “I see.”
3-Demonstrate caring attitude c. “Really?”
4-Analyze and interprets data • Correct answers: “You’re
5-Identify health problem 6-Collect data having difficulty sleeping?”
from all sources 36. The Epidemiologic Triangle consists of three
7-Assess coping abilities. components – the host, environment, and agent. As a
• Correct answers: 1, 3, 2, 6, nurse, you are certain that the three components of the
5, 7, 4 environment are which of the ff:
31. Which of the following is an element of a 1. Host
Comprehensive Community Diagnosis? 2. Agent
a. DEMOGRAPHIC VARIABLES 3. Physical
b. SOCIO-ECONOMIC AND CULTURAL 4. Biological
VARIABLES 5. Environment
c. HEALTH AND ILLNESS PATTERNS 6. Socio-economic
d. HEALTH RESOURCES • Correct answers: 3, 4 and 6
e. POLITICAL/ LEADERSHIP PATTERNS 37. Cultural factors influence community practice.
• Correct answers: All of the The variable that may break up People into Groups
following within the community includes all of the following
32. A primigravida is instructed to offer her breast to except?
the baby for the first time within 30 minutes after a. Ethnicity, Social Class, Language, Blood Type
delivery. What is the purpose of offering the breast this b. Ethnicity, Social Class, Language, Religion,
early? Race, Political Orientation
. To make sure that the baby is able to get the c. Race, Political Orientation
colostrum d. None of the choices
a. To allow the woman to practice breastfeeding in e. All of the choices
the presence of the health worker • Correct answers: Ethnicity,
b. To stimulate milk production by the mammary Social Class, Language,
acini Blood Type
c. To initiate the occurrence of milk letdown 38. A patient asks Nurse Seo about the primary
• Correct answers: To stimulate focus of Community Health Nursing. Which statement
milk production by the made by Nurse Seo is unacceptable?
mammary acini . It focuses on primary level of care
33. The Epidemiologic Triangle consists of three a. The primary focus is performing the triage
components. The model implies that each must be approach
analyzed and understood for comprehensions and b. The individual, family and community are the
prediction of patterns of a disease. A change in any of main focus of public health nursing
c. Community Health Nursing focuses on health danger sign that indicates the need for urgent referral to
promotion aspect a hospital?
• Correct answers: The primary a. Inability to drink
focus is performing the b. High grade fever
triage approach c. Signs of severe dehydration
39. Community Health Nursing is the utilization of d. Crying
the nursing process in the different levels of clientele- • Correct answers: Inability to
individuals, families, population groups and drink
communities, concerned with the promotion of health,
prevention of disease and disability and rehabilitation.”
(Maglaya, et al). Base on the definition of CHN above,
as a Nurse in the community, your main goal is?
. To raise the level of citizenry by helping
communities and families to cope up with the
discontinuities of threats to health in such a way as to
maximize their potential for high-level of wellness.
a. To enhance proper collaboration with others to
the promotion of the client’s optimum level of functioning 44. The Epidemiologic Triangle consists of three
thru’ the use of secondary health care services. components. The model implies that each must be
b. To eliminate health problems in the community analyzed and understood for comprehensions and
by helping the family to become a passive recipient of prediction of patterns of a disease. A change in any of
care. the component will alter an existing equilibrium to
c. All of the choices increase or decrease the frequency of the disease. Label
• Correct answers: To raise the the part of the Epidemiologic Triangle being asked: A is
level of citizenry by helping ___________________?
communities and families to . Etiologic Factor
cope up with the a. Agent
discontinuities of threats to b. Environment
health in such a way as to c. Nurse
maximize their potential for d. Host
high-level of wellness. • Correct answers: Agent
40. You are assessing the family households in your 45. Population-focused nursing practice requires
assigned community. One of of the family you have which of the following processes
assessed is the family of Freddie who is a Muslim . Community diagnosis
Brother, and it is legal in his Religion to have more than a. Community organizing
one wives. He has three wives, Lily, Shelly, and Georgia b. Nursing process
who are living with Freddie in harmony. What type of c. Epidemiologic process
family do they have? • Correct answers: Community
. Single-parent Family diagnosis
a. Nuclear Family 46. CHN is a community-based practice. Which best
b. Compound explains this statement?
c. Extended Family . The nurse has to conduct community diagnosis
d. Gay/lesbian to determine nursing needs and problems
e. Reconstituted a. The service is provided in the natural
f. Foster environment of people
g. Communal b. The services are based on the available
• Correct answers: Compound resources within the community
41. You will gather data for nutritional assessment of c. Priority setting is based on the magnitude of the
Purok Cassano. You will gather information only from health problems identified
families with members who belong to the target • Correct answers: The nurse
population for PEM. What method of delta gathering is has to conduct community
best for this purpose? diagnosis to determine
. Survey nursing needs and
a. Record Review problems
b. Census 47. On the computation for ranking of identifying
c. Review of civil registry health conditions and problems according to priorities
• Correct answers: Survey there are four criteria. On the criteria nature of the
42. The midwife refers to you a family who fails to condition or problem presented, wellness stated is
take action regarding a sick member of the family graded as?
despite her suggestions. In helping the family decide on . 4
what actions to take, you will? a. 3
1. Explain the consequences of inaction b. 5
2. Identify the courses of action open to c. 1
the family and resources needed for d. 2
each e. 0
3. Discuss the consequences of each • Correct answers: 3
course of action available 48. The DOH roles are categorized into three major
4. Influence the family to act on what functions, namely: i. Leadership in Health ii. Enabler and
you think is best for the situation. Capacity Builder iii. Community Organizer iv.
• Correct answers: 2, 3 & 1 Administrator of specific Services.
43. A mother brought her daughter, 4 years old, to . i, iii, iv
the RHU because of cough and colds. Following the a. All of the choices
IMCI assessment guide, which of the following is a b. i, ii, iv
c. ii, iii, iv
d. i, ii, iii previous cases recorded, This Patterns of occurrence
• Correct answers: i, ii, iv and distribution of diseases is called?
49. The primary purpose of conducting an a. ECONOMIC OCCURRENCE
epidemiologic investigation is to? b. EPIDEMIC OCCURRENCE
. Identify geographical location of cases of the c. SPORADIC OCCURRENCE
disease in the community d. PANDEMIC OCCURRENCE
a. Identify groups who are at risk of contracting the e. ENDEMIC OCCURRENCE
disease • Correct answers: EPIDEMIC
b. Delineate the etiology of the epidemic OCCURRENCE
c. Encourage cooperation and support of the 54. A mother brought her 10 month old infant for
community consultation because of fever, which started 4 days prior
• Correct answers: Delineate to consultation. To determine malaria risk as a
the etiology of the epidemic epidemiological nurse, what will be your next action?
50. The Epidemiologic Triangle consists of three . Get a specimen for blood smear.
components. The model implies that each must be a. Ask where the family resides.
analyzed and understood for comprehensions and b. Ask if the fever is present everyday.
prediction of patterns of a disease. A change in any of c. Do a tourniquet test.
the component will alter an existing equilibrium to • Correct answers: Ask where
increase or decrease the frequency of the disease. Label the family resides.
the part of the Epidemiologic Triangle being asked: B is 55. If nurses are busy, hilots can really be of help
___________________? says Mrs. Cruz of Barangay Papaac. Based on your
knowledge as a nurse, you will include on your health
teaching what specific case that could be assigned on
hilots nowadays?
. Immunization
a. None as hilots are not advisable to attend
deliveries nowadays
b. Normal deliveries
c. Would dressing
• Correct answers: None as
. Agent hilots are not advisable to
a. Host attend deliveries nowadays
b. Etiologic Factor 56. You are conducting pre-natal counseling on your
c. Nurse home visit in the community and dealing with a
d. Environment primigravida who is afraid of childbirth due to her beliefs
• Correct answers: Host and misconceptions. The first thing that the community
51. In Sitio De Pinagpala, a pregnant woman named nurses should do is?
Mrs. Kang seeks help from a hilot. She plans to give . Refer to midwife for regular follow-up visit.
birth in her home and not on a health care facility. As a a. Help woman recognize the sign of abnormalities
nurse what will you tell Mrs. Kang about her decision? in pregnancy to enable her to report to the midwife.
. Facility Based Delivery is highly recommended b. Identify the beliefs and misconceptions and their
nowadays as it prevents the risk for postpartum related source.
injuries and neonatal tetanus. c. Immediately correct the beliefs and
a. Hilots are way cheaper than hospital delivery misconceptions.
b. Hilot and Facility Based Delivery are just • Correct answers: Identify the
synonymous, so you can go to choose either one of it. beliefs and misconceptions
c. Hilots render holistic touch compared to Rural and their source
Health Midwives 57. In preparing a primigravida for breastfeeding,
• Correct answers: Facility which of the following will you do?
Based Delivery is highly . Explain to her that putting the baby to breast will
recommended nowadays as lessen blood loss after delivery.
it prevents the risk for a. Instruct her to wash her nipples before and after
postpartum related injuries each breastfeeding.
and neonatal tetanus. b. Teach her nipple stretching exercises if her
52. In performing the bag technique while doing your nipples are everted.
Community Diagnosis, it is very important to observe the c. Tell her that lactation begins within a day after
correct sequence in doing it. All of the following delivery.
sequence are correct sequence, except? d. None of the choices
1. Greet the client and introduce yourself e. All of the choices
2. Upon arriving at client’s home place • Correct answers: Explain to
the bag on a flat surface lined with paper her that putting the baby to
lining breast will lessen blood
3. Do handwashing technique and loss after delivery.
perform the procedure 58. Promotion of health is a major responsibility of
4. Ask for a basin of water if tap water is the CHN, this refers to?
not available . Activities that seek to protect clients from
5. Perform the needed care. potential or actual health threats and their harmful
6. Put out all the specific paraphernalia consequences.
needed for specific case. a. Activities aimed to prevent the spread of
• Correct answers: a. 1, 2, 3, 4 microorganisms to Individual.
and 5 b. Activities directed toward decreasing the
53. The rise of Dengue cases over the last 3 probability of specific illness or dysfunctions in
decades in the Philippines has seen an increase in the individuals.
number of people who have Dengue is higher than the
c. Activities directed toward developing the c. Urban-Rural Distribution
resources of clients that maintain or enhance well-being. d. Population Density
• Correct answers: Activities • Correct answers: Population
directed toward developing Distribution
the resources of clients that 64. This phase of the Community Organizing mean
maintain or enhance well- that a program is already community-managed?Single
being choice.
59. This phase of the Community Organizing covers . Face-to-face
gathering data and encouraging people to identify and a. Helping Phase
analyze their needs and problems? b. Assigning Phase
. Face-to-face c. Entry Phase
a. Loading Phase d. Mission Phase
b. Mission Phase e. Pre-entry Phase
c. Assigning Phase f. Loading Phase
d. Pre-entry Phase g. Phase out
e. Entry Phase • Correct answers: Phase out
f. Helping Phase 65. Which is CONTRARY to the principles in
g. Phase out planning a home visit?
• Correct answers: Helping . Planning of continuing care should involve a
Phase responsible family member
60. In Barangay Nagibana, a rural community, a. A home visit should be conducted in the manner
Clostridium tetani, the bacterium that causes tetanus, is prescribed by the RHU.
acquired in the soil environment, but incidences of b. The plan should revolve around family health
infection occur only rarely and in scattered locations needs.
because most individuals are vaccinated, This Patterns c. A home visit should have a purpose or objective.
of occurrence and distribution of diseases is called? • Correct answers: A home visit
. ECONOMIC OCCURRENCE should be conducted in the
a. EPIDEMIC OCCURRENCE manner prescribed by the
b. SPORADIC OCCURRENCE RHU.
c. ENDEMIC OCCURRENCE 66. In the past year, Barangay A had an average
d. PANDEMIC OCCURRENCE population of 1655. 46 babies were born in that year, 2
• Correct answers: SPORADIC of whom died less than 4 weeks after they were born.
OCCURRENCE They were 4 recorded stillbirths. What is the neonatal
61. Which statements made by a group of mortality rate?
community clients communicate to the nurse a potential . 86.9/1000
warning sign of cancer? Select all that apply. a. 130.4/1000
1. "For the past 2 years, I have had a b. 27.8/1000
chronic productive cough for 3 months c. 43.5/1000
out of the year." • Correct answers: 43.5/1000
2. "I seem to have heartburn an hour 67. You have been assigned as a community health
after I eat eggs and sausage and drink nurse in a poor community providing primary health care
whole milk." services, which of the following characterize Primary
3. "Last month when I was doing my Health Care?
self-breast examination, I noticed a lump 1. Essential health care accessible and
the size of a marble." acceptable to individual and families in
4. "My mole is itchy and the borders the community.
have become uneven with a blackish to 2. The cost of care is affordable to
bluish color." community and country.
5. "Recently I have noticed that my 3. The setting is in tertiary hospitals.
bowel movements appear black." 4. Require health personnel and
• Correct answers: 3, 4, and 5 specialized training.
62. The following choices are the purposes of home • Correct answers: 1 & 2
visit that a nurse must be utilize in performing community 68. The following are uses of Epidemiology, except?
diagnosis and family case study. All of the following are a. Estimate the risk of disease, accident, and
true except? defects
1. To give health teachings regarding b. Motivating, enhancing and seeking wider
the prevention and control of diseases. community participation in decision making and activities
2. To teach a responsible family that have the potential to impact positively on community
member to give the subsequent care. health
3. To give care to the sick, to a c. Complete the clinical picture of chronic disease
postpartum mother and her newborn d. Study the rise and fall of diseases and changes
with the view to teach any family in their character
member to give the subsequent care. e. Study the history of the health population
4. To make use of the inter-referral f. Study the work of health services with a view of
system and to promote the utilization of improving them
community services. g. Diagnose the health of the community
• Correct answers: 2, 3, 4 • Correct answers: Motivating,
63. This can be describe in terms of urban-rural enhancing and seeking
distribution, population density, and crowding index for wider community
proper allocation of resources based on the participation in decision
concentration of population in a certain place:Single making and activities that
choice. have the potential to impact
a. Crowding Index positively on community
b. Population Distribution health
69. Community Organizing (CO) is a social individual, family and
development approach that aims to transform the community
apathetic, individualistic and voiceless poor into a 73. You are assessing the family households in your
dynamic, participatory and politically responsive assigned community. One of of the family you have
community. Which step in community organizing assessed is the family of Mr. and Mrs. Manio who has
involves training of potential leaders in the community? three sons, all of them are married, and have their family
. Core group formation of their own. However, due to financial issue, all of their
a. Community organization sons with their families are still living inside the same
b. Community study roof. What type of family do they have?
c. Integration . Gay/lesbian
• Correct answers: Core group a. Single-parent Family
formation b. Compound
70. You are assigned to Barangay Bacsay for your c. Communal
RLE experience, during your community exposure your d. Foster
Clinical Instructor ask you about the guidelines regarding e. Nuclear Family
the frequency of home visit. You are correct if you f. Reconstituted
answer all of the following except? g. Extended Family
1. The physical needs psychological • Correct answers: Communal
needs and educational needs of the 74. This is also known as community involvement
individual and family. phase of the Community Organizing?
2. The frequency of home visit should . Face-to-face
take into consideration the use of all a. Loading Phase
available information about the patient b. Phase out
and his family through family records. c. Helping Phase
3. The acceptance of the family for the d. Pre-entry Phase
services to be rendered, their interest e. Assigning Phase
and the willingness to cooperate. f. Mission Phase
4. The policy of a specific agency and g. Entry Phase
the emphasis given towards their health • Correct answers: Helping
programs. Phase
5. Take into account the manner 75. This phase of the Community Organizing is also
prescribed by the RHU regarding home known as project site selection?
visit and the number of health personnel . Assigning Phase
already involved in the care of a specific a. Loading Phase
family. b. Mission Phase
6. Careful evaluation of past services c. Face-to-face
given to the family and how the family d. Entry Phase
avails of the nursing services. e. Pre-entry Phase
7. The ability of the nurse to recognize f. Helping Phase
the needs of the family, their knowledge g. Phase out
of available resources and their ability to • Correct answers: Pre-entry
make use of their resources for their Phase
benefits.
• Correct answers: 2, 3, 4, 5, 6
and 7
NCM 113 CHN2 Lec
71. Which of the following phases of the Community
Organizing wherein the Nurse is conducting of MIDTERMS
preliminary social analysis of the community is needed to
be able to plan the most effective way of entering the 1. The community health nurse chose Barangay
community? Sairo in the delivery of health programs and
a. Face-to-face services. Green’s Model was adopted in the
b. Loading Phase conduct of Community Health Nursing activities.
c. Mission Phase The following are the most common type of
d. Entry Phase instrument of data gathering in social diagnosis,
e. Phase out except:
f. Pre-entry Phase a. Survey questionnaire
g. Helping Phase b. Nominal and Focus Groups
h. Assigning Phase c. Structured Interview guide
• Correct answers: Pre-entry d. Medical and Laboratory records
Phase • Correct answers: Medical and
72. Community organizing is a continuous and Laboratory records
sustained process which develop the following, except? 2. One of the models used in CHN is Health Belief
. Organizing people to work collectively and Model. Which of the following are included in this
efficiently. model as its focus?
a. Guiding people to understand the existing . Promoting and health risk behaviors, sick role
condition of their own community. behaviors and attitudes
b. Take action on their immediate and long-term a. Enhancing functional ability, well- being and
needs. better quality of life
c. Mobilizing people to develop their capacity and b. Expanding health education through policy,
readiness. regulatory and related ecological factors
d. Improving nursing services for the individual, c. Challenging the main determinant for unhealthy
family and community. behavioral choice is lack of knowledge
• Correct answers: Improving • Correct answers: Promoting
nursing services for the and health risk behaviors,
sick role behaviors and maximizing the success of
attitudes the efforts of the Community
3. One of the highlights of Millo’s model is: Health Nurses.
. Competence oriented model 8. In CHN, the client is considered as a/an:
a. Policy development . Active partner
b. Health-related behaviors a. Passive partner
c. Educational and ecological diagnosis b. Dormant partner
• Correct answers: Policy c. Submissive partner
development • Correct answers: Active
4. It is a learned practice discipline with the ultimate partner
goal of contributing as individuals and in 9. Which of the following statements is not true in
collaboration with others to the promotion of the the concept of CHN services?
client’s optimum level of functioning through . CHN practice is affected by developments in
teaching and delivery of care.: health technology, in particular, changes in society.
. Occupational health nursing a. The goal of CHN is achieved through multi-
a. School nursing sectoral efforts.
b. Community mental health nursing b. CHN is a part of health care system and the larger
c. Community health nursing human services system.
• Correct answers: Community c. CHN focuses on the recognized needs of the
health nursing health workers for better follow up services.
5. Giving priority to the health problems presented • Correct answers: CHN
by the four levels of clientele is which of the focuses on the recognized
following CHN principles applicable? needs of the health workers
. Active participation of the individual, family and for better follow up services.
community in planning and making decisions for their 10. The community health nurse works with many
health care needs, determine, to a large extent, the people including clients, other nurses, physicians,
success of the CHN programs. social workers, and community leaders,
a. The recognized need of individuals, families and therapists, nutritionists, occupational therapists,
communities provides the basis for CHN practice. psychologists, epidemiologists, biostatisticians,
b. Respect for the values, customs and beliefs of the legislators as a member of the health team:
clients contribute to the effectiveness of care to the client. . Managerial role of the CH Nurse
c. CHN considers the family as the unit of service. a. Collaborative role of the CH Nurse
Its level of functioning is influenced by the degree to which b. Advocate role of the CH Nurse
it can deal with its own problems. c. Clinician role of the CH Nurse
• Correct answers: The • Correct answers:
recognized need of Collaborative role of the CH
individuals, families and Nurse
communities provides the 11. It is a psychological health model that attempts to
basis for CHN practice. explain behavior by focusing on the attitudes and
6. Working with the mission and vision of the agency behaviors of individuals.
the CHN represents is what CHN principle? . Precede-Proceed model
. Knowledge and understanding of the objectives a. Health Promotion model
and policies of the agency facilities goal achievement. b. Health Prevention model
a. Collaborative work relationships with the co- c. Health Belief model
workers and members of the health team facilities • Correct answers: Health Belief
accomplishments of goals. model
b. Utilization of indigenous and existing community 12. The following are the characteristics of a “healthy
resources maximizing the success of the efforts of the community”
Community Health Nurses. 1. Awareness that “we are a family”
c. Continuing staff education program quality 2. Settling of disputes through legitimate
services to client and are essential to upgrade and mechanisms
maintain sound nursing practices in their setting. 3. Decision-making is done by the learned and
• Correct answers: Knowledge the powerful
and understanding of the 4. Conservation of natural resources.
objectives and policies of a. 1, 2, 3
the agency facilities goal b. All of the above
achievement. c. 2, 3, 4
7. The use of local available health resources and d. 1, 2, 4
linkages both public and private, increase the e. Correct answers: 1, 2, 4
awareness of what care they need which they are 13. Among the characteristics of an unhealthy
entitled is which of the following CHN principles? community are:
. Collaborative work relationships with the co- 1. Participation of subgroups in the affairs of the
workers and members of the health team facilities community
accomplishments of goals. 2. Very strong inclination towards the family’s
a. CHN integrates health education and counseling welfare
as vital parts of functions. 3. Instead of the community’s interest
b. Utilization of indigenous and existing community 4. Decision-making is done by the learned and
resources maximizing the success of the efforts of the the powerful
Community Health Nurses. 5. Absence of distributive justice
c. Respect for the values, customs and beliefs of the a. 1, 2, 3
clients contribute to the effectiveness of care to the client. b. All of the above
• Correct answers: Utilization of c. 2, 3, 4
indigenous and existing d. None of the above
community resources • Correct answers: 2, 3, 4
14. The primary focus of the community health b. Community health
nursing practice in terms of maintaining the people’s c. Demography
optimum level of functioning is: • Correct answers: Health
a. Health promotion Indicators
b. Prevention of communicable disease 22. The study of vital events of the population is
c. Reduction of lifestyle diseases called:
d. Disease prevention . Health Indicators
• Correct answers: Health a. Population Profile
promotion b. Community health
15. In the communities, families from the lower c. Demography
income groups are the ones mostly served by the public • Correct answers: Demography
health services and by the community health nurses 23. The theoretical framework of Millo consists of
because: which of the following:
. They have proportionally great number of illness . Individual Perception, Modifying factors,
and health problems Likelihood of axtions
a. Those from the higher income groups do not trust a. Perception of susceptibility,severity, severity,
the government services benefits, costs, motivation & modifying factors
b. The poor people have no choice but to go to b. Health public policy, policy holders & influencer,
government public health services where services are public and media
free. c. Educational & Ecological diagnosis
c. Those from the higher income groups do not get • Correct answers: Health
sick public policy, policy holders
• Correct answers: They have & influencer, public and
proportionally great number media
of illness and health 24. Green’s framework consists of:
problems . Individual Perception, Modifying factors,
16. The diseases today are largely man-made like Likelihood of axtions
poor sanitation, poor garbage disposal, air pollution, a. Perception of susceptibility,severity, severity,
smoking and utilization of chemicals and pesticides. benefits, costs, motivation & modifying factors
These are classified under: b. Health public policy, policy holders & influencer,
. Political factors public and media
a. Behavioral factors c. Educational & Ecological diagnosis
b. Environmental factors • Correct answers: Educational
• Correct answers: Behavioral & Ecological diagnosis
factors 25. Pender’s theoretical framework consists of:
17. Some of the factors in the eco-system which . Individual Perception, Modifying factors,
affect the level of functioning are oppression, crimes and Likelihood of actions
lack of safety in streets and home which are classified a. Perception of susceptibility, severity, severity,
sunder: benefits, costs, motivation & modifying factors
. Socio-economic b. Health public policy, policy holders & influencer,
a. Heredity public and media
b. Environment c. Educational & Ecological diagnosis
c. Political • Correct answers: Individual
• Correct answers: Political Perception, Modifying
18. Margaret Shetland stated that the philosophy of factors, Likelihood of
community health nursing is based on. actions
. The greatest good for the greatest number 26. Health belief model’s theoretical framework
a. The worth and dignity of man consists of:
b. The rule of the majority . Perception of susceptibility, severity, severity,
c. The “fishing rod effect” benefits, costs, motivation & modifying factors
• Correct answers: The worth a. Health public policy, policy holders & influencer,
and dignity of man public and media
19. The primary focus of community health nursing b. Educational & Ecological diagnosis
practice in terms of maintaining the people’s OLOF is: c. Individual Perception, Modifying factors,
. Health promotion Likelihood of actions
a. Disease prevention • Correct answers: Perception
b. Prevention of communicable diseases of susceptibility, severity,
c. Reduction of lifestyle diseases severity, benefits, costs,
• Correct answers: Health motivation & modifying
promotion factors
20. The philosophy of CHN practice is based on the 27. This is an action in the prevention of the
belief that the family is the smallest unit in a democratic emergence or development of risk factors in population in
society. Which age group should be the priority adults of which they have not yet appeared.
the nurses in the community? . Primordial prevention
. Older persons and terminally ill a. Primary prevention
a. Adolescents and b. Secondary prevention
b. Infants and children c. Tertiary prevention
c. All ages regardless of status • Correct answers: Primordial
• Correct answers: All ages prevention
regardless of status 28. Action directed toward health promotion activities
21. These are used to measure the health status of and specific protection from specific illness is what level
the population. of prevention?
. Health Indicators . Primordial prevention
a. Population Profile a. Primary prevention
b. Secondary prevention 38. Eating a well balanced diet.
c. Tertiary prevention . Primary Prevention
• Correct answers: Primary a. Secondary Prevention
prevention b. Tertiary Prevention
29. What level of prevention is toward disability c. Primordial prevention
limitation and rehabilitation? • Correct answers: Primary
. Primordial prevention Prevention
a. Primary prevention 39. Immunization.
b. Secondary prevention . Primary Prevention
c. Tertiary prevention a. Secondary Prevention
• Correct answers: Tertiary b. Tertiary Prevention
prevention c. Primordial prevention
30. Action which is directed toward early diagnosis • Correct answers: Primary
and treatment of diseases is called: Prevention
. Primordial prevention 40. Self breast examination.
a. Primary prevention . Primary Prevention
b. Secondary prevention a. Secondary Prevention
c. Tertiary prevention b. Tertiary Prevention
• Correct answers: Secondary c. Primordial prevention
prevention • Correct answers: Secondary
31. In Green’s model these includes factors that Prevention
reward the desired behavior change including social 41. Bringing a patient to a specializes health care
support, economic rewards and changing social norms. facility to further the management of his problem.
. Predisposing factors . Primary Prevention
a. Reinforcing factors a. Secondary Prevention
b. Enabling factors b. Tertiary Prevention
• Correct answers: Reinforcing c. Primordial prevention
factors • Correct answers: Tertiary
32. It assesses the policies, resources, Prevention
circumstances and prevailing organizational situations 42. Sleeping 6-8 hours daily.
that could hinder or facilitate the development of the . Primary Prevention
health program.: a. Secondary Prevention
. Educational Diagnosing b. Tertiary Prevention
a. Administrative Diagnosis c. Primordial prevention
b. Environmental Diagnosis • Correct answers: Primary
c. Policy Diagnosis Prevention
• Correct answers: 43. A nurse that delivers services and health and
Administrative Diagnosis safety programs to worker populations, workers and
33. It measures the effectiveness of the program community groups.
based on selecting factors in educational diagnosis and . School nurse
often used to evaluate the performance of educators.: a. Occupational health nurse
. Impact Evaluation b. Community mental health nurse
a. Outcome evaluation • Correct answers:
b. Both Occupational health nurse
c. None of the above 44. A nurse who employs primary prevention by
• Correct answers: Impact providing health education that promotes physical and
Evaluation mental health and informs healthcare decisions, prevents
34. It determines the effect the program had in the disease, and enhances school performance.
health and quality of life of the community.: . School nurse
. Impact Evaluation a. Occupational health nurse
a. Outcome evaluation b. Community mental health nurse
b. Both • Correct answers: School
c. None of the above nurse
• Correct answers: Outcome 45. A group of people with common characteristics or
evaluation interests, beliefs and culture living together within a
35. Premarital counselling. territory or geographical boundary is called:
. Primary Prevention . Demography
a. Secondary Prevention a. Community
b. Tertiary Prevention b. both
• Correct answers: Primary • Correct answers: Community
Prevention 46. CHN services must be available sustainable and
36. Splinting a fractured bone. affordable to all regardless of race, creed, color or socio-
. Primary Prevention economic status is what principle of CHN applicable?
a. Secondary Prevention . CHN integrates health education and counseling
b. Tertiary Prevention as vital parts of functions.
• Correct answers: Tertiary a. Respect for the values, customs and beliefs of the
Prevention clients contribute to the effectiveness of care to the client.
37. Case finding. b. Periodic and continuing evaluation provides the
. Primary Prevention means for assessing the degree to which CHN goals and
a. Secondary Prevention objectives are being attained.
b. Tertiary Prevention c. CHN considers the family as the unit of service.
c. Primordial prevention Its level of functioning is influenced by the degree to which
• Correct answers: Secondary it can deal with its own problems.
Prevention
• Correct answers: Respect for 54. Mr X has been compliant. He has personal belief
the values, customs and in his own ability to do something about his existing health
beliefs of the clients problem is proven beneficial.This is:
contribute to the . Modifying factors
effectiveness of care to the a. Perceived benefit
client. b. Self-efficacy
47. Proving health teachings to the four levels of c. Perceived cost
clientele is what role of the nurse? • Correct answers: Self-efficacy
. Advocate role 55. It is any activity which reduces the burden of
a. Clinician Role mortality or morbidity from diseases.
b. Educator role . Promotion
c. Leader role a. Belief
• Correct answers: Educator b. Prevention
role • Correct answers: Prevention
48. Mr X is manifesting high fever, joint pains, loss of 56. This is a framework for integrating nursing and
appetite and headache for 2 days. He submitted himself behavioral science perspectives on factors influencing
for consultation that such signs and symptoms being felt health behaviors.
is indicative of an illness. What component of the Health . Health Belief Model
Belief Model (HBM) is this? a. PRECEDE-PROCEED Model
. Perceived Susceptibility b. Health Prevention Model
a. Perceived severity c. Health Promotion Model
b. perceived barrier • Correct answers: Health
c. Perceived benefit Promotion Model
• Correct answers: Perceived 57. In educational diagnosis, this refers to selecting
Susceptibility factors which are skills, or personal factors such as
49. The concept of Mr. X to submit himself for service and resources accessibility toward achievement
checkup that what he manifested needs prompt attention of modifying behaviors.:
so it will not predispose him to risk conditions is what . Predisposing factors
component of the HBM? a. Reinforcing factors
. Perceived cost b. Enabling factors
a. Perceived severity • Correct answers: Enabling
b. Perceived susceptibility factors
c. Perceived barrier 58. This is the doing phase of Green's Model where
• Correct answers: Perceived all planned strategies and action are now mobilized.:.
severity . Evaluation
50. What component of the HBM is when Mr X a. Administrative and Policy diagnosis
realized that there are existing hindrances to improved his b. Educational diagnois
health condition? c. Implementation
. Perceived severity • Correct answers:
a. Perceived susceptibility Implementation
b. Perceived barrier 59. This is a specialized practice of nursing that
c. Perceived benefit advances the well-being, academic success, and lifelong
• Correct answers: Perceived achievement and health of students. It is called:
barrier . School nursing
51. Mr. X could not complete such recommended a. Occupational health nursing
treat regimen plan for such health problem manifested for b. Community mental health nursing
it requires longer period of time in its management, more • Correct answers: School
rigid diagnostic examination needed and with financial nursing
constraint. This is: 60. It is also known as industrial nursing. This is:
. Perceived benefit . School nursing
a. Perceived cost a. Occupational health nursing
b. Perceived severity b. Community mental health nursing
c. Perceived susceptibility • Correct answers:
• Correct answers: Perceived Occupational health nursing
cost 61. These determinants of community health that
52. Mr. X find ways and with his significant others includes geography, community size, environment and
worked out for the solution of his existing health problem. industrial development. This:
To further the management of his health condition will . Industrial behaviors
mean good prognosis. This is: a. Physical factors
. Perceived severity b. Community organization
a. Perceived susceptibility c. Sociocultural factors
b. Perceived barrier • Correct answers: Physical
c. Perceived benefit factors
• Correct answers: Perceived 62. These determinants affecting community health
benefit refers to beliefs and traditions, social norms, economy,
53. Mr. X complied with the treatment regimen plan politics and religion.:
as per result of the diagnostic examination done and . Industrial behaviors
toward modifying his existing unhealthy lifestyles. This is: a. physical factors
. Modifying factors b. Community organization
a. Cues to action c. Sociocultural factors
b. self-efficacy • Correct answers:
c. Perceived cost Sociocultural factors
• Correct answers: Cues to
action
63. This special field of nursing is integration of a. Health education and community organizing are
mental health care services and linkage with existing necessary in providing community health services
community health development program.: b. Community health nursing in intended primarily
. School nursing for health promotion and prevention and treatment of
a. Occupational health nursing disease.
b. Community mental health nursing c. The goal of community health nursing is to
• Correct answers: Community provide nursing services to people in their own place of
mental health nursing residence
64. These determinants of community health has to • Correct answers: Health
do with concerted effort of many to make a program work education and community
like herd immunity .: organizing are necessary in
. physical factors providing community health
a. sociocultural factors services
b. community organization 6. In which step of community organizing where the
c. individual behaviors community health worker chose one health issue
• Correct answers: individual to work and formulated actions for solving
behaviors community problems?
65. This health model is a participatory model for . Mobilization
creating successful community health promotion and a. Community organization
other public health interventions.: b. Follow-up/extension
. Health Belief model c. Tentative Program planning
a. Health promotion model • Correct answers: Tentative
b. Health prevention model Program planning
c. Precede-Proceed model 7. The community health nurse takes an active role
• Correct answers: Precede- to initiate community participation. What is the
Proceed model primary goal of community organizing?
. To educate the people regarding community
FINALS health problems
a. To mobilize the people to resolve community
1. CHN is a community-based practice. Which best
health problems
explains this statement?
b. To maximize the community’s resources in
a. The service is provided in the natural
dealing with health problems
environment of people
b. The nurse has to conduct community diagnosis • Correct answers: To mobilize
to determine nursing needs and problems the people to resolve
c. The services are based on the available community health problems
resources within the community 8. An indicator of success in community organizing
d. Priority setting is based on the magnitude of the is when people are able to:
health problems identified . Participate in community activities for the
solution of a community problem
• Correct answers: The nurse
a. Implement activities for the solution of the
has to conduct community
community problem
diagnosis to determine
b. Plan activities for the solution of the community
nursing needs and
problem
problems.
c. Identify the health problem as a common
2. Population- focused nursing practice requires
concern
which of the following processes?
. Community organizing • Correct answers: Participate
a. Nursing, process in community activities for
b. Community diagnosis the solution of a community
c. Epidemiologic process problem
9. Which of the following is a function of
• Correct answers: Community
epidemiology in the following situations?
diagnosis
. Identifying the disease condition based on
3. Which step in community organizing involves
manifestations presented by a client
training of potential leaders in the community?
a. Determining factors that contributed to the
. Integration
occurrence of pneumonia in a 3-year-old
a. Community organization
b. Determining the efficacy of the antibiotic used in
b. Community study
the treatment of the 3-year-old client with pneumonia
c. Core group formation
c. Evaluating the effectiveness of the
• Correct answers: Core group
implementation of the Integrated Management of
formation
Childhood Illness
4. As an epidemiologist, the nurse is responsible
• Correct answers: Evaluating
for reporting cases or notifiable diseases. What
the effectiveness of the
law mandates reporting cases of notifiable
implementation of the
diseases?
Integrated Management of
. Act 3573
Childhood Illness
a. RA 3753
10. Which of the following is an epidemiologic
b. RA 1054
function of the nurse during an epidemic?
c. RA 1082
. Conducting assessment of suspected cases to
• Correct answers: Act 3573
detect the communicable diseases
5. Community health nursing is a developmental
a. Monitoring the condition of the cases affected by
service. Which of the following best illustrates
the communicable disease
this statement?
b. Participating in the investigation to determine the
. The community health nurse continuously
source of epidemic
develops himself personally and professionally
c. Teaching the community on preventive • Correct answers: 2 and 3
measures against the disease 17. Population composition is commonly described
• Correct answers: in terms of:
Participating in the a. Births and deaths
investigation to determine b. Age and sex
the source of epidemic. c. Age and births
11. The primary purpose of conducting an d. Sex and population
epidemiologic investigation is to; • Correct answers: Births and
. Delineate the etiology of the epidemic deaths
a. Encourage cooperation and support of the 18. It refers to the probability reducing, controlling
community and eradicating the identified CHN problem:
b. Identify groups who are at risk of contracting the . Modifiability of the problem
disease a. Preventive potential
c. Identify geographical location of cases of the b. Magnitude of the problem
disease in the community c. Social concern
• Correct answers: Delineate • Correct answers: Modifiability
the etiology of the epidemic of the problem
12. Which is a characteristic of person-to-person 19. Removing the barrier to further the management
propagated epidemic? of the patient health problems is what functions of the
. There are more cases of the disease than health worker?
expected. . care provider
a. The disease must necessarily be transmitted a. health educator
through a vector. b. facilitator
b. The spread of the disease can be attributed to a c. researcher
common vehicle. • Correct answers: facilitator
c. There is gradual build up of cases before we 20. Occurrence of the disease implies the habitual
epidemic becomes easily noticeable. presence of disease in a given geographic location
• Correct answers: There is accounting for the low number of both immunes and
gradual build up of cases susceptible:
before we epidemic . Pandemic
becomes easily noticeable. a. Sporadic
13. The number of cases of Dengue fever usually b. Epidemic
increases towards the end of the rainy season. c. Endemic
This pattern of occurrence of Dengue fever is • Correct answers: Endemic
best described as; 21. A situation that occurs when there is marked
. Epidemic occurrence upward fluctuation in disease incidence in a relatively
a. Cyclical/seasonal variation short period time:
b. Sporadic occurrence . Epidemic
c. Secular occurrence a. Pandemic
• Correct answers: b. Endemic
Cyclical/seasonal variation c. Sporadic
14. One of the roles of a community health nurse in • Correct answers: Epidemic
epidemiology as a supervisor mean: 22. Nurse A measured the severity of the problem
. Referring cases or patients on poor drainage system in terms of proportion of the
a. Providing technical guidance problem on drainage system. What criterion in the
b. Proving nursing care to cases referred priority setting did Nurse A utilize?
c. Formulating and implementing training programs . Nature of the problem
• Correct answers: Providing a. Modifiability of the problem
technical guidance b. Magnitude of the problem
15. Health education is a major activity of a c. Preventive potential
community health nurse. The purpose is to: • Correct answers: Magnitude
. Find out people’s reaction to health services of the problem
they can avail. 23. The following are the most common type of
a. Inform the public of the available programs and instruments of data gathering, except:
services they can avail. . Survey questionnaire
b. Provide information on health promotion. a. Observation checklist
c. Facilitate change in attitudes and behaviors b. Interview guide
towards health. c. None of the above
• Correct answers: Facilitate • Correct answers: None of the
change in attitudes and above
behaviors towards health. 24. A method of COPAR that makes used of the
16. Epidemiology is described with following terms: experimental learning as it emerges it from concrete
1. Study of the occurrence of births and action and which encircle succeeding action:
deaths in a given period of time. . Consciousness raising
2. Concerned with the study of the a. Group centered
probable factors that influence the b. Mass based
development of altered health c. Progressive cycle of action-reflection-action
conditions. • Correct answers:
3. Deals with occurrence and distribution Consciousness raising
of altered health conditions of the 25. A critical step in COPAR that involves actual
population. experiences of the people in confronting the people in
4. Concerned with the number of the powerful and the actual exercise of the people
mortality and morbidity related to age power:
and sex distribution. . Integration
a. Groundwork under 28 days & 3 under 1 year old who died. What will
b. Social investigation be the infant mortality rate?
c. Mobilization of action . 833
• Correct answers: Mobilization a. 83.33
of action b. 33.38
26. The major role of the nurse in CHN is: c. 23.38
. Health Care Provider • Correct answers: 83.33
a. Health Educator 33. Compulsory registration of births and deaths in
b. Health Monitor the local civil registrar’s office:
c. Change Agent . RA# 9288
• Correct answers: Health a. RA# 651
Educator b. RA# 156
27. It is checking the results if objectives are c. RA# 7305
achieved: • Correct answers: RA# 651
. Assessment 34. A big barrier to collaborative goal setting
a. Implementation between the health worker and the community is:
b. Planning . Lack of financial resources.
c. Evaluation a. Inability of the family to perceive the problems.
• Correct answers: Evaluation b. Failure to develop a working relationship
28. Nurse Bek is assigned in community Regta. In between them
planning a community health program in community c. Lack of knowledge
Regta to prevent diarrheal disease in the community, • Correct answers: Failure to
what would be the important strategy to consider to be develop a working
taken by the community health worker? relationship between them
. Campaign for the use of safe water and hand 35. A mother once remarked that her son’s having
washing. intestinal parasitism is nothing but a natural condition for
a. Teach the community how to prepare oresol any other child. Such perception could be categorically
solution. classified as.
b. Distribute anti-diarrheal drugs. . Inability to cast assessment due to unavailability
c. Conduct massive cholera and typhoid of sufficient data.
immunization. a. A serious condition but not requiring immediate
• Correct answers: Campaign action.
for the use of safe water b. A problem with moderate social concern.
and hand washing. c. A problem with low social concern.
29. The ultimate goal of the community health • Correct answers: A problem
services being provided by the people is to: with low social concern.
. Help communities cope up with health threats 36. To encourage community participation, which of
and discontinue life. the following activities should be done?
a. Remote reciprocally supportive relationship . Invite people from civic organization.
between people and environment. a. Refer to the doctor for prescription.
b. Raise the level of health of the citizenry. b. Encourage sponsorships to programs.
c. Increase the level of awareness of illness c. Listen when people relate their problems.
• Correct answers: Raise the • Correct answers: Listen when
level of health of the people relate their
citizenry. problems.
30. As a community health nurse, it is expected that 37. Consider the following statements:
you will: 1. In setting priority for the problems, as
. Utilize indigenous and existing community with the nature of the problem, the least
resources. weight is given to health status state.
a. Work with representatives of existing health and 2. Health resource and health related
health related agencies in the community have the same ranking because of
b. Involves the community members in the clinical urgency.
planning and implementation of health programs. 3. Health related is given the least
c. All of the above. weight.
• Correct answers: All of the 4. The weight of the nature of the
above. problem presented have the score of 1.
31. A key concept of health promotion in working • Correct answers:
with the community is that people in the community are Statements 3 & 4 are true
able to: 38. To ensure effective and efficient data collection
. Access to medical insurance to cover their in the conduct of community diagnosis, the community
expenses. health nurse is concerned about the following important
a. Recognize their health needs and help develop points except for one. Which one is this?
strategies to improve their own health care. a. The need to specify methods of data gathering
b. Stop activities such as smoking or alcohol and tools.
consumption that are harmful to them. b. Identification of the types or kinds of data
c. None of the above needed.
• Correct answers: Recognize c. Identifying problem areas.
their health needs and help d. None of these.
develop strategies to • Correct answers: Identifying
improve their own health problem areas
care. 39. The following are methods of data gathering
32. Barangay C has a total number of deaths of 6, except for one. Which one is this?
has a total population of 2,000 and from these there are . Observation
36 registered live births. There are 2 recorded children a. Record review
b. Diagnostic tests c. Crude death rate
c. Data analysis • Correct answers: Swaroop’s
• Correct answers: Data index
analysis 46. In which step are plans formulated for solving
40. There are three components of the environment community problems?
as part of the triads of disease causation. These are: . Mobilization
1. Physical environment a. Community organization
2. Biological environment b. Follow-up/extension
3. Socio-economic environment c. Core group formation
4. Cultural environment • Correct answers: Community
• Correct answers: 1, 2 and 3 organization
41. Nurse Beh was determining the occurrence and 47. Which of the following is the most prominent
distribution of selected environmental, socio-economic feature of community health nursing?
and behavioral conditions important to disease control . It involves providing home care to sick people
and wellness promotion. What specific stage/step is who are not confined in the hospital.
Nurse Beh in the accomplishment of a community a. Services are provided free of charge to people
diagnosis? within the catchment area.
a. Determining the data to be collected b. The community health nurse functions as part of
b. Collecting data a team providing a public health nursing service.
c. Determining the objectives c. Community health nursing focuses on
d. Defining population promotive, not curative services.
• Correct answers: Determining • Correct answers: Community
the objectives health nursing focuses on
42. The community health nurse takes an active role promotive, not curative
in community participation. What is the primary goal of services.
community organizing? 48. According to Margaret Shetland, the philosophy
. To educate the people regarding community of public health nursing is based on which of the
health problems following?
a. To mobilize the people to resolve community . Health and longevity as birthrights.
health problems a. The mandate of the state to protect the
b. To maximize the community’s resources in birthrights of its citizens.
dealing with health problems b. Public health nursing as a specialized field of
c. All of the above nursing.
• Correct answers: To c. The worth and dignity of man.
maximize the community’s • Correct answers: The worth
resources in dealing with and dignity of man
health problems 49. Nurse Bang determines whether resources were
43. In setting the priority of identified community maximized in implementing Ligtas Tigdas, she is
health, Nurse Oui assessed and gathered the resident’s evaluating:
perception on the problem of poor nutritional status of . Effectiveness
children in the. community as they were affected by this a. Efficiency
problem. Oui is actually using what criterion in the b. Adequacy
priority setting of identified community health nursing c. Appropriateness
problem? • Correct answers: Efficiency
. Magnitude of the problem 50. RA 7160 mandates devolution of basic services
a. Preventive potential from the national government to local government units.
b. Modifiability of the problem Which of the following is the major goal of devolution?
c. Social concern . To strengthen local government units.
• Correct answers: Social a. To allow greater autonomy to local government
concern units.
44. Which of the following reasons best explains b. To empower the people and promote their self-
why the community is seen as the primary client by the reliance.
nurse in assessing the community health needs? c. To make basic services more accessible to the
. The community has an indirect influence on the people.
health of the individuals, families and sub populations. • Correct answers: To empower
a. It is in the community as the center of society the people and promote
where services are rarely rendered. their self-reliance.
b. Cultures that abound in community may or may 51. Nurse bang decided to start data collection in
not be changed as the needs arise during health the conduct of community diagnosis. Which type of
promotion. approach of gathering data will provide her with the best
c. The community consists of groups of family with opportunity to observe family dynamics in the
various health problems to deal with as it affects the community?
whole community. . Telephone interview as most accessible
• Correct answers: The a. Group conferences
community consists of b. House to house visit
groups of family with c. Survey with questionnaires
various health problems to • Correct answers: House to
deal with as it affects the house visit
whole community. 52. Data collection has been done by Nurse Bang.
45. Determining mortality rate as to the percentage The category of community health nursing problems is
of the deaths aged 50 years or older is: used in the statement of nursing problems in the care of
. Proportional mortality rate community. The youngest child of the delos Reyes family
a. Swaroop’s index has been diagnosed as mentally retarded. This is
b. Specific death rate classified as:
. Health related • Correct answers: 1–4-year-old
a. Health resource age-specific mortality rate
b. Health status 61. Which of the data be considered and being used
c. All of the above in computing general fertility rate?
• Correct answers: Health . Estimated midyear population
status a. Number of registered live births
53. It is a quantitative and qualitative description of b. Number of pregnancies in the year
the health stats of a given population and factors that c. Number of females of reproductive age
influence their health.: • Correct answers: Number of
. Epidemiology registered live births
a. Demography 62. You are assigned to gather data for nutritional
b. Community diagnosis assessment of Purok 2 in Batangbatang, Victoria,
c. All of the above Tarlac. You will gather information only from families with
• Correct answers: community members who belong to the target population. What
diagnosis method of data gathering is best for this purpose?
54. Which type of community diagnosis focuses in . Census
working out a particular community health problem? a. Survey
. Comprehensive b. Record Review
a. Problem-oriented c. Review of civil registry
b. Both • Correct answers: Survey
• Correct answers: Problem- 63. In the conduct of a census, the method of
oriented population assignment based on the actual physical
55. In the investigation of an epidemic, you compare location of the people is termed;
the present frequency of the disease with the usual . De jure
frequency at this time of the year in the community. This a. De locus
is done during which stage of the investigation? b. De facto
. Establishing the epidemic c. De novo
a. Testing the hypothesis • Correct answers: De facto
b. Formulation of the hypothesis 64. Which of the following professionals can sign the
c. Appraisal of facts birth certificate who attended actually the delivery?
• Correct answers: Establishing . Public health nurse
the epidemic a. Rural health midwife
56. To describe the sex composition of the b. Municipal health officer
population, which demographic tool may be used? c. Any of these health professionals
. Sex ratio • Correct answers: Any of
a. Sex proportion these health professionals
b. Population pyramid 65. You are now to rank identified community health
c. Any of these maybe used. problems in your target community. Which criterion in
• Correct answers: Any of priority setting of health problems is used only in
these maybe used. community health care?
57. Which of the following is a natality rate? . Modifiability of the problem
. Crude birth rate a. Nature of the problem presented
a. Neonatal mortality rate b. Magnitude of the health problem
b. Infant mortality rate c. Preventive potential of the health problem
c. General fertility rate • Correct answers: Magnitude
• Correct answers: Crude birth of the health problem
rate 66. Which of the following is not included as
58. You are computing the crude rate of your demographic variables?
municipality, with a total population of about 18,000 for . Population projection
last year. There were 52 deaths. Among those who died, a. Patterns of migration
20 died because of diseases of the heart and 32 were b. Age and sex distribution
aged 50 years or older. What is the crude death rate? c. Educational level
. 2.9/1000 • Correct answers: Educational
a. 9.2/1000 level
b. 2.3/1000 67. To consider data to be collected in the
c. 5.1/1000 community, there are variables to identify. These
• Correct answers: 2.9/1000 variables in comprehensive community diagnosis may
59. In the past year, Barangay A had an average break up the people into groups within the community.
population of 1655. 46 babies were born in that year, 2 Which of the following are these?
of whom died less than 4 weeks after they were born. 1. ethnic group, religious sect, race
They were 4 recorded stillbirths. What is the neonatal 2. political orientation, dialect or
mortality rate? language
. 27.8/1000 3. physical and topographical
a. 43.5/1000 characteristics
b. 86.9/1000 4. communication network, social class,
c. 130.4/1000 employment status
• Correct answers: 43.5/1000 5. morbidity and mortality rates.
60. Nurse Nah Gahget conducted community • Correct answers: 1, & 2 only
assessment of Barangay A. Which statistic best reflects 68. In health & illness patterns, these incudes:
the nutritional status of a population? a. ethnic group, religious sect, race
. 1–4-year-old age-specific mortality rate b. communication network, social class,
a. Proportionate mortality rate employment status
b. Infant mortality rate c. physical and topographical characteristics
c. Swaroop’s index d. morbidity and mortality rates
• Correct answers: morbidity
and mortality rates
69. This reflects the action potential of the state and
its people to address the health needs and problems of
the community.:
. Political & Leadership patterns
a. Health & Illness Patterns
b. Demographic variables & Cultural variables
c. Socioeconomic & health resources
• Correct answers: Political &
Leadership patterns
70. It is the process of examining data to define
needs strengths, barriers, opportunities, readiness and
resources.:
. Planning
a. Implementation
b. Community analysis
c. Priority setting
• Correct answers: Community
Analysis

You might also like